Download as pdf or txt
Download as pdf or txt
You are on page 1of 171

CHAPTER

10
CAPITAL BUDGETING - BASIC
Question 1] What do you understand by "Capital Budgeting"?
Ans.: Capital budgeting is the process by which the firm decides which long-term investments to make.
Capital budgeting projects, i.e., potential long-term investments, are expected to generate cash flows over
several years. The decision to accept or reject a capital budgeting project depends on an analysis of the
cash flows generated by the project and its cost.
Capital Budgeting Projects:
(1) Independent Project: It is a project whose cash flows are not affected by the accept/reject decision
for other projects. Thus, all independent projects which meet the capital budgeting criterion should be
accepted.
(2) Mutually Exclusive Projects: They are a set of projects from which at most one will be accepted.
For example, a set of projects which are to accomplish the same task. Thus, when choosing between
“Mutually Exclusive Projects” more than one project may satisfy the capital budgeting criterion. However,
only one, i.e., the best project can be accepted.
The firm's cost of capital (Ko) is the discount rate which should be used in capital budgeting. The cost of
capital reflects the firm’s cost of obtaining capital to invest in long term assets. Thus it reflects a weighted
average of the firm's cost of debt, cost of preference shares and cost of equity shares.
Question 2] What is payback period?
Payback period is suitable to measure liquidity of a project rather than its profitability. Comment.
CS (Professional) - June 2016 (5 Marks)
Ans.: The payback period represents the amount of time that it takes for a capital budgeting project to
recover its initial cost. The use of the payback period as a capital budgeting decision rule specifies that all
independent projects with a payback period less than a specified number of years should be accepted.
When choosing among mutually exclusive projects, the project with the quickest payback is preferred.
Merits:
(1) It is simple to apply, easy to understand.
(2) In case of capital rationing, a company is compelled to invest in projects having shortest payback
period.
(3) This method is most suitable when the future is very uncertain. The shorter the payback period, the
less risky is the project. Therefore, it can be considered as an indicator of risk.
(4) This method gives an indication to the prospective investors specifying when their funds are likely
to be repaid.
(5) It does not involve assumptions about future interest rates.
(6) Ranking projects according to their ability to repay quickly may be useful to firms when experiencing
liquidity constraints. They will need to exercise careful control over cash requirements.
Demerits:
(1) It does not indicate whether an investment should be accepted or rejected, unless the Payback
period is compared with an arbitrary managerial target.
(2) The method ignores cash generation beyond the payback period and this can be seen more a
measure of liquidity than of profitability.
(3) It fails to take into account the timing of returns and the cost of capital.
(4) The traditional payback approach does not consider the salvage value of an investment. It fails to
determine the payback period required in order, to recover the initial outlay if things go wrong.
(5) This method makes no attempt to measure a percentage return on the capital invested and is often
used in conjunction with other methods.
Illustration: Form the following details calculate the payback period and rank the projects.
Particulars Project A Project B
Initial investment 1,00,000 1,20,000
Cash flow after tax:
Year 1 50,000 60,000
Year 2 40,000 25,000
Year 3 20,000 30,000
Year 4 10,000 20,000
Year 5 5,000 8,000
Solution:
Project A Project B
Year CFAT Cumulative CFAT Cumulative
CFAT CFAT
1 50,000 50,000 60,000 60,000
2 40,000 90,000 25,000 85,000
3 20,000 1,10,000 30,000 1,15,000
4 10,000 1,20,000 20,000 1,35,000
5 5,000 1,25,000 8,000 1,43,000

In case of Project A ` 90,000 are recovered in 2nd year. We have short fall of ` 10,000. In third year CFAT
is ` 20,000; so proportionate period of 3rd year has to be computed. For ` 20,000 - 12 months For ` 10,000
-?
10,000x12/20,000 = 6 months
Thus, payback period of Project A is 2 years & 6 months.
Alternatively,
Payback period or Project A = 2 years + 1,00,000-90,000/20,000 X 12
= 2 years & 6 months
Payback period of Project B = 3 years + 1,20,000- 1,15,000/20000 X 12
= 3 years -& 3 months
Analysis: Project A should be accepted as its payback period is less than payback period of Project B.
Question 3] Write a short note on: Discounted Payback Period
Ans,: One of the major disadvantages of simple payback period is that it ignores the time value of money.
To counter this limitation, an alternative procedure called discounted payback period may be followed,
which accounts for time value of money by discounting the cash inflows of the project.
In discounted payback period we have to calculate the present value of each cash inflow taking the start
of the first period as zero point. For this purpose the management has to set a suitable discount rate.
Illustration: Form the following details calculate the discounted payback period and rank the projects.
Particulars Project A Project B
Initial investment 3,37,500 6,00,000
Cash flow after tax:
Year 1 - 1,50,000
Year 2 75,000 2,10,000
Year 3 3,30,000 2,40,000
Year 4 2,10,000 2,55,000
Year 5 2,10,000 2,25,000
Company’s cost of capital is 10%. Solution:
Year CFAT PV Project A Project B
Project A Project B Factor PV Cumulative PV Cumulative
10% PV PV
1 75,000 1.50,000 0.909 61,950 61,950 1,36,350 1,36,350
2 3.30,000 2.10,000 0.826 2,47,830 1,73,460 3,09,810
3 2.10,000 2.40,000 0.751 3,09,780 1,80,240 4,90,050
4 2,10,000 2.55,000 0.683
4,53,210 6,64,215
5 2.25,000 0.621
1,43,430 5,83,620 1,74,165 8,03,940

1,30,410 1,39,725
Discounted payback period:
Project A = 3 years + 3,37,500 - 3,09,750/1,43,430 x 12
= 3 years & 2.32 months
Proiect B =3 years + 6,00,000 - 4,90,050/1,74,165 X 12
= 3 years & 7.58 months
Analysis: Project A should be accepted as its discounted payback period is less than discounted payback
period of Project B.
Question 4] What do you understand by 'payback period reciprocal'? How it is calculated?
Ans.: An alternative way of expressing the payback period is as the ‘Payback Period Reciprocal’ which is
expressed as:
Payback period reciprocal = 1/Payback Period X 100
Thus, if a project has a payback period of 2.5 years, then the payback period reciprocal would be:
1/2.5 x 100 =40%
Question 5] Write a short note on: Accounting Rate of Return Method (ARR)
Ans.: The Accounting rate of return (also known as return on investment or return on capital employed)
method employ the normal accounting technique to measure the increase in profit expected to result
from an investment by expressing the net accounting profit arising from the investment as a percentage
of that capital investment. In this method, most often the following formula is applied to arrive at the
accounting rate of return.
Accounting rate of return (ARR) = Average PAT/Initial Investment x 100
Average investment = Initial Investment + Salvage Value/2
Sometimes, initial investment is used in place of average investment. Of the various accounting rates of
return on different alternative proposals, the one having highest rate of return is taken to be the best
investment proposal. For example, in three alternative proposals A, B and C with expected accounting
rates of return of 10%, 20% and 18% respectively. Projects will be selected in order of B, C and A. If the
prevailing rates of interest is taken to be 15% p.a., only proposals B and C will qualify for consideration
and in that order.
Merits:
(1) It is easy to calculate because it makes use of readily available accounting information.
(2) It is not concerned with cash flows, but rather based upon profits, which are reported in annual
accounts and sent to shareholders.
(3) Unlike payback period method, this method does take into consideration all the years involved in
the life of a project.
(4) Where a number of capital investment proposals are being considered, a quick decision can be
taken by use of ranking the investment proposals.
(5) If high profits are required, this is certainly a way of achieving them.
Demerits:
(1) It does not take into accounting time value of money.
(2) It fails to measure properly the rates of return on a project even if the cash flows are even over the
project life.
(3) This method fails to distinguish the size of investment required for individual projects. Competing
investment proposals with the same accounting rate of return may require different amounts of
investment.
(4) It is biased against short-term projects in the same way that payback is biased against longer-term
ones.
(5) Several concepts of investment are used for working out accounting rates of return. Thus there is
no full agreement on the proper measure of the term investment. Thus, different managers have different
meanings when they refer to accounting rate of return.
(6) The accounting rate of return does not indicate whether an investment should be accepted or
rejected, unless the rate of return is compared with the arbitrary management target. It measures the
returns in relation to the outlay and does not evaluate the absolute worth of the returns.
Illustration: Form the following details calculate the ARR and rank the projects.
Particulars Machine A MachineB
Initial investment 8,75,000 8,75,000
Cash flow after tax:
Year 1 2,62,500 87,500
Year 2 3,50,000 2,62,500
Year 3 4,37,500 3,50,000
Year 4 2,62,500 5,25,000
Year 5 1,75,000 3,50,000
Machine has to written off over the period of 5 years by straight line method. Solution:
Particulars Machine A Machine B
Total CFAT 14,87,500 15,75,000
(-) Depreciation for 5 years (8,75,000) (8,75,000)
PAT 6,12,500 7,00,000
Average PAT 1,22,500 1,40,000
ARR = Average PAT/1,22,500 x Initial investment
Machine A = 1,22,500/8,75,000 X 100 = 14%
Machine B = 1,40,000/8,75,000 X 100 = 16%
Analysis: Machine B should be selected as it ARR is high than Machine A.
Question 6] Write a short note on: Net Present Value (NPV) Method
Ans.: The Net Present Value (NPV) of a capital budgeting project indicates the expected impact of the
project on the value of the firm. Projects with a positive NPV are expected to increase the value of the
firm. Thus, the NPV decision rule specifies that all independent projects with a positive NPV should be
accepted. When choosing among mutually exclusive projects, the project with the largest (positive) NPV
should be selected. The NPV is calculated as the present value of the project’s cash inflows minus the
present value of the project’s cash outflows.
Merits:
(1) It is based on the assumption that cash flows, and hence dividends, determine shareholders' wealth.
(2) Cash flows are subjective than profits.
(3) It recognizes the time value of money.
(4) It considers the total benefits arising out of proposals over its lifetime.
(5) The future discount rate normally varies due to longer time span. This rate can be applied in
calculating the NPV by altering the denominator.
(6) This method is particularly useful for the selection of naturally exclusive projects. (In mutually
exclusive projects acceptance of one project tantamount to rejection of the other project).
(7) This method of project selection is instrumental in achieving the financial objective, i.e., the
maximization of the shareholders wealth.
Demerits:
(1) It is difficult to calculate as well as understand it as compared to accounting rate of return method
or payback method.
(2) Calculation of the desired rates of return presents serious problems. Generally cost of capital is the
basis of determining the desired rate. The calculation of cost of capital is itself complicated. Moreover,
desired rates of return will vary from year to year.
(3) This method is an absolute measure when two projects are being considered, this method will
favour the project, which has higher NPV.
(4) This method may not give satisfactory results where two projects having different effective lives are
being compared. Normally, the project with shorter economic life is preferred, if other things are equal.
This method does not attach importance to the shorter economic life of the project.
(5) This method emphasizes the comparison of net present value and disregards the initial investment
involved. Thus, this method may not give dependable results.
The example below illustrates the calculation of Net Present Value.
Consider Capital Budgeting projects A and B which yield the following cash flows over their five year lives.
The cost of capital for the project is 10%.
Project A Project B
Year CFAT CFAT
0 1,00,000 1,00,000
1 50,000 10,000
2 40,000 20,000
3 20,000 20,000
4 20,000 40,000
5 10,000 70,000
Calculate NPV of each project and rank them. Solution:
CFAT PV Factor Present Value
Year
Project A Project B 10% Project A Project B
1 50,000 10,000 0.909 45,450 9,090
2 40,000 20,000 0.826 33,040 16,520
3 20,000 20,000 0.751 15,020 15,020
4 20,000 40,000 0.683 13,660 27,320
5 10,000 70,000 0.621 6,210 43,470
Total present value 1,13,380 1,11,420
(-) Initial investment (1,00,000) (1,00,000)
Net Present Value (NPV) 13,380 11,420
Analysis: Project A should be selected as it NPV is higher than Project B.
Question 7] Write a short note on: Profitability Index
Write a short note on: Desirability Factor
Ans.: Profitability index is an investment appraisal technique calculated by dividing the present value of
future cash flows of a project by the initial investment required for the project.
FORMULA :
Profitability Index = Total present value/Initial investment
Profitability index is actually a modification of the net present value method. While present value is an
absolute measure (i.e. it gives as the total rupee figure for a project), the profitability index is a relative
measure (i.e. it gives as the figure as a ratio).
Decision Rule: Accept a project if the profitability index is greater than 1, stay indifferent if the profitability
index is one and don’t accept a project if the profitability index is below 1.
Profitability index is sometimes called benefit-cost ratio too and is useful in capital rationing since it helps
in ranking projects based on their per rupee return.
Question 8] What is "internal rate of return"? State its acceptance rule.
Internal Rate of Return (IRR) of a project is that rate where Net Present Value (NPV) is zero. Comment.
CS (Professional) - June 2013 (5 Marks)
Ans.: The Internal Rate of Return (IRR) of a capital budgeting project is the discount rate at which the Net
Present Value (NPV) of a project equals zero. The IRR decision rule specifies that all independent projects
with an IRR greater than the cost of capital should be accepted. When choosing among mutually exclusive
projects, the project with the highest IRR should be selected (as long as the IRR is greater than the cost of
capital).
Acceptance rule: The use of IRR, as a criterion to accept capital investment decision involves a comparison
of IRR with the required rate of return known as cut-off-rate. If IRR is greater than cut-off-rate, the project
should be accepted. If IRR is less than the cut-off-rate, the project is rejected. In case, the IRR is equal to
the cut-off-rate, the firm is indifferent.
Merits:
(1) It considers the time value of money.
(2) It takes into account the total cash inflows and cash outflows.
(3) It is easier to understand: for example, if told that IRR of an investment is 20% against the desired
rate of an investment is ` 15,3%.
Demerits:
(1) It does not use the concept of desired rate of return, whereas it provides the rate of return, which
is indicative of the profitability of investment proposal.
(2) It involves tedious calculations, based on trial and error method.
(3) It produces multiple rates, which can be confusing.
(4) Projects selected based on higher IRR may not be profitable.
(5) Unless the life of the project can be accurately estimated, assessment of cash flows cannot be
correctly made.
(6) Single discount rate ignores the varying future interest rates.
Consider Capital Budgeting projects A and B which yield the following cash flows over their five year lives.
The cost of capital for the project is 10%..
Project A Project B
Year Cash Flow Cash Flow
0 1,00,000 1,00,000
1 50,000 10,000
2 40,000 20,000
3 20,000 20,000
4 20,000 40,000
5 10,000 70,000
Calculate IRR of each project and rank them.
Calculation of IRR of Project A:
Factor to be located = Initial Investment/Average CFAT = 1,00,000/28,000 =3.571
Looking at annuity table in 5 years row 12% rate is selected.
Since NPV at 12% is positive next higher rate 17% is taken.
PV Factor Present Value
Year CFAT
12% 17% 12% 17%
1 50,000 0.893 0.855 44,650 42,750
2 40,000 0.797 0.731 31,880 29,240
3' 20,000 0.712 0.624 14,240 12,480
4 20,000 0.636 0.534 12,720 10,680
5 10,000 0.567 0.456 5,670 4,560
Total present value 1,09,160 99,710
(-) Initial investment (1,00,000) (1,00,000)
Net Present Value (NPV) 9,160 (290)
IRR 12 + 9,160/9,160 + 290 X 5
= 12 + 9,160/9,450 X 5
= 12+ 4.85
= 16.85%
Calculation of IRR of Project B:
Factor to be located = Initial Investment/Average CFAT = 1,00,000/32,000 =3.125
Looking at annuity table in 5 years row 18% rate is selected.
Since NPV at 18% is negative next lower rate 12% is taken, (you can taken any lower rate)
PV Factor Present Value
Year CFAT
18% 12% 18% 12%
1 10,000 0.847 0.893 8,470 8,930
2 20,000 0.718 0.797 14,360 15,940
3 20,000 0.609 0.712 12,180 14,240
4 40,000 0.516 0.636 20,640 25,440
5 70,000 0.437 0.567 30,590 39,690
Total present value 86,240 1,04,240
(-) Initial investment (1,00,000) (1,00,000)
Net Present Value (NPV) (13,760) 4,240
IRR = 12 +4,240/4,240 + 13,760 X 6
= 12 + 4,240/18,000 x 6
= 12 + 1.41
= 13.4196
Analysis: Project A should be selected as it IRR is higher than Project B.
Question 9] Define: Modified internal rate of return method.
Ans.: There are several limitations attached with the concept of the conventional IRR. The MIRR addresses
some of these deficiencies. For example, it eliminates multiple IRR rates; it addresses the reinvestment
rate issue and produces results, which are consistent with the NPV method.
Under this method, all cash flows, apart from the initial investment,- are brought to the terminal value
using an appropriate discount rate (usually the cost of capital). This results in a single stream of cash inflow
in the terminal year. The MIRR is obtained by assuming a single outflow in the zeroth year and the terminal
cash in flow as mentioned above. The discount rate which equates the present value of the terminal cash
in flow to the zeroth year outflow is called the MIRR.
FV ( positive cash flows, reinvestment rate)
MIRR =
− PV ( Negative cash flow finance rate)
Question 10] Explain the concept of multiple rate of return.
Ans.: In cases where project cash flows change signs or reverse during the life of a project for example,
an initial cash outflow is followed by cash inflows and subsequently followed by a major cash outflow,
there may be more than one internal rate of return (IRR). The following graph of discount rate versus net
present value (NPV) may be used as an illustration:

In such situations if the cost of capital is less than the two IRRs, a decision can be made easily, however,
otherwise the IRR decision rule may turn out to be misleading as the project should only be invested if the
cost of capital is between IRR1 and IRR2. To understand the concept of multiple IRRs it is necessary to
understand the implicit reinvestment assumption in both NPV and IRR techniques.
Question 11] If NPV Method & IRR Method gives conflicting results, under which method decision is
more appropriate?
Ans.: Using NPV, all projects with a positive net present value, when discounted at the firms cost of capital,
will be accepted. Using IRR all projects, which yield an internal rate of return in excess of the firms, cost
of capital will be chosen.
However, although IRR and NPV lead to the same conclusion regarding project acceptability, the ranking
of a set of projects obtained from IRR does not necessarily agree with that produced using NPV. Since, in
the latter case, the ranking may vary according to particular discount rate used.
Argument about the merits of the relative rankings in simple accept/reject situations is thus concerned
with the question of value. It is argued that the IRR measures only the quality of the investment while NPV
takes into account both the quality and the scale. This is because the IRR provides a relative measure of
value (% IRR) while the NPV provides an absolute measure (` surplus). Thus, the IRR would rank, for
example: a 100% return on an investment of ` 1 considerably higher than a 20% return on an investment
of ` 10 lakhs, whereas the reserve would be true using NPV (as long as the cost of capital is below 20%).
While one project may have a higher rate of profit per unit of capital invested than another, if it has fewer
units of capital invested in it, it may make a smaller contribution to the wealth of the firm. Thus, if the
objective is to maximize the firm’s wealth, then the ranking of project NPVs provides the correct
measures. If the objective is to maximize the rate of profitability per unit of capital invested, then IRR
would provide the correct ranking of .projects, but this objective could be achieved by rejecting all but the
most highly profitable projects. This is clearly unrealistic and, therefore, one would conclude that NPV
ranking is correct and IRR unsatisfactory as a measure of relative project value.
Question 12] What do you mean by capital rationing? Illustrate some of its advantages.
Write a short note on: Capital Rationing
CS (Professional) - Dec 2000 (4 Marks), June 2007 (5 Marks) CS (Professional) - June 2010 (5 Marks)
The device of capital rationing is adopted to control capital expenditure. Comment.
CS (Professional) - Dec 2013 (5 Marks)
Ans.: Capital rationing refers to a situation where a company cannot undertake all positive NPV projects,
it has identified because of shortage of capital. Under this situation, a decision maker is compelled to
reject some of the viable projects having positive net present value because of shortage of funds. It is
known as a situation involving capital rationing.
Illustration: A Ltd. has an investment budget of ` 25 lakhs for next year. It has under consideration three
projects A, B & C (B & C are mutually exclusive) and all of them can be completed within a year. Further
details are given below:

Project Investment required (` in lakhs) Net present value


A 14 5.6
B 12 7.2
C 10 5.0
Recommend the best policy to utilize the investment budget, supported by proper reasoning. Solution:
Investment allocation strategy under capital rationing situation:
Project Investment NPV Comments

A&B 26 12.8 Funds available are only ` 25 lakhs and this proposal is not possible to implement.
B& C 22 12.2 Since B & C are mutually exclusive, it is not possible to take B & C simultaneously.
A&C 24 10.6 The possible option is only to take up A & C even though NPV is lowest.
Capital rationing may be effected through budget ceiling. A firm may resort to capital rationing when it
follows the policy of financing investment proposals only by ploughing back its retained earnings. In that
case, capital expenditure in a given period cannot exceed the amount of retained earnings available for
reinvestment. Management may also introduce capital rationing when a department is authorized to
make investments up to a limit beyond which investment decisions will be made by higher level
management.
Capital rationing may result in accepting several small investment proposals then accepting a few large
investment proposals so that there may be full utilization of budget ceiling. This may result in accepting
relatively less profitable investment proposals if full utilization of budget is a primary consideration.
Similarly, capital rationing also means that the firm foregoes the next most profitable investment falling
after the budget ceiling even though it is estimated to yield a rate of return much higher than the required
rate of return. Thus, capital rationing does not lead optimum results.
There are few advantages of practicing capital rationing:
(1) Budgeting: The first and an important advantage is that capital rationing introduces a sense of strict
budgeting of corporate resources of a company. Whenever there is an injunction of capital in the form of
more borrowings or stock issuance capital, the resources are properly handled and invested in profitable
projects.
(2) Less wastage: Capital rationing prevents wastage of resources by not investing in each and every
new project available for investment.
(3) Fewer projects: Capital rationing ensures that limited numbers of projects are selected by imposing
capital restrictions. This helps in keeping the number of active projects to minimum and thus
manages them well.
(4) Higher returns on investments: Through capital rationing, companies invest only in projects where
the expected return is high, thus eliminating projects with lower returns on capital.
(5) Stability: As the company is not investing in every project, the finances are not over-extended. This
helps in having adequate finances for tough times and ensures more stability and increase in the stock
price of the company.
Question 13] Under capital rationing, the standard net present value (NPV) decision rule no longer holds
true. Comment.
CS (Professional) - Dec 2009 (5 Marks), June 2012 (5 Marks)
Ans.: The term capital rationing refers to the situation where the funds available to a firm are limited. The
implicit assumption within the NPV decision rule does not hold true i.e. a firm cannot accept all positive
NPV projects because of shortage of capital. Capital rationing is of two types - Hard and Soft, characterized
by sources of capital expenditure constraints. Hard capital rationing occurs when constrains is externally
imposed while soft rationing occurs, when constraints are imposed internally by management. As in the
capital rationing NPV decision rule does not hold true, so it has to be modified. In single period rationing
condition, the simple NPV decision rule is replaced by feasibility set approach or benefit cost ratio analysis
and in multi-period capital rationing the optimal solution is obtained through linear programming.
Question 14] Capital rationing does not always lead to optimum results. Comment.
CS (Professional) - Dec 2011 (5 Marks), Dec 2016 (4 Marks)
Ans.: A Firm may fix the maximum amount that can be invested in capital projects, during a given period
of time. The firm then attempts to select a combination of investment proposals that will be within the
specific limits providing maximum profitability and put them in descending order according to their rate
of return. Capital rationing may result in accepting several small investment proposals than a few large
investment proposals so that there may be full utilization of budgeted amount. This may result in
accepting relatively less profitable investment proposals if full utilization of budget is a primary
consideration.
Similarly, capital rationing may also mean that the firm foregoes the next most profitable investment
following after the budget ceiling even though it is estimated to yield a rate of return much higher than
the required rate of return. Thus capital rationing does not always lead to optimum results.
Question 15] Distinguish between: Capital Budgeting & Capital Rationing
CS (Professional) - Dec 2014 (4 Marks), Dec 2015 (5 Marks)
CS (Executive) - Dec 2018 (5 Marks)
Ans.: Following are the main points of distinction between capital budgeting & capital rationing:
Points Capital Budgeting Capital Rationing
Meaning Capital budgeting is the process by which the firm Capital rationing refers to a situation where a
decides which long-term investments to make. company cannot undertake all positive NPV
projects, it has identified because of shortage of
capital.
Points Capital Budgeting Capital Rationing
Acceptance of As per capital budgeting process project having Under capital rationing situation, a decision
projects positive is selected. maker is compelled to reject some of the viable
projects having positive net present value
because of shortage of funds.
Emphasis It emphasizes on profitable projects. It emphasizes on effective utilization of short
funds.
Question 16] Distinguish between: Internal rate of return & Accounting rate of return
Ans.: Following are the main points of distinction between IRR & ARR method:
Points Internal rate of return (IRR) Accounting rate of return (ARR)
Meaning Internal rate of return is a percentage discount The Accounting rate of return method employ
rate used in capital investment appraisals, which the normal accounting technique to measure the
brings the cost of a project and its future cash increase in profit expected to result from an
inflows into equality. It is the rate of return, investment by expressing the net accounting
which equates the present value of anticipated profit arising from the investment as a
net cash hows with the initial outlay. The IRR is percentage of that capital investment.
also defined as the rate at which the net present
value is zero.
Firm/Period IRR is firm specific. ARR is period specific.
Calculation It is difficult to calculate. It is easy to calculate.
Time value of It does not take into account Time Value of
It takes into account Time Value of Money
money Money.
Emphasis It emphasizes on cash flow. It emphasizes on profits.
Question 17] Distinguish between: NPV Method & IRR Method.
CS (Professional) - Dec 2005 (4 Marks), June 2016 (4 Marks)
Ans.: Following are the main point of distinction between NPV & IRR Method
Points Net Present Value Method (NPV) Internal Rate of Return Method (IRR)
Meaning Net present value is obtained by discounting all Internal rate of return is a percentage discount
cash outflows and inflows attributable to a capital rate used in capital investment appraisals, which
investment project by a chosen percentage e.g., brings the cost of a project and its future cash
the entity’s weighted average cost of capital. The inflows into equality. It is the rate of return, which
method discounts the net cash flows from the equates the present value of anticipated net cash
investment by the minimum required rate of flows with the initial outlay. The IRR is also
return, and deducts the initial investment to give defined as the rate at which the net present value
the yield from the funds invested. If yield is is zero.
positive, the project is acceptable. If it is negative
the project in unable to pay for itself and is thus
unacceptable.
It does not take into consideration cost of capital
Cost of capital It uses discount rate which is firms cost of capital
(Ke) (K)
This method is difficult to understand and
Calculation This method is easy to understand and calculate.
calculate.
Cash inflow & This Method does not take into consideration
This method considers cash inflow & outflow.
outflow cash inflow & outflow.
NPV Method does not suffer from the limitation of
Multiple rate IRR may give negative rate or multiple rates.
multiple rates.
Question 18] Distinguish between: Net Present Value & Profitability Index
CS (Professional) - June 2008 (5 Marks)
Ans.: Following are the main point of distinction between net present value & profitability index:
Points Net Present Value (NPV) Profitability Index (PI)
Meaning Net present value is obtained by discounting all Profitability index is an investment appraisal
cash outflows and inflows attributable to a capital technique calculated by dividing the present
investment project by a chosen percentage e.g., value of future cash flows of a project by the
the entity’s weighted average cost of capital. initial investment required for the project.

Formula NPV = Total present value - Initial investment Profitability Index = Total present value/Initial
investment
Acceptance of As per this method, project having positive NPV As per this method, project having profitability
project are selected. index above one are selected.
Question 19] Discuss the need for social cost benefit analysis.
Ans.: Several hundred crores of rupees are committed every year to various public projects. Analysis of
such projects has to be done with reference to social costs and benefits, since they cannot be expected
to yield an adequate commercial return on the funds employed, at least during the short run. Social cost
benefit analysis is important for the private corporations also who have a moral responsibility to
undertake socially desirable projects.
The need for social cost benefit analysis arises due to the following:
(i) Market prices are used to measure costs and benefits in project analysis; they do not represent social
values due to imperfections in the market.
(ii) Monetary cost benefit analysis fails to consider the external effects of the project. The external effects
can be positive like development of infrastructure or negative like pollution and imbalance in
environment.
(iii) Taxes and subsidies are transfer payments and hence ignored in social benefit cost analysis.
(iv) The SCBA is essential for measuring the redistribution effect of benefits of project, as benefits going
to poorer section are more important than going to sections which are economically better off.
(v) Projects manufacturing tobacco products are not distinguished from those generating power or
producing necessities of life. Thus merit wants are important appraisal criterion for SCBA.
Question 20] Write a short note on: Importance of capital budgeting
Ans.: Capital budgeting decisions are of paramount importance in financial decision. So it needs special
care on account of the following reasons:
(1) Long-term Implications: A capital budgeting decision has its effect over a long time span and
inevitably affects the company’s future cost structure and growth. A wrong decision can prove disastrous
for the long-term survival of firm. It leads unwanted expansion of assets, which results in heavy operating
cost to the firm. On the other hand, lack of investment in asset would influence the competitive position
of the firm. So the capital budgeting decisions determine the future destiny of the company.
(2) Involvement of large amount of funds: Capital budgeting decisions need substantial amount of
capital outlay. This underlines the need for thoughtful, wise and correct decisions as an incorrect decision
would not only result in losses but also prevent the firm from earning profit from other investments which
could not be undertaken.
(3) Irreversible decisions: Capital budgeting decisions in most of the cases are irreversible because it is
difficult to find a market for such assets. The only way out will be to scrap the capital assets so acquired
and incur heavy losses.
(4) Risk and uncertainty: Capital budgeting decision is surrounded by great number of uncertainties.
Investment is present and investment is future. The future is uncertain and full of risks. Longer the period
of project, greater may be the risk and uncertainty. The estimates about cost, revenues and profits may
not come true.
(5) Difficult to make: Capital budgeting decision making is a difficult and complicated exercise for the
management. These decisions require an overall assessment of future events which are uncertain. It is
really a marathon job to estimate the future benefits and cost correctly in quantitative terms subject to
the uncertainties caused by economic-political social and technological factors.
Question 21] Write a short note on: Kinds of capital budgeting decisions
Ans.: Generally the business firms are confronted with three types of capital budgeting decisions:
(1) Independent proposals: Business firm is confronted with alternative investment proposals. If the
proposal is accepted, the firm incur the investment and not otherwise. Broadly, all those investment
proposals which yield a rate of return greater than cost of capital are accepted and the others are rejected.
Under this criterion, all the independent prospects are accepted.
(2) Mutually exclusive decisions: It includes all those projects which compete with each other in a way
that acceptance of one precludes the acceptance of other or others. Thus, some technique has to be used
for selecting the best among all and eliminates other alternatives.
(3) Capital rationing decisions: Capital budgeting decision is a simple process in those firms where fund
is not the constraint, but in majority of the cases, firms have fixed capital budget. So large number of
projects competes for the limited budget. So the firm rations them in a manner so as to maximize the long
run returns. Thus, capital rationing is concerned with the selection of a group of investment out of many
investment proposals ranked in the descending order of the rate of return.
Question 22] Why do you think the capital budgeting decisions are vital for any firm?
Ans.: Capital budgeting is a process that attempts to determine the merits of an investment project.
Before any large project begins, the capital budgeting process should be utilized. Capital budgeting is
important because it creates accountability and measurability. Any business that seeks to invest its
resources in a project, without understanding the risks and returns involved, would be held as
irresponsible by its owners or shareholders. Furthermore, if a business has no way of measuring the
effectiveness of its investment decisions, chances are that the business will have little chance of surviving
in the competitive marketplace.
One of the main objectives of the businesses is to earn profits. The capital budgeting process is a
measurable way for businesses to determine the long-term economic and financial profitability of any
investment project. Capital budgeting is also vital to a business because it creates a structured step by
step process that enables a company to:
1. Develop and formulate long-term strategic goals
2. Seek out new investment projects
3. Estimate and forecast future cash flows
4. Facilitate the transfer of information
5. Monitoring and control of expenditures
6. Creation of decision
Unlike other business decisions that involve a singular aspect of a business, a capital budgeting decision
involves two important decisions at once: a financial decision and an investment decision. By taking on a
project, the business has agreed to make a financial commitment to a project, and that involves its own
set of risks. Projects can run into delays, cost overruns and regulatory restrictions that can all delay or
increase the projected cost of the project.
In addition to a financial decision, a company is also making an investment in its future direction and
growth that will likely have an influence on future projects that the company considers and evaluates. So
to make a capital investment decision only from the perspective of either a financial or investment
decisions can pose serious limitations on the success of the project.
PROBLEMS & SOLUTIONS - BASIC
PAYBACK PERIOD, NPV, ARR& IRR
Problem No. 1] ABC Ltd. is considering investing in a project that costs ` 5,00,000. The estimated salvage
value is zero, tax rate is 35%. The company uses straight line depreciation for tax purposes and the
proposed project has profit before charging depreciation as follows:
Year 1 2 3 4 5
Profit before charging depreciation 1,50,000 1,75,000 2,00,000 2,50,000 3,25,000
Company’s cost of capital is 10%. Determine the following:
(i) Pay back period
(II) NPV
(iii) Profitability index
(iv) IRR
(v) ARR
(vi) Discounted payback period
Ans.: Calculation of CFAT:
Year 1 2 3 4 5
Profit before depreciation 1,50,000 1,75,000 2,00,000 2,50,000 3,25,000
(-) Depreciation (1,00,000) (1,00,000) (1,00,000) (1,00,000) (1,00,000)
Profit before tax 50,000 75,000 1,00,000 1,50,000 2,25,000
(-) Tax @ 35% (17,500) (26,250) (35,000) (52,500) (78,750)
PAT
32,500 48,750 65,000 97,500 1,46,250
(+) Depreciation CFAT
1,00,000 1,00,000 1,00,000 1,00,000 1,00,000
1,32,500 1,48,750 1,65,000 1,97,500 2,46,250

Calculation of payback period:


Year CFAT Cumulative
CFAT
1 1,32,500 1,32,500
2 1,48,750 2,81,250
3 1,65,000 4,46,250
4 1,97,500 6,43,750
5 2,46,250 8,90,000

Payback period of Project A =3 years + 5,00,000-4,46,250/1 97 500 x 12


= 3 years & 3.27 months
Calculation of NPV:
Since, company's cost of capital is 10%; NPV at 10% has to be calculated.
PV Factor PV
Year CFAT
10% 1096
1 1,32,500 0.909 1,20,443
2 1,48,750 0.826 1,22,867
3 1,65,000 0.751 1,23,915
4 1,97,500 0.683 1,34,893
5 2,46,250 0.621 1,52,921
Total present value 6,55,039
(-) Initial investment (5,00,000)
Net Present Value (NPV) 1,55,039
Since NPV is positive at 10%, it is advised to buy machine.
Profitability Index = Total present value/Initial investment = 6,55,039/5,00,000 = 1.31
Since, Profitability Index > 1, project is acceptable.
Calculation of IRR: Since NPV at 10% is much larger, higher discount rate has to be selected for IRR
calculations.
PV Factor Present Value
Year CFAT
1896 21% 18% 21%
1 1,32,500 0.847 0.826 1,12,228 1,09,445
2 1,48,750 0.718 0.683 1,06,802 1,01,596
3 1,65,000 0.609 0.564 1,00,485 93,060
4 1,97,500 0.516 0.467 1,01,910 . 92,233
5 2,46,250 0.437 0.386 1,07,611 95,052
Total present value 5,29,036 4,91,386
(-) Initial investment (5,00,000) (5,00,000)
Net Present Value (NPV) 29,036 (8,614)

IRR = 18 + 29,036/29,036 + 8,614 X 3


= 18 + /29,036/37,650 X 3
= 18+ 2.31
= 20.3196
Calculation of ARR:
ARR = Average PAT/Initial investment x 100 = 78,000/5,00,000 X 100 = 15.696
Since, IRR is greater than company’s cost of capital, it advised to buy the machine.
Calculation of discountedPresent Value Cumulative PV
payback period :
Year
1 1,20,443 1,20,443
2 1,22,867 2,43,310
3 1,23,915 3,67,225
4 1,34,893 5,02,118
5 1,52,921 6,55,039

Discounted payback period = 3 years + 5,00,000 - 3,67,225/1,34,893 X 12


= 3 years & 11.81 months
Problem No. 2] The investment data for a machine are as follows:
Capital outlay : ` 2,50,000
Depreciation : 25% p.a. on WDV
Tax rate : 35%
Cost of capital : 12%
Sale price at the end of 5th year of the machine is ` 35,000.
Forecasted annual income before charging depreciation is as follows:
Year 12345
Profit before charging depreciation 1,30,000 1,00,000 70,000 40,000 21,000
On the basis of available data, set out calculations, illustrating and comparing the following methods of
evaluating capital budgeting decisions:
(i) Payback Period (ii) NPV (iii) IRR (iv) ARR (v) Discounted Payback Period
Ans.: Calculation of depreciation under WDV method and net cash flow from disposal of machine at the
end of 5th year.

Purchase cost 2,50,000


(62,500)
(-) Depreciation for 1st year 1,87,500
(-) Depreciation for 2nd year (46,875)
(-) Depreciation for 3rd year 1,40,625
(35,156)

(-) Depreciation for 4th year 1,05,469


(26,367)

(-) Depreciation for 5th year 79,102


(19,776)

(-) Loss on sale (Bal Fig.) 59.326


24.326

Sale price 35,000

(+) Tax benefit on loss on sale (24,326 X 35%) 8,514

Net cash flow on sale 43,514

Calculation of CFAT:
Year 1 2 3 4 5
Profit before depreciation 1,30,000 1,00,000 70,000 40,000 21,000
(-) Depreciation (62,500) (46,875) (35,156) (26,367) (19,775)
Profit before tax 67,500 53,125 34,844 13,633 1,225
(-) Tax @ 35% (23,625) (18,594) (12,195) (4,772) (429)
PAT 43,875 34,531 22,649 8,86.1 796
(+) Depreciation 62,500 46,875 35,156 26,367 19,775
CFAT 1,06,375 81,406 57,805 35,228 20,571
Net cash flow on sale - - - - 43,514
Net CFAT 1,06,375 81,406 57,805 35,228 64,085
Calculation of payback period:
Year CFAT Cumulative
CFAT
1 1,06,375 1,06,375
2 81,406 1,87,781
3 57,805 2,45,586
4 35,228 2,80,814
5 64,085 3,44,899

Payback period = 3 years + 2,50,000 - 2,45,586/35,228 X 12


= 3 years & 1.5 months
Since, company’s cost of capital is 12%; NPV at 12% has to be calculated. Calculation of IRR:
PV Factor Present Value
Year CFAT
12% 14% 12% 14%
1 1,06,375 0.893 0.877 94,993 93,291
2 81,406 0.797 0.769 64,881 62,601
3 57,805 0.712 0.675 41,157 39,018
4 35,228 0.636 0.592 22,405 20,855
5. 64,085 0.567 0.519 36,336 33,260
Total present value 2,59,772 2,49,025
(-) Initial investment (2,50,000) (2,50,000)
Net Present Value (NPV) 9,772 (975)

IRR = 12 + 9,772/9,772 + 975 X 2


= 12 + 9,772/10,747 X 2
= 12+ 1.82
= 13.82%
Profitability Index = Total Present Value/Initial Investment = 2,59,772/2,50,000 =1.04
Since, Profitability Index > 1, project is acceptable.
Computation of ARR:
ARR = Average PAT/Initial investment x 100 = 22,142/2,50,000 X 100 = 8.86%
Calculation of discounted payback period:
Year PV Cumulative
PV
1 94,993 94,993
2. 64,881 1,59,874
3 41,157 2,01,031
4 22,405 2,23,436
5 36,336 2,59,772

Discounted payback period = 4 years + 2,50,000-2,23,436/36.336 X 12


= 4 years & 8.77 months
Problem No. 3] A company is considering which of two mutually exclusive projects it should undertake.
The Finance Director thinks that the project with the higher NPV should be chosen whereas the Managing
Director thinks that the one with the higher IRR should be undertaken especially as both projects have
the same initial outlay and length of life. The company anticipates a cost of capital of 10% and the net
after-tax cash flows of the projects are as follows:

Year 0 1 2 3 4 (` WO) 5
Cash Flows: Project X (200) 35 80 90 75 20
Project Y (200) 218 10 10 4 3
Required:
(a) Calculate the NPV and IRR of each project.
(b) State, with reasons, which project you would recommend.
(c) Explain the inconsistency in the ranking of the two projects. The discount factors are as follows:
Year 0 1 2 3 4 5
PV Factors 1096 1.00 0.91 0.83 0.75 0.68 . 0.62
2096 1.00 0.83 0.69 0.58 0.48 0.40
Ans.: Calculation of IRR:

Project X : (` '000) PV Factor Present Value


CFAT
Year 10% 20% 10% .. . 20%
1 35 0.91 0.83 31.85 29.05
2 80 0.83 0.69 66.40 55.20
3 90 0.75 0.58 67.50 52.20
4 75 0.68 0.48 51.00 36.00
5 20 0.62 0.40 12.40 8.00
Total present value 229.15 180.45
(-) Initial investment (200.00) (200.00)
Net Present Value (NPV) 29.15 (19.55)
IRR = 10 + 29.15/29.15 + 19.55 X 10
= 10 + 29.15/48.7 x 10
= 10+ 5.99
= 15.99% say 16%
Project Y: (` ’000)
PV Factor Present Value
Year CFAT
10% 20% 10% 20%
1 218 0.91 0.83 198.38 180.94
2 10 0.83 0.69 8.30 6.90
3 10 0.75 0.58 7.50 5.80
4 4 0.68 0.48 2.72 1.92
5 3 0.62 0.40 1.86 1.20
Total present value 218.76 196.76
(-) Initial investment (200.00) (200.00)
Net Present Value (NPV) 18.76 (3.24)
IRR= 10 + 18.76/18.76 + 3.24 X 10
= 10 + 18.76/22 x 10
= 10+ 8.53
= 18.53%
Ranking under various methods:
Particulars x Y Ranking
X Y
Net present value (at 10%) 29.15 18.76 I II
Internal rate of return 16.00% 18.53% II I
Analysis: Both projects are acceptable because they generate the positive NPV at the company’s cost of
capital at 10%. Project X has higher NPV as compared to Project Y, hence Project X should be selected.
If company follows IRR method then Project Y should be selected, as its IRR is greater than Project X.
Where NPV and IRR gives contradictory results, a project with higher NPV is generally preferred because
of higher return in absolute terms.
Problem No. 4] Projects X and Y are analyzed and you have determined the following parameters. Advise
the investor on the choice of a project:
Project X Project Y
Investment ` 7 Cr. `5 Cr.
Project life 8 years 10 years
Construction period 3 years 3 years
Cost of capital 15% 18%

NPV @ 12% ` 3,700 ` 4,565

NPV @ 18% ` 325 `325


IRR 45% 32%
Rate of return 18% 25%
Payback 4 years 6 years
BEP 45% 30%
Profitability 1.76 1.35
Ans.: Ranking under various methods:
Particulars Project X Project Y Ranking Reason
Project X Project Y
Project having lower payback period will
Payback period 4 years 6 years I n
rank first.

Net present value (at 12%) ` 3,700 ` 4,565 II I Project having higher NPV will rank first.
Net present value (at 18%) ` 325 ` 325 Equal Equal Project having higher NPV will rank first.

Profitability index 1.76 1.35 I n Project having higher PI will rank first.
Internal rate of return 45% 32% I n Project having higher IRR will rank first.
Accounting rate of return 18% 25% II i Project having higher ARR will rank first.
BEP 45% 30% II i Project having lowest BEP will rank first.
Cost of capital 15% 18% I H Project having lowest K will rank first.
Problem No. 5] A Ltd. is considering the question of taking up a new project which requires an investment
of ` 200 lakhs on machinery and other assets. The project is expected to yield the following gross profits
(before depreciation and tax) over the next five years:

Year 1 2 3 (` in lakhs) 4 5
Gross Profit 80 80 90 90 75
The cost of raising the additional capital is 12% and the assets have to be depreciated at 20% on ‘written
down value’ basis. The scrap value at the end of the five-year period may be taken as zero. Income-tax
applicable to the company is 50%.
Calculate the net present value of the project and advise the management whether the project has to be
implemented. Also calculate the internal rate of return of the project.
Note: Present value of ` 1 at different rates of interest is as follows:
Year 10% 12% 14% 16%
1 0.91 0.89 0.88 0.86
2 0.83 0.80 0.77 0.74
3 0.75 0.71 0.67 0.64
4 0.68 0.64 0.59 0.55
5 0.62 0.57 0.52 0.48
Ans.: Calculation of depreciation under WDV method and net cash flow from disposal of machine at the
end of 5th year.

Particulars (` in lakhs)
Purchase cost 200.00
(-) Depreciation for 1st year (40.00)
160.00
(-) Depreciation for 2nd year (32.00) 128.00 .
(-) Depreciation for 3rd year (25.60)
102.40
(-) Depreciation for 4th year (20.48)
81.92
(-) Depreciation for 5th year (16.38)
65.54
(-) Loss on sale (Bal Fig.) (65.54)
Sale price Nil
(+) Tax benefit on loss on sale (65.54 X 50%) 32.77
Net cash flow on sale 32.77
Calculation of CFAT:
Year 1 2 3 4 5
Profit before depreciation 80.00 80.00 90.00 90.00 75.00
(-) Depreciation (40.00) (32.00) (25.60) (20.48) (16.38)
Profit before tax 40.00 48.00 64.40 69.52 58.62
(-) Tax @ 50% (20.00) (24.00) (32.20) (34.76) (29.31)
PAT 20.00 24.00 32.20 34.76 29.31
(+) Depreciation 40.00 32.00 25.60 20.48 16.38
CFAT 60.00 56.00 57.80 55.24 45.69
Net cash flow on sale - - - - 32.77
Net CFAT 60.00 56.00 57.80 55.24 78.46

Calculation of PV Factor Present Value


IRR : CFAT
Year 1296 1696 1296 1696
1 60.00 0.89 0.86 53.40 51.60
2 56.00 0.80 0.74 44.80 41.44
3 57.80 0.71 0.64 41.04 36.99
4 55.24 0.64 0.55 35.35 30.38
5 78.46 0.57 0.48 44.72 37.66
Total present value 219.31 198.07
(-) Initial investment (200.00) (200.00)
Net Present Value (NPV) 19.31 (1.93)
IRR = 12+19.31/19.31 + 1.93 X 4
= 12 + 19.31/21.24 x 4
= 12 + 3.64
= 15.6496
Decision Rule:
(1) NPV is positive at 12%, Project is acceptable.
(2) IRR (15.64%) is greater than Cost of Capital (12%), Project is acceptable.
Problem No. 6] A hospital is considering purchasing a diagnostic machine costing ` 80,000. The projected
life of the machine is 8 years and has an expected salvage value of ` 6,000 at the end of 8 years. The annual
operating cost of the machine is ` 7,500. It is expected to generate revenues of ` 40,000 per year for eight
years. Presently, the hospital is outsourcing the diagnostic work and is earning commission income of `
12,000 per annum; net of taxes.
Assume tax rate 30%.
Required:
Whether it would be profitable for the hospital to purchase the machine? Give your recommendation
under:
(i) Net Present Value method («) Profitability Index method
Ans.: Calculation of CFAT:
Particulars `
Revenue 40,000
(-) Operating cost (7,500)
(-) Depreciation (9,250)
Profit before tax 23,250
(-) Tax @ 3096 (6,975)
PAT 16,275
(+) Depreciation 9,250
CFAT 25,525
(-) Loss of commission (12,000)
NET CFAT 13,525

Year CFAT PV Factor 10% PV


1 to 8 13,525 5.335 72,156
8 * 6,000 0.467 2,802 * Salvage value
Total present value 74,958
(-) Initial investment (80,000)
Net Present Value (NPV) (5,042)
Analysis: Since NPV is negative, investment is not a profitable.
Profitability Index = Total Present Value/Initial Investment = 74,958/80,000 = 0.937
Analysis: Since profitable index is less than 1, project is not acceptable.
Problem No. 7] Zenith Industries Ltd. is thinking of investing in a project costing ` 20 lakhs. The life of the
project is five years and the estimated salvage value of the project is zero. Straight-line method of charging
depreciation is followed. The tax rate is 50%. The expected cash flows before tax are as follows:
Year 1 2 3 4 5
Cash flow before depreciation on 4,00,000 6,00,000 8,00,000 8,00,000 10,00,000
You are required to determine:
(i) Payback period for the investment
(ii) Average rate of return on the investment
(iii) Net present value at 10% cost of capital
(iv) Benefit cost ratio
Ans.: Calculation of CFAT:
Year 1 2 3 4 ‘51
Profit before depreciation . 4,00,000 6,00,000 8,00,000 8,00,000 10,00,000
(-) Depreciation (4,00,000) (4,00,000) (4,00,000) (4,00,000) (4,00,000)
Profit before tax - 2,00,000 4,00,000 4,00,000 6,00,000
(-) Tax @ 50% - (1,00,000) (2,00,000) (2,00,000) (3,00,000)
PAT - 1,00,000 2,00,000 2,00,000 3,00,000
(+) Depreciation 4,00,000 4,00,000 4,00,000 4,00,000 4,00,000
CFAT ‘ 4,00,000 5,00,000 6,00,000 6,00,000 7,00,000
Calculation of payback period:
Year CFAT Cumulative CFAT 4,00,000
1 4,00,000
2 5,00,000 9,00,000

3 6,00,000 15,00,000
4 6,00,000 21,00,000
5 7,00,000 28,00,000
Payback Period = 3 years + 20,00,000- 15,00,000/6,00,000 x 12
= 3 years & 10 months
Calculation of ARR:
ARR = Average PAT/Initial investment X 100 = 1,60,000/20,00,000 x 100 = 8%
Calculation of net present value:
Year CFAT PV Factor 10% Present Value
1 4,00,000 0.909 3,63,600
2 5,00,000 0.826 4,13,000
3 6,00,000 0.751 4,50,600
4 6,00,000 0.683 4,09,800
5 7,00,000 0.621 4,34,700
Total present value 20,71,700
(-) Initial investment (20,00,000)
Net Present Value (NPV) 71,700
Calculation of benefit cost ratio:
Profitability Index = Total Present Value/Initial investment = 20,71,700/20,00,000 = 1.036
Problem No. 81 XYZ Ltd. is considering purchase of a machine in replacement of an old one. Two models
‘MOLIN’ and ‘SKODA’ are offered at prices of ` 22.50 lakhs and ` 30 lakhs respectively. Further particulars
regarding these models are given below.

MOLIN (` lakhs) SKODA


Economic life (years) .5 6
Scrap value at the end of the economic life 2 2.50
After tax annual cash inflows: Year
1 5.00 6.00
2 7.50 8.00
3 10.00 10.00
4 9.00 12.00
5 8.50 10.50
6 - 9.50
Present value factors at 12% per annum are as follows:
Year 1 2 3 4 5 6
PV factors 0.893 0.797 0.712 0.636 0.567 0.507
Evaluate the two proposals under:
(a) Payback period and
(b) Net present value method
(c) Which model would you recommend and why?
Ans. : MOLIN SKODA
Year CFAT Cumulative CFAT CFAT Cumulative CFAT
1 5.00 5.00 6.00 6.00
2 7.50 12.50 8.00 14.00
3 10.00 22.50 10.00 24.00
4 9.00 31.50 12.00 36.00
5 * 10.50 42.00 10.50 46.50
6 - - * 12.00 58.50
**includes scarp value.
Payback Period:
Project A =3 years
Project B =3 years + 30 - 24/12 X 12
= 3 years & 6 months
Calculation of NPV: (` in Lakhs)
CFAT PV Factor Present Value
Year
MOLIN SKODA 12% MOLIN SKODA
1 5.00 6.00 0.893 4.4650 5.3580
2 7.50 8.00 0.797 5.9775 6.3760
3 10.00 10.00 0.712 7.1200 7.1200
4 9.00 12.00 0.636 5.7240 7.6320
5 10.50 10.50 0.567 5.9535 5.9535
6 - 12.00 0.507 - 6.0840
Total present value 29.2400 38.5235
(-) Initial investment (22.5000) (30,0000)
Net Present Value (NPV) 6.7400 8.5235
Analysis: SKODA should be selected as it NPV is higher than MOLIN.
Problem No. 9] United Ltd. has an investment budget of ` 100 lakhs for 2014-2015. It has short listed two
Projects A and B after completing the market and technical appraisals. The management wants to
complete the financial appraisal before making the investment. Further particulars regarding the two
projects are given below:
P in lakhs)
A B
Investment required 100 90
Average annual cash inflow before depreciation & tax (estimate) 28 24
Salvage value = Nil for both projects.
Estimate life = 10 years for both projects.
The company follows straight line method of charging depreciation. Its tax rate is 50%. You are required
to calculate:
(a) Payback period and
(b) IRR of the two projects.
Note: PV of an annuity of ` 1 for ten years at different discount rates is given below.
Rate 96 10 11 12 13 14 15
Annuity value for 10 years 6.1446 5.8892 5.6502 5.4262 5.2161 5.0188
Ans.: Calculation of CFAT: in lakhs)
A B
Profit before depreciation 28.00 24.00
(-) Depreciation (10.00) (9.00)
Profit before tax 18.00 15.00
(-) Tax @ 5096 (9.00) (7.50)
PAT 9.00 7.50
(+) Depreciation 10.00 9.00
CFAT 19.00 16.50
Computation of Payback Period:
Payback Period = Initial Investment/Annual CFAT
Project A = 100/19 = 5.26 years
Project B = 90/16.5 = 5.45 years
Calculation of IRR:
Project A:
PV Factor PV
Year CFAT 1296 1596 1296 15%
1 to 10 19 5.6502 5.0188 107.3538 95.3572
(-) Initial investment (100.00) (100.00)
Net Present Value (NPV) 7.3538 (4.6428)

IRR = 12 + 7.3538/7.3538 + 4.6428 x 3


= 12 + 73538/11.9966 x 3
= 12 +1.84
= 13.8496
Project B : PV Factor PV
Year CLA1 1296 1596 1296 1596
1 to 10 16.5 5.6502 5.0188 93.2283 82.8102
(-) Initial investment (90.00) (90.00)
Net Present Value (NPV) 3.2283 (7.1898)

IRR= 12 + 3.2283/ 3.2283 + 7.1898 x 3


= 12 + 3.2283/10.4181 x 3
= 12 + 0.93
= 12.93%
Problem No. 10] A company is considering whether it should spend ` 4 lakhs on a project to manufacture
and sell a new product. The unit variable cost of the product is ` 6. It is expected that the new product can
be sold at ` 10 per unit.
The annual .fixed costs (only cash) will be ` 20,000. The project will have a life of six years with scrap value
of ` 20,000. The cost of capital of the company is 15%. The only uncertain factor is the volume of sales. To
start with, the company expects to sell at least 40,000 units during the first year. Ignore taxation.
Required:
(i) Net present value of the project based on the sales expected during the first year and on the assumption
that it will continue at the same level during the remaining years.
(ii) The minimum volume of sales required to justify the project.
Note: Annuity of ` 1 at 15% for six years has a present value of ` 3.7845 and present value of ` 1 received
at the end of sixth year at 15% is ` 0.4323.
Ans.: Contribution per unit =10-6 = 4
Total contribution (40,000 X 4) 1,60,000
(-) Fixed cost (20,000)
Net cash inflow per year 1,40,000

Year Net cash flow PV Factor 15% PV


1 to 6 1,40,000 3.7845 5,29,830
6 20,000 0.4323 8,646
5,38,476
(-) Initial investment (4,00,000)
Net Present Value (NPV) 1,38,476
Calculation of minimum volume of sales required to justify the project:
Let the total unit be ‘x'.
Total contribution (x X 4) 4x
(-) Fixed cost (20,000)
Net cash inflow per year 4x - 20,000

Year Net cash flow PV Factor 15% PV


1 to 6 4* -20,000 3.7845 15.138*-75,690
6 20,000 0.4323 8,646
15.138*-67,044
(-) Initial investment (4,00,000)
Net Present Value (NPV) 0
15.138* - 67,044 - 4,00,000 = 0 15.138* = 4,67,044 * = 30,852.42 say 30,852 units
Problem No. 11] Following are the data on a capital project being evaluated by the management of X Ltd.
Project M:
Annual cost saving ` 40,000
Useful life 4 years
IRR 15%
Profitability index (PI) 1.064
NPV ?
Cost of capital ?
Cost of project p
Payback period ?
Salvage value 0
Find the missing values considering the following table of discount factor only:
PV Factor 15% 14% 13% 12%
1 0.869 0.877 0.885 0.893
2 0.756 0.769 0.783 0.797
3 0.658 0.675 0.693 0.712
4 0.572 0.592 0.613 0.636
2.855 2.913 2.974 3.038
Ans.: Calculation of Cost of project:
Annual cost saving = Cash inflow = ` 40,000 Useful life = 4 years IRR =15%
At 15% IRR, total present value of cash inflow is equal to initial cash outlay.
Total present value of cash inflow @ 15% for 4 years is 2.855 = 40,000 X 2.855 = 1,14,200 Thus, Project
cost = ` 1,14,200.
Calculation of payback period:
Year CFAT Cumulative CFAT
1 40,000 40,000
2 40,000 80,000
3 40,000 1,20,000
4 40,000 1,60,000
Payback period
Project A = 2 years + 1,14,200-80,000/40,000 X 12
= 2 years & 10.26 months Calculation of cost of capital:
Profitability Index = Total Present Value/Initial Investment
1.064= Total Present Value/1,14,200
Total Present Value = 1,21,509
Year Cash Flow PV Factor PV
1 to 4 40,000 x 40,000x
40,000x= 1,21,509 x = 3.038
Looking at present value table, discount factor for 4 years is 3.038.
Hence, Cost of capital = 12%
Net present value = 1,21,509 - 1,14,200 = 7,309
Problem No. 12] The cash flows of projects C and D are reproduced below:
Cash Flow NPV IRR
C0 c1 c2 C3 at 1096
C - 10,000 + 2,000 + 4,000 + 12,000 + 4,139 26.596
D - 10,000 + 10,000 + 3,000 + 3,000 + 3,823 37.696
(i) Why there is a conflict of rankings?
(ii) Why should you recommend Project C in spite of lower internal rate of return?
Time 1 2 3
PVIF 0.101 0.9090 0.8264 0.7513
PVIF 0.141 0.8772 0.7695 0.6750
PVIF 0.15 t 0.8696 0.7561 0.6575
PVIF 0.30 t 0.7692 0.5917 0.4552
PVIF 0.40 t 0.7143 0.5102 0.3644
Ans.: Calculation of NPV at different discounting rates:
(1) At 0% discount rate:
Project C = 2,000 + 4,000 + 12,000 - 10,000 = 8,000 Project D = 10,000 + 3,000 + 3,000 - 10,000 = 6,000
(2) At 10% discount rate:
Project C = 4,139 (given)
Project D = 3,823 (given)
(3) At 15% discount rate:
CFAT PV Factor Present Value
Year
Project C Project D 15% Project C Project D
1 2,000 10,000 0.8696 1,739 8,696
2 4,000 3,000 0.7561 3,024 2,268
3 12,000 3,000 0.6575 7,890 1,973
Total present value * 12,653 12,937
(-) Initial investment (10,000) (10,000)
Net Present Value (NPV) 2,653 2,937
(4) At 30% discount rate:
CFAT PV Factor Present Value
Year
Project C Project D 3096 Project C Project D
1 2,000 10,000 0.7692 1,538 7,692
2 4,000 3,000 0.5917 2,367 1,775
3 12,000 3,000 0.4552 5,462 1,366
Total present value 9,367 10,833
(-) Initial investment (10,000) (10,000)
Net Present Value (NPV) (633) 833
(5) At 40% discount rate:
CFAT PV Factor Present Value
Year
Project C Project D 4096 Project C Project D
1 2,000 10,000 0.7143 1,429 7,143
2 4,000 3,000 0.5102 2,041 1,531
3 12,000 3,000 0.3644 4,373 1,093
Total present value (7,843) 9,767
(-) Initial investment (10,000) (10,000)
Net Present Value (NPV) (2,157) (233)
Ranking of Projects:
Particulars C D Ranking
C D
Net present value (at 0%) 8,000 6,000 I 11
Net present value (at 10%) 4,139 3,823 I II
Net present value (at 15%) 2,653 2,937 II I
Net present value (at 30%) (633) 833 II I
Net present value (at 40%) (2,157) (233) II I
Internal rate of return 26.596 37.696 II I
Problem No. 13] Company UVW has to make a choice between two identical machines, in terms of
capacity, ‘A’ and ‘B’. They have been designed differently, but do exactly the same job.
Machine ‘A’ costs ` 7,50,000 and will last for three years. It costs ` 2,00,000 per year to run. Machine ‘B’ is
an economy model costing only ` 5,00,000, but will last for only two years. It costs ` 3,00,000 per year to
run.
The cash flows of Machine ‘A’ and 'B' are real cash flows. The costs are forecasted in rupees of constant
purchasing power. Ignore taxes. The opportunity cost of capital is 9%.
Required:
Which machine the company UVW should buy?
The present value (PV) factors at 9% are:
Year t1 t2 t3
PVIF 0.09 t 0.9174 0.8417 0.7722
Ans.: Machine A: Present value of cash outflow:
Year Cash outflow PV Factor PV
0 7,50,000 1,0000 7,50,000
1 to 3 2,00,000 2.5313 5,06,260
12,56,260
Equivalent PV of annual cash outflow = 12,56,260/2.5313 = 4,96,290
Machine B: Present value of cash outflow:
Year Cash outflow PV Factor PV
0 5,00,000 1,0000 5,00,000
1 to 2 3,00,000 1.7591 5,27,730
10,27,730

Equivalent PV of annual cash outflow = 10,27,730/1.7591 = 5,84,236


Analysis: Company UVW should buy Machine ‘A’ since equivalent annual cash outflow is less than that of
Machine B.
Problem No. 14] A company is considering the proposal of taking up a new project, which requires an
investment of ^ 400 lakhs on machinery and other assets. The project is expected to yield the following
earnings (before depreciation and taxes) over the next five years:
Year 1 2 3 4 5
Earnings (7 in Lakhs) 160 160 180 180 150
The cost of raising the additional capital is 12% and assets have to be depreciated at 20% on 'written down
value' basis. The scrap value at the end of the five years’ period may be taken as zero. Income tax
applicable to the company is 50%.
You are required to calculate the net present value of the project and advise the management to take
appropriate decision. Also calculate the internal rate of return of the project.
Note: Present value of ` 1 at different rates of interest are as follows:
Year 1096 1296 1496 1696
1 0.91 0.89 0.88 0.86
2 0.83 0.80 0.77 0.74
3 0.75 0.71 0.67 0.64
4 0.68 0.64 0.59 0.55
5 0.62 0.57 0.52 0.48
Ans.: Calculation of depreciation under WDV method and net cash flow from disposal of machine at the
end of 5th year.
Particulars ` in Lakhs
Purchase cost 400.00
(-) Depreciation for 1st year (80.00)
320.00
(-) Depreciation for 2nd year (64.00)
256.00
(-) Depreciation for 3rd year (51.20)
204.80
(-) Depreciation for 4th year (40.96)
163.84
(-) Depreciation for 5th year (32.77)
131.07
(-) Loss on sale (Bal. Fig.) (131.07)
Sale price Nil
(+) Tax benefit on loss on sale (131.07 X 50%) 65.54
Net cash flow on sale 65.54

Calculation of CFAT: (` in Lakhs)


Year 1 2 3 4 5
Earnings 160.00 160.00 180.00 180.00 150.00
(-) Depreciation (80.00) (64.00) (51.20) (40.96) (32.77)
Profit before tax 80.00 96,00 128.80 139.04 117.23
(-) Tax @ 50% (40.00) (48.00) (64.40) (69.52) (58.62)
PAT 40.00 48.00 64.40 69.52 58.61
(+) Depreciation 80.00 64.00 51.20 40.96 32.77
CFAT 120.00 112.00 115.60 110.48 91.38
Tax benefit on loss on sale - - - 65.54
Net CFAT 120.00 112.00 115.60 110.48 156.92

Calculation of IRR: (` in Lakhs)


PV Factor Present Value
Year CFAT
12% 16% 12% 16%
1 120.00 0.89 0.86 106.80 103.20
2 112.00 0.80 0.74 89.60 82.88
3 115.60 0.71 0.64 82.08 73.98
4 110.48 0.64 0.55 70.71 60.76
5 156.92 0.57 0.48 89.44 75.32
Total present value 438.63 396.14
(-) Initial investment (400.00) (400.00)
Net Present Value (NPV) 38.63 (3.86)
IRR = 12 + 38.63/38.63 + 3.86 X 4
= 12+ 38.63/42.49 x 4
= 12 + 3.64
= 15.64%
Problem No. 15] Consider the following mutually exclusive projects:
Cash Flow ft )
Projects C0 C1 c2 c3 c4
A - 10,000 6,000 2,000 2,000 12,000
B - 10,000 2,500 2,500 5,000 7,500
C - 3,500 1,500 2,500 500 5,000
D - 3,000 0 0 3,000 6,000
Required:
(i) Calculate the payback period for each project.
(ii) If the standard payback period is 2 years, which project will you select? Will your answer differ, if
standard payback period is 3 years?
(iii) If the cost of capital is 10%, compute the discounted payback period for each project. Which projects
will you recommend, if standard discounted payback period is (i) 2 years; (ii) 3 years?
(iv) Compute NPV of each project. Which project will you recommend on the NPV criterion? The cost of
capital is 10%. What will be appropriate choice criteria in this case?
The PV factor at 10% are:
Year 1 2 3 4
PV factor at 10% (PV/F 0.10, t) 0.9091 0.8264 0.7513 0.6830
Ans.: Calculation of payback period:

Project A Project B Project C Project D


Year CFAT Cumulative CFAT Cumulative CFAT Cumulative
CFAT Cumulative CFAT CFAT CFAT CFAT
1 6,000 6,000 2,500 2,500 1,500 1,500 0 0
2 2,000 8,000 2,500 5,000 2,500 4,000 0 0
3 2,000 10,000 5,000. 10,000 500 4,500 3,000 3,000
4 12,000 22,000 7,500 17,500 5,000 9,500 6,000 9,000
Project A = 3 years
Project B = 3 years
Project C = 1 year + 3,500 - 1,500/2,500 x 12
= 1 year & 9.6 months
Project D = 3 years
If standard payback period is 2 years, Project C is the only acceptable project. But if standard payback
period is 3 years, all the four projects are acceptable. Calculation of NPV:
CFAT PV Factor PV *
Year
A B C D 10% A B C D
1 6,000 2,500 1,500 0 0.9091 5,455 2,273 1,364 0
2 2,000 2,500 2,500 0 0.8264 1,653 2,066 2,066 0
3 2,000 5,000 500 3,000 0.7513 1,503 3,757 376 2,254
4 12,000 7,500 5,000 6,000 0.6830 8,196 5,123 3,415 4,098
Total present value 16,807 13,218 7,220 6,352
(-) Initial investment (10,000) (10,000) (3,500) (3,000)
Net Present Value (NPV) 6,807 3,218 3,720 3,352
Ranking on NPV method I IV II III
Calculation of discounted payback period:
Project A Project B Project C Project D
Year PV Cumulative PV Cumulative PV Cumulative
PV PV PV PV Cumulative PV
1 5,455 5,455 2,273 2,273 1,364 1,364 0 0
2 1,653 7,108 2,066 4,339 2,066 3,430 0 0
3 1,503 8,611 3,757 8,096 376 3,806 2,254 2,254
4 8,196 16,807 5,123 13,219 3,415 7,221 4,098 6,352

Project A =3 years + 10,000-8,611/8,196 x 12


= 3 years & 2.03 months
Project B = 3 years + 10,000-8,096/5,123 x 12
= 3 years & 4.46 months
Project C =2 years + 35,000 - 3.430/376 x 12
= 2 years & 2.23 months
Project D =3 years + 3,000 - 2,254/4,098 x 12
= 3 years & 2.18 months
If standard discounted payback period is 2 years, no project is acceptable on discounted payback period
criterion. If standard discounted payback period is 3 years, Project C is acceptable on discounted payback
period criterion.
Problem No. 16] XYZ Ltd. is planning to introduce a new product with a project life of 8 years. The project
is to be set up in Special Economic Zone (SEZ), qualifies for one time (at starting) tax free subsidy from the
State Government of ` 25,00,000 on capital investment. Initial equipment cost will be ` 1.75 Crores.
Additional equipment costing ` 12,50,000 will be purchased at the end of the third year from
the cash inflow of this year. At the end of 8 years, the original equipment will have no resale value, but
additional equipment can be sold for ` 1,25,000. A working capital of ` 20,00,000 will be needed and it will
be released at the end of eighth year. The project will be financed with sufficient amount of equity capital.
The sales volumes over eight years have been estimated as follows:
Year 1 2 3 4-5 6-8
Units 72,000 1,08,000 2,60,000 2,70,000 1,80,000

A sales price of ` 120 per unit is expected and variable expenses will amount to 60% of sales revenue. Fixed
cash operating costs will amount ` 18,00,000 per year. The loss of any year will be set off from the profits
of subsequent two years. The company is subject to 30% tax rate and considers 12% to be an appropriate
after tax cost of capital for this project. The company follows straight line method of depreciation.
Required:
Calculate the net present value of the project and advise the management to take appropriate decision.
Note: The PV factors at 12% are
Year 1 2 3 4 5 6 7 8
0.893 0.797 0.712 0.636 0.567 0.507 0.452 0.404
Ans.: Calculation of cash outflow of investment:
Particulars
Cash outlay 1,75,00,000
(-) Subsidy from government (25,00,000)
Working capital 20,00,000
Initial investment 1,70,00,000
Contribution per unit = 120 - 72 = 48
Depreciation on original equipment = 1,50,00,000/8 = 18,75,000
Depreciation on additional equipment = 12,50,000- 1,25,000/5 = 2,25,000
Calculation of CFAT:
Year 1 2 3 4-5 6-8
Contribution 34,56,000 51,84,000 1,24,80,000 1,29,60,000 86,40,000
(-) Fixed cost (18,00,000) (18,00,000) (18,00,000) (18,00,000) (18,00,000)
(-) Depreciation (18,75,000) (18,75,000) (18,75,000) (21,00,000) (21,00,000)
Profit before tax (2,19,000) 15,09,000 88,05,000 90,60,000 47,40,000
(-) Tax @ 30% - ** (3,87,000) (26,41,500) (27,18,000) (14,22,000)
PAT (2,19,000) 11,22,000 61,63,500 63,42,000 33,18,000
(+) Depreciation 18,75,000 18,75,000 18,75,000 21,00,000 21,00,000
CFAT 16,56,000 29,97,000 80,38,500 84,42,000 54,18,000
** (15,09,000 - 2,19,000) X 30% = 3,87,000
CFAT for 8th year = 54,18,000 + 20,00,000 (Working capital released) + 1,25,000 (Sale price of additional
equipment) = 75,43,000
Calculation of
NPV
Year CFAT PV Factor 12% Present Value
1 16,56,000 0.893 14,78,808
2 29,97,000 0.797 23,88,609
3 80,38,500 0.712 57,23,412
4 84,42,000 0.636 53,69,112
5 84,42,000 0.567 47,86,614
6 54,18,000 0.507 27,46,926
7 54,18,000 0.452 24,48,936
8 75,43,000 0.404 30,47,372
Total present value 2,79,89,789
(-) Initial investment (1,70,00,000)
(-) Additional equipment in 3rd year (1,25,000 X 0.712) (89,000)
Net Present Value (NPV) 1,09,00,789
Analysis: Project should be selected as its NPV is positive.
Problem No. 17] A company wants to invest in machinery that would cost ` 50,000 at the beginning of
year 1. It is estimated that the net cash inflows from operations will be ` 18,000 per annum for 3 years, if
the company opts to service a part of the machine at the end of year 1 at ` 10,000. In such a case, the
scrap value at the end of year 3 will be ` 12,500. However, if the company decides not to service the part,'
then it will have to be replaced at the end of year 2 at ` 15,400. But in this case, the machine will work for
the 4th year also and get operational cash inflow of ` 18,000 for the 4th year. It will have to be scrapped
at the end of year 4 at ` 9,000. Assuming cost of capital at 10% and ignoring taxes, will you recommend
the purchase of this machine based on the net present value of its cash flows?
If the supplier gives a discount of ` 5,000 for purchase, what would be your decision? (The present value
factors at the end of years 0, 1, 2, 3, 4, 5 and 6 are respectively 1, 0.9091, 0.8264, 0.7513, 0.6830, 0.6209
and 0.5645).
Ans.: Option I: Purchase Machinery and Service its Part at the end of Year 1.
Year Details Cash Flow PV Factor PV
0 Purchase cost (50,000) 1,0000 (50,000)
1 Cash flow from operations 18,000 0.9091 16,364
1 Service a part (10,000) 0.9091 (9,091)
2 Cash flow from operations 18,000 0.8264 14,875
3 Cash flow from operations 18,000 0.7513 13,523
3 Scrap value 12,500 0.7513 9,391
Net present value (NPV) (4,938)
Since, Net Present Value is negative; therefore, this option is not to be considered.
If Supplier gives a discount of ` 5,000 then,
NPV = - 4,938 + 5,000 = + 62
In this case, Net Present Value is positive but very small, therefore, this option may not be advisable.
Option II: Purchase Machinery and Replace Part at the end of Year 2.
Year Details Cash Flow PV Factor PV
0 Purchase cost (50,000) 1,0000 (50,000)
1 Cash flow from operations 18,000 0.9091 16,364
Year Details Cash Flow PV Factor PV
2 Cash flow from operations 18,000 0.8264 14,875
2 Replace cost (15,400) 0.8264 (12,727)
3 Cash flow from operations 18,000 . 0.7513 13,523
4 Cash flow from operations 18,000 0.6830 12,294
4 Scrap value 9,000 0.6830 6,147
Net present value (NPV) 476
Net Present Value is positive, but very low as compared to the investment.
If the Supplier gives a discount of ` 5,000, then NPV = 476 + 5,000 = 5,476
Decision: Option II is worth investing as the net present value is positive and higher as compared to Option
I.
Problem No. 18] The management of P Limited is considering to select a machine out of two mutually
exclusive machines. The company’s cost of capital is 12% and corporate tax rate for the company is 30%.
Details of the machines are as follows:
Machine I Machine II

Cost of machine ` 10,00,000 ` 15,00,000


Expected life 5 years 6 years

Annual income before tax and depreciation ` 3,45,000 ` 4,55,000


You are required to:
(i) Calculate the discounted pay-back period, net present value and internal rate of return for each
machine
(ii) Advise the management of P Limited as to which machine they should take up.
Ans.: Calculation of CFAT:
Particulars Machine I Machine II
Annual income before tax and depreciation 3,45,000 4,55,000
(-) Depreciation (2,00,000) (2,50,000)
Profit before tax 1,45,000 2,05,000
(-) Tax @ 30% (43,500) (61,500)
PAT 1,01,500 1,43,500
(+) Depreciation 2,00,000 2,50,000
CFAT 3,01,500 3,93,500
Calculation of IRR: Machine I:
PV Factor Present Value
Year CFAT
12% 16% 12% 16%
1 3,01,500 0.893 0.862 2,69,240 2,59,893
2 3,01,500 0.797 0.743 2,40,296 2,24,015
3 3,01,500 0.712 0.641 2,14,668 1,93,262
4 3,01,500 0.636 0.552 1,91,754 1,66,428
5 3,01,500 0.567 0.476 1,70,951 1,43,514
PV Factor Present Value
Year CFAT
12% 1696 1296 1696

Total present value 10,86,909 9,87,112


(-) Initial investment (10,00,000) (10,00,000)
Net Present Value (NPV) 86,909 (12,888)
IRR = 12 + 86,909/86,909 + 12,888 x 4
= 12 + 86,909/99,797 x 4
= 12 + 3.48
= 15.4896
Machine II:
PV Factor Present Value
Year CFAT
1296 16% 12% 16%
1 3,93,500 0.893 0.862 3,51,396 3,39,197
2 3,93,500 0.797 0.743 3,13,620 2,92,371
3 3,93,500 0.712 0.641 2,80,172 2,52,234
4 3,93,500 0.636 0.552 2,50,266' 2,17,212
5 3,93,500 0.567 0.476 2,23,115 1,87,306
6 3,93,500 0.507 0.410 1,99,505 1,61,335
Total present value 16,18,074 14,49,655
(-) Initial investment (15,00,000) (15,00,000)
Net Present Value (NPV) 1,18,074 (50,345)
IRR = 12 + 1,18,074/1,18,074 + 50,345 x 4
= 12 + 1,18,074/1,68,419 x 4
= 12 + 2.80
= 14.8096
Calculation of discounted payback period:
Machine I Machine II
Year PV Cumulative PV PV Cumulative PV
1 2,69,240 2,69,240 3,51,396 3,51,396
2 2,40,296 5,09,536 3,13,620 6,65,016
3 2,14,668 7,24,204 2,80,172 9,45,188
4 1,91,754 9,15,958 2,50,266 11,95,454
5 1,70,951 10,86,909 2,23,115 14,18,569
6 - - 1,99,505 16,18,074

Project A =4 years + 10,00,000-9,15,958/1,70,951 x 12


= 4 years & 5.9 months
Project B =5 years + 15,00,000- 14,18,569/1,99,505 x 12
= 5 years & 4.9 months Ranking under various methods:
Particulars Machine I Machine II Ranking
Machine I Machine 13
Net present value (at 12%) 86,908 1,18,074 II I
Internal rate of return 15.48% 14.80% I II
Discounted payback period 4 years & 5.9 months 5 years & 4.9 months I II
Problem No. 19] A company has to make a choice between two machines X and Y. The two machines are
designed differently, but have identical capacity and do exactly the same job. Machine X cost ` 5,50,000
and will last for three years. It costs ` 1,25,000 per year to run. Machine Y is an economy model costing `
4,00,000, but will last for two years and costs ` 1,50,000 per year to run. These are real cash flows. The
costs are forecasted in Rupees of constant purchasing power. Opportunity cost of capital is 12%. Ignore
Taxes. Which machine company should buy?
t=1 t=2 t=3
PVIF 0.12 ` 0.8929 0.7972 0.7118
PVIF 0.12,2 = 1-6901
PVIF0.12,3 =2.4019
Ans.: Machine X: Present value of cash outflow:
Year Cash outflow PV Factors PV
0 5.50,000 1,0000 5,50,000
1 to 3 1.25,000 2.4019 3,00,238
8,50,238
Equivalent PV of annual cash outflow = 8,50,238/2.4019 = 3,53,986
Machine Y: Present value of cash outflow:
Year Cash outflow PV Factors PV
0 4,00,000 1,0000 4,00,000
1 to 2 1,50,000 1.6901 2,53,515
6,53,515
Equivalent PV of annual cash outflow = 6,53,515/1.6901 = 3,86,672
Analysis: Company should buy Machine X since equivalent annual cash outflow is less than that of
Machine Y.
Problem No. 20] A Ltd. is considering the purchase of a machine which will perform some operations
which are at present performed by workers. Machines X and Y are alternative models. The following
details are available:
Machine X Machine Y
? ?
Cost of machine 1,50,000 2,40,000
Estimated life of machine 5 years 6 years
Estimated cost of maintenance p.a. 7,000 11,000
Estimated cost of indirect material p.a. 6,000 8,000
Estimated savings in scrap p.a. 10,000 15,000
Estimated cost of supervision p.a. 12,000 16,000
Estimated savings in wages p.a. 90,000 1,20,000
Depreciation will be charged on straight line basis. The tax rate is 30%. Evaluate the alternatives according
to:
(i) Average rate of return method, and
(ii) Present value index method assuming cost of capital being 10%.
(The present value of 1.00 @ 10% p.a. for 5 years is 3.79 and for 6 years is 4.355)
Ans.: Calculation of CFAT:
Year Machine X Machine Y
Savings in scrap 10,000 15,000
Savings in wages 90,000 . 1,20,000
Cost of maintenance (7,000) (11,000)
Cost of indirect material (6,000) (8,000)
Cost of supervision (12,000) (16,000)
Depreciation (30,000) (40,000)
Profit before tax 45,000 60,000
(-) Tax @ 3096 (13,500) (18,000)
PAT 31,500 42,000
(+) Depreciation 30,000 40,000
CFAT 61,500 ' 82,000
ARR = Average PAT/Initial investment x 100
Machine X = 31,500/1,50,000 x 100 = 21%
Machine Y = 42,000/2,40,000 X 100 = 17.5%
Calculation of NPV:
Machine X:
Year CFAT 1096 Present Value
1 to 5 61,500 3.791 2,33,147
(-) Initial investment (1,50,000)
Net Present Value (NPV) 83,147
Machine Y:
Year CFAT
10% Present Value
1 to 6 82,000
4.355 3,57,110
(:) Initial investment (2,40,000)
Net Present Value (NPV) 1,17,110
Calculation of Profitability Index:
Profitability Index = Total Present Value/Initial Investment
Machine X = 2,33,147/1,50,000 = 1,554
Machine Y= 3,57,110/2,40,000 =1.488
Ranking under various methods:
Particulars Machine X Machine Y Ranking
Machine X Machine Y
Net present value (at 10%) 83,147 1,17,110 II I
Profitability index 1.554 1.488 I II
Average rate of return 21% 17.5% I II
Problem No. 21] ANP Ltd. is providing the following information:
Annual cost of saving ` 96,000
Useful life 5 years
Salvage value Zero
Internal rate of return 15%
Profitability index 1.05
Table of discount factor:

Discount factor 1 2 3 4 5 Total


15% 0.870 0.756 0.658 0.572 0.497 3.353
14% 0.877 0.769 0.675 0.592 0.519 3.432
13% 0.886 0.783 0.693 0.614 0.544 3.520
You are required to calculate:
(i) Cost of the project
(ii) Payback period
(iii) Net present value of cash inflow (iv) Cost of capital
Ans.: Calculation of Cost of project:
Annual cost saving = Cash inflow = ` 96,000 Useful life = 5 years IRR =15%
At 15% IRR, total present value of cash inflow is equal to initial cash outlay.
Total present value of cash inflow @ 15% for 5 years is 3.353 = 96,000 X 3.353 = 3,21,888
Thus, Project cost = ` 3,21,888. Calculation of payback period:
Year CFAT Cumulative
CFAT
1 96,000 96,000
2 96,000 1,92,000
3 96,000 2,88,000
4 96,000 3,84,000
5 96,000 4,80,000
Payback period:
Project A = 3 years + 3,21,888-2,88,000/96,000 x 12
= 3 years & 4.24 months Calculation of cost of capital
Probability Index = Total Present Value/Initial Investment
1.05 = Total Present Value/3,21,888
Total Present Value = 3,37,982
Year Cash Flow PV Factor PV

1 to 5 96,000 ` 96,000.x
96,000x = 3,37,982
x = 3.520
Looking at present value table, discount factor for 5 years is 3.520
Hence, Cost of capital =1396
Net present value = 3,37,982 - 3,21,888 = 16,094
CAPITAL RATIONING
Problem No. 22] In a capital rationing situation (investment limit ` 25 lakhs), suggest the most desirable
feasible combination on the basis of the following data (indicate justification):
(` in lakhs)
Project Initial outlay NPV
A 15 6.00
B 10 4.50
C 7.5 3.60
D 6 3.00
Projects B and C are mutually exclusive.
Ans.: Investment allocation strategy under capital rationing situation:
Project Investment NPV
A&B 25.00 10.50
A&C 22.50 9.60
A&D 21.00 9.00
B& D 16.00 7.50
C&D 13.50 6.60
From the above analysis it is observed that Project A & B combination gives highest NPV; hence project A
& B combination should be adopted as it will maximize wealth.
Problem No. 23] S Ltd. has ` 10,00,000 allocated for capital budgeting purposes. The following proposals
and associated profitability indexes have been determined:
Project Amount (` ) Profitability
Index
1 3,00,000 1.22
2 1,50,000 0.95
3 3,50,000 1.20
4 4,50,000 1.18
5 2,00,000 1.20
6 4,00,000 1.05
Which of the above investments should be undertaken? Assume that projects are indivisible and there is
no alternative use of the money allocated for capital budgeting.
Ans.: Ranking of Projects on profitability index:
Project Amount Profitability Index Rank
1 3,00,000 1.22 1
2 1,50,000 0.95 6
3 3,50,000 1.20 2
4 4,50,000 1.18 4
5 2,00,000 1.20 3
6 4,00,000 1.05 5
Calculation of NPV:
Project . Amount Profitability Cash Inflow (2) X (3) NPV (4) - (2)
Index
(1) (2) (3) (4) (5)
1 3,00,000 1.22 3,66,000 66,000
2 1,50,000 0.95 1,42,500 (7,500)
3 3,50,000 1.20 4,20,000 70,000
4 4,50,000 1.18 5,31,000 81,000
5 2,00,000 1.20 2,40,000 40,000
6 4,00,000 1.05 4,20,000 20,000
Selection of Projects:
(1) Profitability Index Method: Assuming the projects are indivisible and there is no alternative use of
unutilized amount, S Ltd. is advised to undertake investment in Projects 1, 3 & 5, which will give NPV of `
1,76,000 and unutilized amount will be ` 1,50,000.
(2) Net Present Value Method: As per this method Projects 3, 4 & 5 can be undertaken which will `
1,91,000 and no money will remain unspent.
Suggestion: From the above analysis, we can observe that, selection of Projects under NPV Method will
maximize S Ltd. net cash inflow by ` 15,000 (1,91,000 - 1,76,000). Hence, it is suggested to undertake
Projects 3, 4 & 5.
Problem No. 24] Delhi Machinery Manufacturing Company wants to replace the manual operation by the
new machine. Using payback period, suggest the most profitable investment. Ignore taxation.
Machine-X Machine-Y
Original investment ` 4,000 ` 18,000
Estimated life of machine 4 years 5 years

Estimated saving in scrap ` 500 ` 800

Estimated saving in wages ` 6,000 ` 8,000

Additional cost of maintenance ` 800 ` 1,000

Additional cost of supervision ` 1,200 ` 1,800


[CS (Professional) - Dec. 1993] (5 Marks)
Ans.: Computation of cash flow:
Particulars Machine-X Machine-Y
Cash inflow: .
- Saving in scrap 500 800
- Saving in wages 6,000 8,000
Cash outflow:
- Cost of maintenance (800) (1,000)
- Cost of supervision (1,200) (1,800)
Net cash inflow 4,500 6,000

Machine-X
Year Cash flow Cumulative cash flow
1 4,500 4,500
2 4,500 9,000
3 4,500 13,500
4 4,500 18,000

Machine-Y
Year Cash flow Cumulative cash flow
1 6,000 6,000
2 6,000 12,000
3 6,000 18,000
4 6,000 24,000
5 6,000 30,000
Payback Period:
Machine X: 4,000/4,500 X 12 = 10.67 months
Machine Y: 3 years
Analysis: Since payback period of Machine-X is less it is advised to Delhi Machinery Manufacturing
Company to replace the manual operation by the Machine-X.
Problem No. 25] Indo Plastics Ltd. is a manufacturer of high quality plastic products. Rasik, President, is
considering computerizing the company’s ordering, inventory and billing procedures. He estimates that
the annual savings from computerization include a reduction of 4 clerical employees with annual salaries
of ` 50,000 each. ` 30,000 from reduced production delays caused by raw materials inventory problems. `
25,000 from lost sales due to inventory stock-outs and ` 18,000 associated with timely billing procedures.
The purchase price of the system is ` 2,50,000 and installation costs are ` 50,000. These outlays will be
capitalized (depreciated) on a straight-line basis to a zero books salvage value, which is also its market
value at the end of five years. Operation of the new system requires two computer specialists with annual
salaries of ` 80,000 per person. Also annual maintenance and operating (cash) expenses of ` 22,000 are
estimated to be required. The company’s tax rate is 40% and its required rate of return (cost of capital)
for this project is 12%.
You are required to:
(i) Find the project’s initial net cash outlay.
(ii) Find the project’s operating and terminal value cash flows over its 5 year life.
(iii) Evaluate the project using NPV method.
(iv) Evaluate the project using PI method.
(v) Calculate the project’s payback period.
(vi) Find the project’s cash flows and NPV [parts (I) through (iii)] assuming that the system can be sold for
` 60,000 at the end of five years even though the book salvage value will be zero, and
(vii) Find the project’s cash flows and NPV [parts (i) through (iii)] assuming that the book salvage value for
depreciation purposes is ` 40,000 even though the machine is worthless in terms of its resale value.
Note: Present value of annuity of Re. 1 at 12% rate of discount for 5 years is 3.605
Present value of ` 1 at 12% rate of discount, received at the end of 5 years is 0.567.
[CS (Professional) - June 1996 & Dec. 2002] (20 Marks)
Ans.: (i) Calculation of initial investment:

Particulars `
Purchase price of the system 2,50,000
Installation costs 50,000
Initial investment 3,00,000
(ii) Calculation of CFAT:

Particulars `
Savings (Cash inflow):
- Reduction of 4 clerk salaries (50,000 X 4) 2,00,000
- Reduction in production delays 30,000
- Reduction in lost sales 25,000
- Gain due to timely billing procedures 18,000
Expenses (Cash outflow):
- Computer specialists annual salaries (80,000 X 2) (1,60,000)
- Maintenance & operating expenses (22,000)
Cash profit 91,000
Particulars ?
(-) Depreciation (3,00,000/5) (60,000)
Profit before tax 31,000
(-) Tax @ 40% (12,400)
PAT 18,600
(+) Depreciation 60,000
CFAT 78,600
(iii) Evaluation of project using NPV method:
Year CFAT PV Factor 12% PV
1 to 5 78,600 3.605 , 2,83,353
(-) Initial investment (3,00,000)
Net Present Value (NPV) (16,647)
Analysis: NPV is negative, hence project is not acceptable.
(iv) Evaluation of project using profitability index:
Profitability Index = 2,83,353/3,00,000 = 0.945
Since, PI is less than 1; Project is not acceptable.
(v) Calculation of payback period:
Year CFAT Cumulative
CFAT
1 78,600 78,600
2 78,600 1,57,200
3 78,600 2,35,800
4 78,600 3,14,400
5 78,600 3,93,000

Payback Period = 3 years + 3,00,000-2,35,800/78,600 x 12


= 3 years & 9.8 months
(vi) Evaluation of project using NPV method assuming that system can be sold for ` 60,000 at the end
of 5 years:
Year CFAT PV Factor 12% PV
1 to 5 78,600 3.605 2,83,353
5 *36,000 0.567 20,412
Total PV 3,03,765
(-) Initial investment (3,00,000)
Net Present Value (NPV) 3,765
Since, NPV is positive, project can be selected.
* Post-tax salvage value = 60,000 (1 - 0.4) = 36,000
Original cost of machine 3,00,000
(-) Depreciation for 5 years (3,00,000)
Book value at the end of 5 years 0
(+) Profit on sale 60,000
Sale price 60,000
(-) Tax on profit (60,000 X 40%) (24,000)
Net cash inflow 36,000
(vii) Evaluation of project’s cash flows and NPV assuming that the book salvage value for depreciation
purposes is ` 40,000 even though the machine is worthless in terms of its resale value:
Cash profit 91,000
(-) Depreciation (3,00,000 - 40,000/5) (52,000)
Profit before tax 39,000
(-) Tax @ 40% (15,600)
PAT 23,400
(+) Depreciation .52,000
CFAT 75,400
Evaluation of project using NPV method:
Year CFAT PV Factor 12% PV
1 to 5 75,400 3.605 2,71,817
5 ‘16,000 0.567 9,072
Total Present Value 2,80,889
(-) Initial investment (3,00,000)
Net Present Value (NPV) (19,111)
Analysis: NPV is negative, hence project is not acceptable.
Original cost of machine 3,00,000
(-) Depreciation for 5 years (2,60,000)
Book value at the end of 5 years 40,000
(-) Loss on sale (40,000)
Sale price 0
(+) Tax benefit on loss (40,000 X 40%) 16,000
Net cash inflow *16,000
Problem No. 26] A Company is contemplating to purchase a machine. Two machines A and B are available,
each costing ` 5,00,000. In comparing the profitability of the machines, a discounting rate of 10% is to be
used and machine is to be written off in five years on straight line method of depreciation with nil residual
value. Cash inflows after tax are expected as follows:
Year Machine A Machine B
1 1,50,000 50,000
2 2,00,000 1,50,000
3 2,50,000 2,00,000
4 1,50,000 3,00,000
5 1,00,000 2,00,000
Indicate which machine would be profitable using the following methods of ranking investment proposals:
(1) Payback Method
(2) Net Present Value Method
(3) Profitability Index Method
(4) Internal Rate of Return Method
(5) Accounting Rate of Return Method
(6) Discounted Payback Period Method
[CS (Professional) - June 1998] (20 Marks)
Ans.:
Machine A Machine B
Year CFAT Cumulative CFAT CFAT Cumulative CFAT
1 1,50,000 1,50,000 50,000 50,000
2 2,00,000 3,50,000 1,50,000 2,00,000
3 2,50,000 6,00,000 2,00,000 4,00,000
4 1,50,000 7,50,000 3,00,000 7,00,000
5 1,00,000 8,50,000 2,00,000 9,00,000
Computation of payback period:
Machine A =2 years + 5.00. 000-3,50,000/2,50,000 x 12
= 2 years & 7.2 months
= 5,00,000 - 4,00,000
Machine B =3 years + 5,00,000 - 4,00,000/3,00,000 x 12
= 3 years & 4 months
Since, company's cost of capital is 10%; NPV at 10% has to be calculated. Problem also asks to calculate
IRR; so let’s calculate IRR directly.
Machine A:
PV Factor Present Value
Year CFAT
10% 20% 10% 20%
1 1,50,000 0.909 0.833 1,36,350 1,24,950
2 2,00,000 0.826 0.694 1,65,200 1,38,800
3 2,50,000 0.751 0.579 1,87,750 1,44,750
4 1,50,000 0.683 0.482 1,02,450 72,300
5 1,00,000 0.621 0.402 62,100 40,200
Total present value 6,53,850 5,21,000
(-) Initial investment (5,00,000) (5,00,000)
Net Present Value (NPV) 1,53,850 21,000
IRR = 10 + 1,53,850/1,53.850 = 21,000 x 10
= 10 + 1,53,850/1,32,850 x 10
= 10 + 11.58
= 21.5896
Machine B:
PV Factor Present Value
Year CFAT
10++ : :: 20'i 1096 20%
1 50,000 0.909 0.833 45,450 41,650
2 1,50,000 0.826 0.694 1,23,900 1,04,100
3 2,00,000 0.751 0.579 1,50,200 1,15,800
4 3,00,000 0.683 0.482 2,04,900 1,44,600
5 2,00,000 0.621 0.402 1,24,200 80,400
Total present value 6,48,650 4,86,550
(-) Initial investment (5,00,000) (5,00,000)
Net Present Value (NPV) 1,48,650 (13,450)
IRR = 10 + 1,48,650/1,48,650 + 13,450 x 10
= 10 + 1,48,650/1,62,100 x 10
= 10 + 9.17
= 19.1796
Calculation of Profitability Index:
Profitability Index = Total Present Value/Initial Investment
Machine A = 6,53,850/5,00,000 - 1.308
Machine B = 6,48,650/5,00,000 = 1.297
Calculation of ARR:
Particulars Machine A Machine B
Total CFAT 8,50,000 9,00,000
(-) Depreciation for 5 years (5,00,000) (5,00,000)
PAT 3,50,000 4,00,000
Average PAT 70,000 80,000
ARR = Average PAT/Initial investment x 100
Machine A = 70,000/5,00,000 x 100= 14%
Machine B = 80,000/5,00,000 X 100 = 1696
Calculation of discounted payback period:
Machine A Machine B
Present Value Cumulative PV Present Value Cumulative PV
1 1,36,350 1,36,350 45,450 45,450
2 1,65,200 3,01,550 1,23,900 1,69,350
3 1,87,750 4,89,300 1,50,200 3,19,550

4 1,02,450 5,91,750 2,04,900 5,24,450


5 62,100 6,53,850 1,24,200 6,48,650

Project A = 3 years + 5,00,000-4,89,300/1,02,450 x 12


= 3 years & 1.25 months
Project B = 3 years + 5,00,000-3,19,550/2,04,900 x 12
= 3 years & 10.57 months Ranking under various methods:
Particulars Machine A Machine B Ranking
Machine A Machine B
Payback period 2 years & 7.2 months 3 years & 4 months I U
Net present value (at 10%) 1,53,850 1,48,650 I n
Profitability index 1.308 1.297 I II
Internal rate of return 21.5896 19.1796 I II
Average rate of return 1496 1696 II I
Discounted payback 3 years & 1.25 3 years & 10.57 I II
period months months
Problem No. 27] EEC Ltd. is considering the purchase of a machine. Two machine LM and PM are available
each costing ` 1,00,000. Both machines will last for 5 years with no residual value. In comparing the
profitability of machine, a discount rate of 10% is to be used. Earning after taxation and charging
depreciation on straight line are expected to be as follows:
Year LM PM

(`) (`)
1 10,000 (10,000) Loss
2 20,000 10,000
3 30,000 20,000
4 10,000 40,000
5 Nil 20,000
Indicate which machine would be a more profitable investment under the various methods of ranking
investment proposals, viz ARR, Payback, NPV, and Profitability Index.
[CS (Professional) - Dec. 1998] (20 Marks)
Ans.:
Computation of CFAT of Machine LM:
Year 1 2 3 4 5
Profit after depreciation & tax 10,000 20,000 30,000 10,000 -
(+) Depreciation 20,000 20,000 20,000 20,000 20,000
Cash flow after tax (CFAT) 30,000 40,000 50,000 30,000 20,000
Computation of CFAT of Machine PM:
Year 1 2 3 4 5

Profit after depreciation & tax (10,000) 10,000 20,000 40,000 20,000
(+) Depreciation 20,000 20,000 20,000 20,000 20,000
Cash flow after tax (CFAT) 10,000 30,000 40,000 60,000 40,000
Calculation of ARR:
ARR = Average PAT/Initial investment x 100
Machine LM = 14,000/1,00,000 x 100 = 14%
Machine PM = 16,000/1,00,000 x 100 = 16%
Calculation of payback period:
LM PM
Year CFAT Cumulative CFAT Cumulative
CFAT CFAT
1 30,000 30,000 10,000 10,000
2 40,000 70,000 30,000 40,000
3 50,000 1,20,000 40,000 80,000
4 30,000 1,50,000 60,000 1,40,000
5 20,000 1,70,000 40,000 1,80,000
Payback Period:
Machine LM = 2 years + 1,00,000 - 70,000/50,000 x 12
= 2 years & 7.2 months
Machine PM = 3 years + 1,00,000-80,000/60,000 x 12
= 3 years & 4 months
Calculation of NPV:
Year CFAT PV Factor Present Value
LM PM 10% LM PM
1 30,000 10,000 0.909 27,270 9,090
2 40,000 30,000 0.826 33,040 24,780
3 50,000 40,000 0.751 37,550 30,040
4 30,000 60,000 0.683 20,490 40,980
5 20,000 40,000 0.621 12,420 24,840
Total present value 1,30,770 1,29,730
(-) Initial investment (1,00,000) (1,00,000)
Net Present Value (NPV) 30,770 29,730
Calculation of benefit Profitability Index:
Profitability Index = Total Present Value/Initial Investment
Machine LM = 1,30,770/1,00,000 = 1.308
Machine PM = 1,29,730/1,00,000 = 1.297
Ranking under various methods:
Particulars Machine Machine Ranking Reason
LM PM
Machine Machine
LM PM
Accounting rate of 14% 16% II I Project having higher ARR will rank
return first.
Payback period 2 years & 7.2 3 years & 4 I II Project having lower payback
months months period will rank first.
Net present value (At 30,770 29,730 I II Project having higher NPV will
10%) rank first.
Profitability 1.308 1.297 I II Project having higher PI will rank
index first.'

Problem No. 28] Project X involves an initial outlay of ` 16.2 million. Its life span is expected to be 3 years.
The cash streams generated by it are expected to be as follows:
Year 1 2 3
Cash inflow (` in millions) 8 7 6
You are required to calculate IRR.
[CS (Professional) - Dec. 1999] (8 Marks)
Ans.:
Calculation of IRR: R in Millions)
Year CFAT PV Factor Present Value
10% . 20% 10% 20%
1 8 0.909 0.833 7.272 6.664
2 7 0.826 0.694 5.782 4.858
3 6 0.751 0.579 4.506 3.474
Total present value 17.56 14.996
(-) Initial investment (16.2) (16.2)
Net Present Value (NPV) 1.36 (1.204)
** Since NPV at 10% is positive, higher rate 20% is taken.
IRR = 10 + 1.36/1.36 + 1.204 x 10
= 10 + 1.36/2.564 x 10
= 10 + 5.30 = 15.30%
Problem No. 29] The initial outlay of the project is ` 1,00,000 and it generates cash inflow of ` 50,000, `
40,000, ` 30,000 and ` 20,000 in the four years of its life span.
You are required to calculate:
(1) Net present value (NPV)
(2) Profitability index (PI)
[CS (Professional) - Dec. 1999] (12 Marks)
Ans.:
Computation of Net Present Value (NPV):
Year CFAT PV Factor PV
10% 10%
1 50,000 0.9091 45,455
2 40,000 0.8264 33,056
3 30,000 0.7513 22,539
4 20,000 0.6830 13,660
Total present value 1,14,710
(-) Initial investment (1,00,000)
Net Present Value (NPV) 14,710
Profitability Index = Total present value/Initial investment = 1,14,710/1,00,000 = 1.147
Problem No. 30] Software Enterprise is considering the purchase of computer system for its research and
development division which would cost ` 35,00,000. The operation and maintenance cost excluding
depreciation are expected to be ` 7,00,000 per annum. It is estimated that the useful life of the system
would be 6 years, at the end of which the disposal value is expected to be ` 1,00,000.
The tangible benefit expected from the system in the form of reduction in design and draftsmanship costs
would be ` 12,00,000 per annum. The disposal of used drawings, office equipment and furniture initially
is anticipated to net ` 9,00,000.
As a capital expenditure in research and development, the proposal would attract a 100% write-off for tax
purpose. The gain arising from disposal of used assets may be considered tax-free. The corporate tax rate
is 35%. The required rate of return of the company is 12%.
After appropriate cash flow analysis, advise company of the financial viability of the proposal. Ignore tax
value of salvage. [CS (Professional) - June 2000] (14 Marks)
Ans.:
Calculation of initial investment:
Cost of computer system 35,00,000
(-) Disposal of used drawings, office equipment and furniture (9,00,000)
Net investment 26,00,000
Computation of CFAT:
Cash inflow due to reduction in design and draftsmanship costs 12,00,000
(-) Operation and maintenance cost (7,00,000)
(-) Depreciation (35,00,000 - l,00,000)/6 (5,66,667)
(66,667)
(+) Tax benefit on loss (66,667 X 35%) 23,333
(43,334)
(+) Depreciation 5,66,667
CFAT 5,23,333
Calculation of Net Present Value (NPV):
PV Factor PV
Year CFAT
12% 12%
1 to 6 5,23,333 4.111 21,51,422
6 1,00,000 0.507 50,700
Total present value 22,02,122
(-) Initial investment (26,00,000)
Net Present Value (NPV) (3,97,878)
Analysis: Since NPV is negative, hence proposal is not acceptable.
Problem No. 31] Nishce Ltd. is an all equity financed company. The current market price of the share is `
180. It had just paid a dividend of ` 15 per share and expected future growth in dividends is 12%. Currently,
it is the evaluating a proposal requiring funds of ` 20,00,000 with annual inflow of ` 10,00,000 for 3 years.
Find out the NPV of the proposal if:
(1) It is financed from retained earnings.
(2) It is financed by issuing fresh equity (floatation costs 5%)
[CS (Professional) - Dec. 2000] (10 Marks)
Ans.: If financed from retained earnings:
Ke = D1/P0 + g D1 = Do (1 + g)
= 15(1+0.12)
= 16.58/180 +0.12 =16.8
= 0.2133 i.e. 21.3396
Year CFAT PV Factor 21.3396 PV
1 to 3 10,00,000 2.0634 20,63,400
(-) Initial investment (20,00,000)
Net Present Value (NPV) 63,400
If financed from by issuing fresh equity:
Flotation cost = 180 X 5% = 9
Ke = D1/NP + g
Net Proceeds = 180 - 9
= 16.8/171 + 0.12 = 171
= 0.2182 i.e. 21.82 96
ear CFAT PV Factor 21.8296 PV
1 to 3 10,00,000 2.0479 20,47,900
(-) Initial investment (20,00,000)
Net Present Value (NPV) 47,900
Problem No. 32] Xpert Engineering Ltd. is considering buying one of the following two mutually exclusive
investment projects:
Project-A Buy a machine that requires an initial investment outlay of ` 1,00,000 and will generate the cash
flows after tax (CFAT) of ` 30,000 per year for 5 years.
Project-B Buy a machine that requires an initial investment outlay of ` 1,25,000 and will generate the cash
flows after tax (CFAT) of ` 27,000 per year for 8 years.
Which project should be undertaken? The company uses 10% cost of capital to evaluate the projects.
[CS (Professional) - Dec 2001] (6 Marks)
Ans.: Calculation of NPV of Project-A:
Year CFAT PV Factor 1096 PV
1 to 5 30,000 3.791 1,13,730
(-) Initial investment (1,00,000)
Net Present Value (NPV) 13,730
Calculation of NPV of Project-B:
Year CFAT PV Factor 1096 PV
1 to 8 27,000 5.335 1,44,045
(-) Initial investment . (1,25,000)
Net Present Value (NPV) 19,045
Equivalent NPV:
Project-A =13,730/3.791 = 3,622
Project-B = 19,045/5.335 = 3,570
Analysis: Company should undertake Project-A since equivalent NPV is more as compared to Project-B.
Problem No. 33] Playmates Ltd. manufactures toys and other short lived fad items. The research and
development department has come up with an item that would make a good promotional gift for office
equipment dealers. As a result of efforts by the sales personnel, the firm has commitments for this
product. To produce the quantity demanded, Playmates Ltd. will need to buy additional space. It appears
that about 25,000 sq. ft. will be needed; 12,500 sq. ft. of presently unused space, but leased at the rate of
` 3 per sq. ft. per year, is available. There is another 12,500 sq. ft. adjoining facility available at the annual
rent of ` 4 per sq. ft.

The equipment will be purchased for ` 9,00,000. It will require ` 30,000 in modification and ` 1,50,000 for
installation. The equipment will have salvage value of about ` 2,80,000 at the end of the third year. It is
subject to 25% depreciation on reducing balance basis. The firm has no other assets in this block. No
additional general overhead costs are expected to be incurred.
The estimates of revenues and costs for this product for three years have been developed as follows:
Year 1 Year 2 Year 3
Sales 10,00,000 20,00,000 30,00,000
Less: Cost: _
- Material, labour & overheads 4,00,000 7,50,000 3,50,000
- Overheads allocated 40,000 75,000 35,000
- Rent 50,000 50,000 50,000
- Depreciation 2,70,000 2,02,500 Nil
Total costs 7,60,000 10,77,500 4,35,000
Earnings before taxes 2,40,000 9,22,500 25,65,000
Less: Taxes 84,000 3,22,875 8,97,750
Earnings after taxes 1,56,000 5,99,625 16,67,250
If the company sets a required rate of return of 20% after taxes, should this project be accepted?
[CS (Professional) - June 2002] (20 Marks)
Ans.: Calculation of tax rate:
Tax rate = 84,000/2,40,000 x 100 = 55%
Calculation of initial investment:
Cost of equipment 9,00,000
Modification & installation cost (30,000 + 1,50,000) 1,80,000
Initial investment 10,80,000
Opportunity cost of lease rent lost = 12,500 sq. ft. X 3 = 37,500.
Calculation of depreciation and net cash inflow on disposal of asset:
Cost of investment in project 10,80,000
(-) Depreciation of 1st year @ 25% (2,70,000)
8,10,000
(-) Depreciation of 2nd year @ 25% (2,02,500)
6,07,500
(-) Loss on sales (3,27,500)
Sales price 2,80,000
(+) Tax benefit on loss (3,27,500 X 35%) 1,14,625
Net cash inflow from sale of asset at the end of 3 years 3,94,625
Calculation of CFAT:
Year 1 Year 2 Year 3
Sales 10,00,000 20,00,000 30,00,000
(-) Cost: (4,00,000) (7,50,000) (3,50,000)
- Material, labour & overheads
- Rent (50,000) (50,000) (50,000)
- Depreciation (2,70,000) (2,02,500) Nil
- Opportunity cost of lease rent lost (37,500) (37,500) (37,500)
Earnings before taxes 2,42,500 9,60,000 25,62,500
(-) Taxes @35% (84,875) (3,36,000) (8,96,875)
Earnings after taxes 1,57,625 6,24,000 16,65,625
(+) Depreciation 2,70,000 2,02,500 Nil
CFAT 4,27,625 8,26,500 16,65,625
Cash inflow from sale - .- 3,94,625
Net CFAT 4,27,625 8,26,500 20,60,250
Calculation of NPV:
Year CFAT PV Factor 20% PV
1 4,27,625 0.833 3,56,212
2 8,26,500 0.694 5,73,591
3 20,60,250 0.579 11,92,885
Total present value 21,22,688
(-) Initial investment (10,80,000)
Net Present Value (NPV) 10,42,688
Analysis: Since NPV is positive, project can be accepted.
Problem No. 34] Sell-Well Ltd. is considering to install a large stamping machine. Two machines being
considered are as follows:
Machine-A: It costs ` 50,000 and will require cash running expenses of ` 15,000 p.a. It has useful life of 6
years and thereafter, it is expected to yield ` 2,000 salvage value.
Machine-B: It costs ` 65,000 and will require cash running expenses of ` 12,000 p.a. It has useful life of 10
years and thereafter, it is expected to yield ` 5,000 salvage value.
Both machines would be depreciated on straight line basis. Corporate tax rate is 50%. Cost of capital is
10%. Which machine should be brought by the Sell-Well Ltd.?
[CS (Professional) - June 2003] (12 Marks)
Ans.: Computation of depreciation:
Machine-A = 50,000-2,000/6 = 8,000
Machine-B = 65,000-5,000/10 = 6,000
Computation of tax benefit on depreciation: Machine-A = 8,000 X 50% = 4,000 Machine-B = 6,000 X 50%
= 3,000 Calculation of net cash outflow:
Particulars Machine-A Machine-B
Cash running expenses 15,000 12,000
(-) Tax benefit @ 50% (7,500) (6,000)
(-) Tax benefit on depreciation (4,000) (3,000)
Net cash outflow 3,500 3,000
Present value of cash outflow of Machine A:
Year Cash outflow PV Factor 10% PV
0 50,000 1,000 50,000
1 to 6 3,500 4.355 15,243
6 *(2,000) 0.564 (1,128)
64,115
* Cash inflow from sale of machine at the-end of 6th year.
Equivalent PV of annual cash outflow = 64,115/4.355 = 14,722
Present value of cash outflow of Machine B:
Year Cash outflow PV Factor 10% PV
0 65,000 1,000 65,000
1 to 10 3,000 6.145 18,435
*(5,000) 0.386 (1,930)
10
81,505
* Cash inflow from sale of machine at the end of 10th year.
Equivalent PV of annual cash outflow = 81,505/6.145 = 13,264
Analysis: Since, equivalent PV of annual cash outflow of Machine-B is less, it is advised to purchase
Machine-B.
Problem No. 35] Shivalik Ltd. is currently considering a project which will yield the following returns over
a period of time:
Year Gross Yield (` )
1 1,00,000
2 1,00,000
3 90,000
4 90,000
5 80,000

Initial investment in the machinery works to ` 2,50,000 and depreciation to be charged on machinery
would be 25% p.a. on written down value basis with its residual value in the 5th year. Scrap value at the
end of 5th year is nil. Income-tax rate applicable is 35%. If average cost of capital is 12%, advise the
management whether project should be accepted as per internal rate of return method.
[CS (Professional) - June 2004] (8 Marks)
Ans.: Calculation of depreciation and cash flow in last year:
Cost of asset 2,50,000
(-) Depreciation for Year 1 (62,500)
1,87,500
(-) Depreciation for Year 2 (46,875)
1,40,625
(-) Depreciation for Year 3 (35,156)
1,05,469
(-) Depreciation for Year 4 (26,367)
79,102
(-) Depreciation for Year 5 (19,775)
Book value at the end of 5th year 59,327
(-) Loss on sale (59,327)
Sale price Nil
(-} Tax benefit on loss (59,327 X 35%) 20,764
Net cash flow on sale of asset 20,764
Calculation of CFAT:
Particulars Year 1 Year 2 Year 3 Year 4 Year 5
Gross Yield 1,00,000 1,00,000 90,000 90,000 80,000
(-) Depreciation (62,500) (46,875) (35,156) (26,367) (19,775)
EBT 37,500 53,125 54,844 63,633 60,225
(-) Tax @ 35% (13,125) (18,594) (19,195) (22,272) (21,079)
PAT 24,375 34,531 35,649 41,361 39,146
(+) Depreciation 62,500 46,875 35,156 26,367 19,775
CFAT 86,875 81,406 70,805 67,728 58,921
Net cash flow on sale of asset - - - - 20,764
Net CFAT 86,875 81,406 70,805 67,728 79,685

PV Factor Present Value


Year CFAT
12% 18% 12% 18%
1 86,875 0.893 0.847 77,579 73,583
2 81,406 0.797 0.718 64,881 58,450
3 70,805 0.712 0.609 50,413 43,120
4 67,728 0.636 0.516 43,075 34,948
5 79,685 0.567 0.437 45,181 34,822
Total present value 2,81,129 2,44,923
(-) Initial investment (2,50,000) (2,50,000)
Net Present Value (NPV) 31,129 (5,077)
IRR = 12 + 31,129/31,129 + 5,077 x 6
= 12 + 31,129/36,206 x 6
= 12 + 5.16
17.16%
Problem No. 36] The management of Urmila Ltd. is considering an investment project costing ` 1,50,000
and it will have a scrap value of ` 10,000 at the end of its 5 years life. Transportation charges and
installation charges are expected to be ` 5,000 and ` 25,000 respectively. If the project is accepted, a spare
part inventory of ` 10,000 must also be maintained. It is estimated that the spare parts will have an
estimated scrap value of 60% of their initial cost after 5 years. Annual revenue from the project is expected
to be ` 1,70,000; and annual labour, material and maintenance expenses are estimated to be ` 15,000, `
50,000 and ^ 5,000 respectively. The depreciation and taxes for five years will be:
Year Depreciation Tax

(`) (`)
1 72,000 11,200
2 43,200 22,720
3 32,400 27,040
4 21,600 31,360
5 800 39,680
Calculate the net cash flows for each year and cost of the project. Evaluate the project at 12% rate of
interest.
[CS (Professional) - Dec 2004 & Dec 2014] (10 Marks)
Ans.: Calculation of initial investment:
Particulars `
Cost of machine 1,50,000
Transportation charges 5,000
installation charges 25,000
Spare part inventory 10,000
Initial investment 1,90,000
Calculation of scrap value:

Particulars `
10,000
6,000
Scrap value of machine Scrap value of spare part Net scrap value 16,000
Calculation of CFAT:
Year 1 2 3 4 5
Revenue 1,70,000 1,70,000 1,70,000 1,70,000 1,70,000
Costs:
Material 50,000 50,000 50,000 50,000 50,000
Labour 15,000 15,000 15,000 15,000 15,000
Maintenance expenses 5,000 5,000 5,000 5,000 5,000
Depreciation 72,000 43,200 32,400 21,600 800
1,42,000 1,13,200 1,02,400 91,600 70,800
Profit before tax 28,000 56,800 67,600 78,400 99,200
(-) Tax (11,200) (22,720) (27,040) (31,360) (39,680)
Year 1 2 3 4 5
PAT 16,800 34,080 40,560 47,040 59,520
(+) Depreciation 72,000 43,200 32,400 21,600 800
CFAT 88,800 77,280 72,960 68,640 60,320
Net scrap value - - - - 16,000
Net CFAT 88,800 77,280 72,960 68,640 76,320
Calculation of net present value:
Year CFAT PV Factor 12% Present Value
1 88,800 0.8929 79,290
2 77,280 0.7972 61,608
3 72,960 0.7118 51,933
4 68,640 0.6355 43,621
5 76,320 0.5674 43,304
Total present value 2,79,756
(-) Initial investment (1,90,000)
Net Present Value (NPV) 89,756
Analysis: Since NPV of the project is positive, it is advised to take the project.
Problem No. 37] Madhuri Ltd. is evaluating a project for which the initial investment required is ` 50 lakh
to be met by internally generated funds of ` 10 lakh, from a rights issue of ` 15 lakhs and the rest from a
term loan @12% per annum. Rights issue will involve flotation cost of 5% and the term loan processing
will cost 1%. Corporate tax rate is 40%. The risk-free rate of interest is 6.5%, market return is 15% and the
relevant asset beta for the investment is estimated to be 1.5. Net operating cash inflows after tax from
the project are:
Year-1: l 15 lakhs; Year-2: ` 35 lakhs; and Year-3: ` 15 lakhs.
Besides these cash inflows, residual value of ` 5 lakhs (net of taxes) is also expected at the end of third
year. Should the project be taken up?
[CS (Professional) - Dec 2005] (8 Marks)
Ans.: Calculation of cost of capital
Ke =Rf + p(Rm-Rf)
= 6.5 + 1.5(15-6.5)
= 19.25%
Calculation of present value of cash inflow of project:
Year CFAT PV Factor 19.25% PV
1 15,00,000 0.839 12,58,500
2 35,00,000 0.703 24,60,500
3 20,00,000 0.590 11,80,000
Total present value 48,99,000
(-) Initial investment (50,00,000)
Net Present Value (NPV) (1,01,000)
Initial investment required = 50,00,000
Internally generated funds = 10,00,000
Right issue = 15,00,000
12% Term loan = 25,00,000
The term loan processing cost is 1%. Hence, loan amount to be raised -
= 25,00,000/100-1 x 100 = 25,25,253
Rights issue has flotation cost of 5%. Hence, amount to be raised by right issue -
= 15,00,000/100 - 5 x 100 = 15,78,947
Calculation of adjusted NPV:
Base NPV (1,01,000)
Flotation cost of right issue (15,78,947 - 15,00,000) (78,947)
Processing cost of term loan (25,25,253 - 25,00,000) (25,253)
Tax benefit on interest on term loan (25,25,253 X 12% X 40% X 2.402) 2,91,152
Adjusted NPV 85,952
Analysis: Since, adjusted.NPV is positive, it is suggested to take the project.
Problem No. 38] Karishma Ltd. is considering to manufacture a new product which will involve use of a
new machine costing ` 1,50,000 and an existing machine, which was purchased 2 year ago at a cost of `
80,000, having current book value of ` 60,000. There is sufficient underutilized capacity on this machine.
It is also estimated that annual sales of the product will be 5,000 units at ` 32 per unit with following cost
composition:
?
Direct material 7

Direct labour (4 Hours per unit @ ` 2 per unit) 8

Fixed cost (including depreciation) 9


24

The project would have 5 year life, with residual value of ` 10,000 for new machine. Direct labour being
continuously in short supply, the labour resources would have to be diverted from other work, currently
earning a profit of ` 1.50 per direct labour hour. Fixed overheads absorption rate would be ` 2.25 per hour
and actual expenditure on fixed overheads will not change. The requirement of working capital would be
` 10,000 at the beginning of first year, ` 15,000 in the second and subsequent years till the end of the
project when it will be recovered. The company’s cost of capital is 20%. Ignoring tax implications, decide
if the project is worth accepting.
[CS (Professional) - Dec 2006] (20 Marks)
Ans.:
Depreciation on new machine = 1,50,000- 10,000/5 = 28,000
Calculation of initial investment:
Cost of machine 1,50,000
(+) Working capital 10,000
1,60,000
In addition to above working capital for subsequent year is 15,000. Thus additional working capital for
subsequent year will be ` 5,000 as ` 10,000 will be introduced at the beginning of project.
Calculation of cash inflow per annum-
Sales (5,000 X 32) 1,60,000
Costs:
- Material (5,000 X 7) 35,000
- Labour (5,000 X 8) 40,000
- Opportunity cost of direct labour (5,000 X4X 1.5). 30,000
- Cash fixed costs (5,000X9)-2 8,000 17,000
Total cost 1,22,000
Cash inflow per annum 38,000
Calculation of present value of cash inflow:
Year CFAT PV Factor 20% PV
1 to 5 38,000 2.991 1,13,658
5 *15,000 0.402 6,030
5 **10,000 0.402 4,020
1,23,708
* Working capital realized.
** Residual value of machine.
Calculation of present value of cash outflow:
Year CFAT PV Factor 20% PV
0 1,60,000 1,000 1,60,000
1 5,000 0.833 4,165
1,64,165
Net Present Value = Present value of cash inflow - Present value of cash outflow = 1,23,708- 1,64,165 =
(40,457)
Analysis: Since, NPV is negative, project should not be accepted.
Problem No. 39] Electrofast Ltd. is a manufacturing organization. It is- manufacturing electronic
equipments in which a Component-X is used, which is purchased from a local supplier at a cost of ` 40
each. In order to bring down the cost and improve its competitiveness, the company has a proposal to
install a machine for the manufacture of Component-X. It has the following two options:
Option-1 Installation of semi-automatic machine involving annual fixed expenses of ` 22 lakhs and a
variable cost of ` 18 per component manufactured.
Option-2 Installation of automatic machine involving an annual fixed cost of ` 40 lakhs and a variable cost
of` 15 per component manufactured.
You are required to -
(i) Find the annual requirement of Component-X to justify a switch over from purchase of components to
manufacture of the same by installing (i) semi-automatic machine and (ii) automatic machine.
(ii) If the annual requirements of the Component-X is 8,00,000 units, which machine would you advise the
company to install ?
[CS (Professional) - Dec 2007] (10 Marks)
Ans.: (i) Statement showing comparative output required:
Particulars Semi-automatic machine Automatic machine
Purchase price of component (i.e. buying cost) 40 40
(-) Variable cost per component (i.e. making cost) 18 15
Saving in cost if we make machine 22 25

Computation of components required to be produced to justify the installation of the machine = Fixed
cost/Saving
Semi-automatic machine = 22,00,000/22 = 1,00,000 units
Automatic machine = 40,00,000/25 = 1,60,000 units
Selection of machine when annual requirement is 8,00,000 units:
Particulars Semi-automatic Automatic
machine machine
Variable cost (8,00,000 X 18); (8,00,000 X 15) 1,44,00,000 1,20,00,000
Fixed cost 22,00,000 40,00,000
Total cost 1,66,00,000 1,60,00,000
Analysis: Total cost in case of automatic machine is less, hence it will be beneficial to install automatic
machine.
Problem No. 40] An iron ore company is considering investing in a new processing facility. The company
extracts ore from an open pit mine. During a year, 1,00,000 tons of ore is extracted. If the output from
the extraction process is sold immediately upon removal of dirt, rocks and other impurities, a price of `
1,000 per ton of ore can be obtained. The company has estimated that its extraction costs amount to 70%
of the net realizable value of the ore.
As an alternative to selling all the ore at ` 1,000 per ton, it is possible to process further 25% of the output.
The additional cash cost of further processing would be ` 100 per ton. The processed ore would yield 80%
final output and can be sold at ` 1,350 per ton.
For additional processing the company would have to install equipments costing ` 100 lakhs. The
equipment is expected to have a useful life of 5 years with no salvage value. The company follows the
straight line method of depreciation. Additional working capital requirement is estimated at ` 10 lakhs.
The company’s cut-off rate for such investments is 15%. Assume corporate tax rate 30% (including
surcharge and education cess).
Should the company install the equipment for further processing of the iron ore?
[CS (Professional) - June 2009 & Dec 2012] (20 Marks)
Ans.: Calculation of initial investment:
Equipment cost 1,00,00,000
Additional working capital requirement 10,00,000
Net initial investment 1,10,00,000
Processed final output = 1,00,000 tons X 25% X 80% = 20,000 Additional price per unit = 1,350 - 1,000 =
350
Calculation of cash inflow:
Incremental revenue (20,000 X 350) 70,00,000
(-) Cost of processing (25,000 X 100) (25,00,000)
(-) Depreciation on equipment (1,00,00,000/5) (20,00,000)
EBIT 25,00,000
(-) Tax @30% (7,50,000)
PAT 17,50,000
(+) Depreciation 20,00,000
CFAT 37,50,000
Calculation of NPV:
Year CFAT PV Factor 15% PV
1 to 5 37,50,000 3.352 1,25,70,000
5 *10,00,000 0.497 4,97,000
Total present value 1,30,67,000
(-) Ihitial investment (1,10,00,000)
Net Present Value (NPV) 20,67,000
Analysis: Since, NPV is positive it is advised to invest in new processing facility.
Problem No. 41 ] The management of Techno Craft Ltd. is evaluating the following data of a capital project:

Annual cost saving (` ) Project “GEE” 80,000

Useful life (Years) 5


Internal rate of return (%) 12
Profitability index (PI) 1.270457697
NPV ?
Cost of capital p
Cost of project ?
Payback ?
Salvage value 0
Find the missing values considering the following table of discount factors only:
Discount Factor 13% 12% 9% 6% 3%
1 Years 0.885 0.893 0.917 0.943 0.971
2 Years 0.783 0.797 0.842 0.890 0.943
3 Years 0.693 0.712 0.772 0.840 0.915
4 Years 0.613 0.636 0.708 0.792 0.888
5 Years 0.543 0.567 0.650 0.747 0.863
3.517 3.605 3.889 4.212 4.580
[CS (Professional) - June 2009] (10 Marks)
Ans.: Calculation of Cost of project:
Annual cost saving = Cash inflow = ` 80,000
Useful life = 5 years
IRR = 12%
At 12% IRR, total present value of cash inflow is equal to initial cash outlay.
Total present value of cash inflow @12% for 5 years is 3.605 = 80,000 X 3.605 — 2,88,400 Thus, Project
cost = ` 2,88,400.
Calculation of payback period:
Year CFAT Cumulative
CFAT
1 80,000 80,000
2 80,000 1,60,000
3 80,000 2,40,000
4 80,000 3,20,000

Payback period = 3 years + 2,88,400 - 2,40,000/80000 X 12


= 3 years & 7.26 months Calculation of cost of capital:
Profitability Index = Total Present Value/Initial Investment
1.270457697 = Total Present Value/2,88,400
Total Present Value = 3,66,400
Year Cash Flow PV Factor PV
1 to 5 80,000 X 3,66,400
80,000.x = 3,66,400 x = 4.58
Looking at present value table, discount factor for 5 years is 4.58.
Hence, Cost of capital = 3%
Net Present Value = 3,66,400 - 2,88,400 = 78,000
Problem No. 42] A company is considering three methods of attracting customers to expand its business
by undertaking - (A) advertising campaign; (B) display of neon signs; and (C) direct delivery service. The
initial outlay for each alternative is as under:
A ` 1,00,000
B 1 1,50,000
C ` 1,50,000
If A is carried out, but not B, it has an NPV of ` 1,25,000. If B is done, but not A, B has an NPV of 145,000.
However, if both are done, then NPV is ` 2,00,000. The NPV of the delivery system C is ` 90,000. Its NPV is
not dependent on whether A or B is adopted and the NPV of A or B does not depend on whether C is
adopted.
Which of the investments should be made by the company if
(i) Firm has no budget constraint; and
(ii) The budgeted amount is only ` 2,50,000?
[CS (Professional) - Dec 2010] (8 Marks)
Ans.: Calculation of expected NPVs:

Alternatives Initial Outlay (`) Expected NPV (` )


A 1,00,000 1,25,000
B 1,50,000 45,000
C 1,50,000 90,000
A+B 2,50,000 2,00,000
A+C 2,50,000 2,15,000
B+C 3,00,000 1,35,000
A+B+C 4,00,000 2,90,000
(i) No Budget Constraint: The firm should adopt all the three modes of attracting customers. Its outlay in
this case would be ` 4,00,000 and the expected NVP would be ` 2,90,000.
(ii) Budget Constraint: If the budget constraints are limited to ` 2,50,000 then the firm should adopt mode
A + C only. In this case, the outlay would be ` 2,50,000 and the total NPV would be ` 2,15,000.
Problem No. 43] Modern Electronics wants to take up a new project of the manufacture if an electronic
device which has good market. Further details are given below:
Cost of the project as estimated (` in lakhs)
Land 2.00 (will be incurred at the beginning of year 1)
Building 3.00 (will be incurred at the end of year 1)
Machinery 10.00 (will be incurred at the end of year 2)
Working capital 5.00 (will be incurred at the beginning of year 3)
20.00
The project will go into production from the beginning of year 3 and will be operational for a period of 5
years. The annual working results are estimated as follows:

(` in lakhs)
Sales 20.00
Less: Variable cost 8.00
Fixed cost (excluding depreciation) 4.00
Depreciation of Assets 2.00

At the end of the operational period, it is expected that the fixed assets can be sold for ` 5 lakhs (without
any profit)
Cost of capital of the firm is 10%. Applicable tax rate is 50%.
You are required to evaluate the proposal, by working out the net present value and advise the firm.
[CS (Professional) - Dec 2013] (10 Marks)
Ans.: Calculation of CFAT:
Particulars ` in Lakhs
Sales 20.00
(-) Variable cost (8.00)
(-) Fixed cost (4.00)
(-) Depreciation on machinery (2.00)
Profit before tax 6.00
(-) Tax @ 5096 (3.00)
PAT 3.00
(+) Depreciation 2.00
CFAT 5.00
` in Lakhs
Year Cash Flow PV Factor 1096 PV
3 to 7 5.00 3.133 15.665
7 5.00 0.513 2.565
7 5.00 0.513 2.565
Total present value of cash inflow 20.795
0 2.00 1,000 2,000
1 3.00 0.909 2.727
2 10.00 0.826 8.260
2 5.00 0.826 4.130
Total present value of cash outflow 17.117
Net present value (NPV) 3.678
* Salvage value
** Working capital recovered
Analysis: NPV is positive, project can be accepted.
Problem No. 44] Snowman Enterprises is considering the possibility of manufacturing a particular
component which at present is being bought from outside.
The manufacture of the component would call for an investment of ^ 7,50,000 in a new machine besides
an additional investment of ` 50,000 in working capital. The life of the machine would be 10 years with a
salvage value of ` 50,000. The estimated savings (before tax) would be ` 1,80,000 per annum. The income-
tax rate is 50%. The company’s required rate of return is 10%. Depreciation is provided on straight line
basis.
Suggest the firm whether this investment be made or not. Show your workings.
[CS (Professional) - June 2015] (4 Marks)
Ans.: Calculation of initial investment:
Equipment cost 7,50,000
Additional working capital requirement 50,000
Net initial investment 8,00,000
Calculation of CFAT:
Estimated savings 1,80,000
(-) Depreciation (7,50,000 - 50,000/10) (70,000)
EBIT 1,10,000
(-) Tax @ 5096 (55,000)
PAT 55,000
(+) Depreciation 70,000
CFAT 1,25,000
Calculation of NPV:
Year CFAT PV Factor 1096 PV
1 to 10 1,25,000 6.145 7,68,125
10 *50,000 0.386 19,300
10 **50,000 0.386 19,300
Total present value 8,06,725
(-) Initial investment (8,00,000)
Net Present Value (NPV) 6,725
* Salvage value realized.
** Working capital realized.
Analysis: Since, NPV is positive, the firm may purchase machine.
Problem No. 45] Saswat Dairy Ltd. (SDL) is planning to buy dairy plant costing ` 400 lakhs. Milk Board
provides 10% subsidy on the capital cost. It can process milk to produce cheese with the capacity of 1,800
tonnes per annum. The selling price of cheese is taken as ` 50 per kg. The management expects the life of
the plant at 8 years and the depreciation shall be computed using straight line method. However, the
plant can be sold at ` 50 lakhs at the end of its useful life. The utilization of plant is expected as below:
Years 1 2 3 4 to 8
Capacity Utilization 60% 70% 80% 90%
The variable cost constituting primarily of the raw material (milk) is placed at 40% while the fixed expenses
are ` 300 lakhs per annum. The company pays tax @ 35%. The additional working capital required is ` 100
lakhs.
Find the following -
(a) Cash flows of the project from Year 0 to Year 8
(b) NPV of the project
(c) IRR of the project
(d) Payback period and
(e) Should the project be accepted based on NPV and IRR.
[CS (Professional) - June 2015] (16 Marks)
Ans.: Calculation of initial investment: (7 in Lakhs)
Dairy plant cost 400
(-) Subsidy from Milk Board (40)
Additional working capital requirement 100
Net initial investment 460
Selling price per tonne = 50 X 1,000 = 50,000 i.e. 0.5 Lakh
Calculation of CFAT: (` in Lakhs)
Particulars 1 2 3 4 to 8
Capacity Utilization 60% 70% 80% 90%
Production in tonne 1,080 1,260 1,440 1,620
Sales (in lakhs) 540 630 720 810
(-) Variable cost (216) (252) (288) (324)
(-) Fixed cost (300) (300) (300) (300)
(-) Depreciation (360 - 50)/8 (38.75) (38.75) (38.75) (38.75)
EBT (14.75) 39.25 93.25 147.25
(-) Tax @ 35% - *(8.575) (32.6375) (51.5375)
PAT (14.75) 30.675 60.6125 95.7125
(+) Depreciation 38.75. 38.75 38.75 38.75
CFAT 24 69.425 99.3625 134.4625
* Tax for 2nd year = 39.25 X 35% = 13.7375
Tax benefit for loss of year 1 = 14.75 X 35% = 5.1625
Net tax for 2nd year after setting of tax benefit for loss of first year = 13.7375 - 5.1625 = 8.575
Additional cash flow at the end of 8th year from sale after tax = 50 (1 - 0.35) = 32.5 CFAT for 8th year =
134.4625 + 32.5 + 100** = 266.9625. ** Working capital released. Calculation of payback period
Year CFAT Cumulative CFAT
1 24 24
2 69.425 93.425
3 99.3625 192.7875
4 134.4625 327.2500

5 134.4625 461.7125
6 134.4625 596.1750
7 134.4625 730.6375
8 266.9625 997.6000
Payback period = 4 years + 460 - 327.2500/134.4625 x 12
= 4 years & 11.85 months
Calculation of NPV & IRR: (` in lakhs)
PV Factor Present Value
Year CFAT
10% 20% 10% 20%
1 24 0.909 0.833 21.8160 19.9920
2 69.425 0.826 0.694 57.3451 48.1810
3 99.3625 0.751 0.579 74.6212 57.5309
4 134.4625 0.683 0.482 91.8379 64.8109
5 134.4625 0.621 0.402 83.5012 54.0539
6 134.4625 0.565 0.335 75.9713 45.0450
7 134.4625 0.513 0.279 68.9793 37.5150
8 266.9625 0.467 0.233 124.6715 62.2023
Total present value 598.7435 389.3310
(-) Initial investment (460) (460)
Net Present Value (NPV) 138.7435 (70.669)
IRR - 10 + 138.7435/138.7435 + 70.669 x 10
= 10 + 138.7435/ 209.4125 x 10
= 10 + 6.63
= 16.63%
Acceptance of Project based on NPV and IRR:
(1) As per NPV rule, if the NPV of the project is positive project is accepted. If the NPV is negative,
project will be rejected. In given problem cost of capital is not given, so if the at company’s cost of capital
NPV is positive project is accepted.
(2) As per IRR Rule, if the company's cost of capital is less than IRR, project can be accepted. IRR for the
given project is 16.63%, thus if the company’s cost of capital less below 16.63%, project can be accepted
otherwise rejected.
Problem No. 46] ABC Chemicals Ltd. is considering two mutually exclusive proposals. Your advice is sought
for choice between the two options under consideration:
(i) Purchase of petrol truck
(ii) Purchase of a battery powered truck
Year Petrol truck Battery powered truck

Purchase cost (`) 0 1,50,000 2,50,000

Operating cost (`) 1 24,000 12,000

2 34,000 12,000
3 29,000 12,000
4 31,000 12,000
5 - 12,000
Assume an investment incentive of 100% initial depreciation allowance and a 30% incidence of corporate
tax. No depreciation is allowed on subsequent years. Taxes are promptly paid. A return of 10% after tax
as investment incentives is required.
You are required to find out equivalent cost for two options.
[CS (Professional) - Dec 2015] (4 Marks)
Ans.: Calculation of net cash outflow of petrol truck:
Particulars 1 2 3 4 5
Operating cost 24,000 34,000 29,000 31,000 -
Tax benefit on operating cost (7,200) (10,200) (8,700) (9,300) -
Tax benefit on depreciation *(45,000) - - - -
Net cash outflow (28,200) 23,800 20,300 21,700
* As an investment incentive 100% depreciation allowance available at the end of first year. Tax benefit
on depreciation = 1,50,000 X 100% X 30% = 45,000
Calculation of net cash outflow of battery powered truck:
Particulars 1 2 3 4 5
Operating cost 12,000 12,000 12,000 12,000 12,000
Tax benefit on operating cost (3,600) (3,600) (3,600) (3,600) (3,600.)
Tax benefit on depreciation Net cash outflow **(75,000) - - - -
(66,600) 8,400 8,400 8,400 8,400
** Tax benefit on depreciation = 2,50,000 X 100% X 30% = 75,000 Present value of cash outflow of petrol
truck:
Year Cash outflow PV Factor 10% PV
0 1,50,000 1,000 1,50,000
1 (28,200) 0.909 (25,634)
2 23,800 0.826 19,659
3 20,300 0.751 15,245
4 21,700 0.683 14,821
5 - 0.621 1,74,091
Equivalent PV of annual cash outflow = 1,74,091/3,791 = 45,922
Present value of cash outflow of battery powered truck:
Year Cash outflow PV Factor 10% PV
0 2,50,000 1,000 2,50,000
1 (66,600) 0.909 (60,539)
2 8,400 0.826 6,938
3 8,400 0.751 6,308
4 8,400 0.683 5,737
5 8,400 0.621 5,216
2,13,660
Equivalent PV of annual cash outflow = 2,13,660/3.791 = 56,360
Analysis: As equivalent PV of annual cash outflow of petrol truck is less, it advised to go with option of
purchase of petrol truck.
Problem No. 47] A company has two alternatives for selecting a new machine to replace its existing
machine. The cash flows under the two alternatives are as follows:
Machine-A Machine-B

(` in lakh) (` in lakh)
Year 0 cash outflow 25 40
Year 1 cash inflow Nil 10
Year 2 cash inflow 5 14
Year 3 cash inflow 20 16
Year 4 cash inflow 14 17
Year 5 cash inflow 14 15
You are required to appraise the two alternatives using net present value and profitability index methods.
The cost of capital of the company is 15%.
[CS (Professional) - Dec 2015] (8 Marks)
Ans.: Calculation of net present value: (` in lakhs)
Year CFAT PV Factor Present Value

Machine-A Machine-B 15% Machine-A Machine-B


1 0 10 0.870 0 8.700
2 5 14 0.756 3.780 10.584
3 20 16 0.658 13.160 10.528
4 14 17 0.572 8.008 9.724
5 14 15 0.497 6.958 7.455
Total present value 31.906 46.991
(-) Initial investment (25) (40)
Net Present Value (NPV) 6.906 6.991
Analysis: As NPV of Machine-B is higher, it should be selected. Calculation of Profitability Index:
Profitability Index = Total Present Value/Initial Investment
Machine-A = 31.906/25 =1.276
Machine-B = 46.991/40 = 1.175
Analysis: As Profitability Index of Machine-A is higher, it should be selected.
Problem No. 48] Aman Ltd. is producing a single Product-X and presently commanding a market share of
15%. The following cost details are provided:

` `
Sales price 100
Less: Material 40
Labour 20
Overheads 10 70
Contribution 30
Less: Fixed cost 20
Profit 10
The current volume of sale of Product-X is 15,000 units. It has now been estimated that the market share
can be increased up to 25% from next year for 3 years if the following promotional expenses are incurred
in the corresponding previous year:
For Year-1 ` 1,00,000

For Year-2 ` 75,000


For Year-3 ` 50,000
There will be an increase in fixed cost by ` 30,000 when production is increased from present level. The
company wants to achieve 15% return and would apply discounted cash flow technique for evaluation.
You are required to evaluate the impact of above situation on profitability when - (i) Market share is
increased by 25% and
(ii) Market share is increased by 20% [CS (Professional) - Dec 2015] (8 Marks)
Ans.: Calculation of increase in production:
If market share is increased by 25% If market share is increased by 20%
15% - 15,000 units 15% - 15,000 units
25% - ? 20%-?
25 X 15,000/15 = 25,000 20X 15,000/15 = 20,000
Increase on production = 25,000 - 15,000 = 10,000 Increase on production = 20,000 - 15,000 = 5,000
Calculation of incremental cash flow if market share is increased by 25%:
Particulars Year 0 Year 1 Year 2 Year 3
Contribution (10,000 X 30) - 3,00,000 3,00,000 3,00,000
Promotional expenses (1,00,000) (75,000) (50,000) -
Fixed cost - (30,000) (30,000) (30,000)
Net cash inflow (1,00,000) 1,95,000 2,20,000 2,70,000
It is assumed that promotional expense will be incurred at the beginning of each period. (If you incur
promotional expense at the beginning then only benefit will be received throughout the year)
Calculation of present value:
Year Cash inflow PV Factor PV
0 (1,00,000) 1,000 (1,00,000)
1 1,95,000 0.870 1,69,650
2 2,20,000 0.756 1,66,320
3 2,70,000 0.658 1,77,660
4,13,630
Calculation of incremental cash flow if market share is increased by 20%:
Particulars Year 0 Year 1 Year 2 Year 3
Contribution (5,000 X 30) - 1,50,000 1,50,000 1,50,000
Promotional expenses (1,00,000) (75,000) (50,000) -
Fixed cost - (30,000) (30,000) (30,000)
Net cash inflow (1,00,000) 45,000 70,000 1,20,000
Calculation of present value:
Year Cash inflow PV Factor PV
0 (1,00,000) 1,000 (1,00,000)
1 45,000 0.870 39,150
2 70,000 0.756 52,920
3 1,20,000 0.658 78,960
71,030
Analysis: As NP V under both options is positive after taking into account fixed costs as well as promotional
expense, profitability position of the firm will improve if market share is increased in both cases.
Problem No. 49] Calculate the payback period, accounting rate of return, net present value and internal
rate of return for the following investment:
Year Cash flow (` )
0 (30,000)
1 4,000
2 10,000
3 20,000
4 11,000
The rate for discounted cash flow (DCF) calculation is 12%. Accounting profits are the same as cash flow
except that the initial expenditure should be depreciated over 4 years; there is no resale value at year 4.
[CS (Professional) - Dec 2015] (10 Marks)
Ans.: Tax rate is not given in problem, hence Cash Flow = CFAT.
Calculation of payback period:
Year CFAT Cumulative
CFAT
1 4,000 4,000
2 10,000 14,000
3 20,000 34,000
4 11,000 45,000
Payback period = 2 years + 30,000- 14,000/20,00,000 x 12
= 2 years & 9.6 months
Calculation of NPV & IRR:
PV Factor Present Value
Year CFAT
1296 2096 1296 2096
1 4,000 0.893 0.833 3,572 3,332
2 10,000 0.797 0.694 7,970 6,940
3 20,000 0.712 0.579 14,240 11,580
4 11,000 0.636 0.482 6,996 5,302
Total present value 32,778 27,154
(-) Initial investment (30,000) (30,000)
Net Present Value (NPV) 2,778 (2,846)
IRR = 12 + 2,778/2,778 + 2,846 x 8
= 12 +2,778/5,624 x 8
= 12 + 3.95
= 15.95%
Problem No. 50] Aalia Ltd. has two alternative projects under consideration. Project-A requires a capital
outlay of ` 1,60,000 and Project-B needs ` 1,80,000. Both are estimated to provide a cash flow for 5 years;
Project-A: ` 45,000 per year and Project-B: ` 58,000 per year. The cost of capital is 10%. Show which project
is preferable from the view point of net present value?
[CS (Professional) - June 2016] (4 Marks)
Ans.: Calculation of net present value:
CFAT PV Factor Present Value
Year
Project-A Project-B 1096 Project-A Project-B
1 to 5 45,000 58,000 3.791 1,70,595 2,19,878
(-) Initial investment (1,60,000) (1,80,000)
Net Present Value (NPV) 10,595 39,878
Analysis: Since, NPV of Project-B is higher, it should be selected.
CHAPTER
11
CAPITAL BUDGETING - LEASE OR BOY DECISIONS
Question 1] What do you mean by 'lease'?
Ans.: A lease is a contractual arrangement calling for the lessee (user) to pay the lessor (owner) for use of
an asset. Property, buildings, vehicles & business equipments are common assets that are leased.
A lease agreement is a contract between two parties, the lessor and the lessee. The lessor is the legal
owner of the asset. The lessee obtains the right to use the asset in return for regular rental payments. The
lessee also agrees to abide by various conditions regarding their use of the property or equipment.
A famous quote by Donald B. Grant says, “ Why own a cow when the milk is so cheap? All you really need
is milk and not the cow." The concept of Lease is influenced by this quote. We can compare ‘milk’ with the
‘rights to use an asset' and ‘cow' with the ‘asset’ itself. Ultimately, a person who wants to manufacture a
product using machinery can get to use that machinery under a leasing arrangement without owning it.
Question 2] Write a short note on: Finance Lease
Ans.: As per AS-19, a lease is classified as a finance lease if it transfers substantially all the risks and
rewards incident to ownership. Title may or may not eventually be transferred.
Whether a lease is a finance lease depends on the substance of the transaction rather than its form.
Examples of situations which would normally lead to a lease being classified as a finance lease are:
(a) The lease transfers ownership of the asset to the lessee by the end of the lease term.
(b) The lessee has the option to purchase the asset at a price which is expected to be sufficiently lower
than the fair value at the date the option becomes exercisable such that, at the inception of the lease, it
is reasonably certain that the option will be exercised.
(c) The lease term is for the major part of the economic life of the asset even if title is not transferred.
(d) At the inception of the lease the present value of the minimum lease payments amounts to at least
substantially all of the fair value of the leased asset.
(e) The leased asset is of a specialized nature such that only the lessee can use it without major
modifications being made.
Indicators of situations which individually or in combination could also lead to a lease being classified as
a finance lease are:
(1) If the lessee can cancel the lease, the lessor's losses associated with the cancellation are borne by
the lessee.
(2) Gains or losses from the fluctuation in the fair value of the residual fall to the lessee (for example in
the form of a rent rebate equaling most of the sales proceeds at the end of the lease).
(3) The lessee can continue the lease for a secondary period at a rent which is substantially lower than
market rent.
Operating Lease: A lease is classified as an operating lease if it does not transfer substantially all the risks
and rewards incident to ownership. As per AS-19, a lease which is not financial lease may be treated as
operating lease.
Question 3] Distinction Between: Finance Lease & Operating Lease
CS (Professional) - Dec 2007 (5 Marks), Dec 2009 (5 Marks)
Ans.: Following are the main points of distinction between finance lease and operating lease:
Points Finance Lease Operating Lease
Meaning A lease is classified as a finance lease if it transfers A lease is classified as an operating lease if it does
substantially all the risks and rewards incident to not transfer substantially all the risks and rewards
ownership. incident to ownership.
Ownership Title may or may not be transferred at the Ownership of the asset remains with the lessor
beginning of lease but may be transferred for the entire lease period.
subsequently at the end of lease.
Lease Term Lease term is generally more than or equal to Lease term extends to less than 75% of the
estimated economic life of the asset leased. projected useful life of the leased asset.
Cancellation Financial lease may be cancelled only if both Operating lease may be cancelled before the
parties Le. lessor and lessee agrees. expiry date.
Accounting Financial lease is treated like loan. Lessee is Operating lease is treated generally like renting.
Effect treated as owner and hence asset acquired under That means, the lease payments are treated as
finance lease is shown as asset in the balance operating expenses.
sheet of lessee.
Purchase Financial lease allows the lessee to have a In operating lease, the lessee does not have any
Option purchase option. option to buy the asset during the lease period.
Expenses In financial lease, lessee bears insurance, Lessee pays only the monthly lease payment in
maintenance and taxes. operating lease.
Tax Benefit In financial lease, lessee can claim tax benefit on Since operating lease is as good as renting, lease
interest and depreciation both as financial lease payment is considered as expense and thus tax
is treated like a loan. benefit only on lease payment is available.
Running Cost In a financial lease, running cost and
administration expenses are has to be borne by In operating lease, no running or administration
lessee. costs are to be borne by lessee.
Question 4] Write a short note on: Hire Purchase
Ans.: Hire purchase is a contract in which goods are delivered and the price is allowed to be paid in
instalments on the condition that property (ownership) in goods will be passed only on payment of all the
instalments.
In order to facilitate purchases of costly durable goods by the customers, many sellers of such goods offer
to accept the price of goods in instalments along with interest on outstanding balances for different
periods. In a sale, the property in the goods passes from the seller to the buyer as soon as contract is
entered into even though payment for the goods is agreed to be made and received in instalments. In
other words, the buyer becomes the owner of the goods immediately on the signing of the contract of
sale. If subsequently the buyer fails to pay any instalments, the seller can only sue for the balance of price
(of course along with interest thereon) and cannot ask for the return of the goods. The buyer can
legitimately further sell the goods before he has made the full payment. The instalment system, as it is
called, is thus not very satisfactory from the point of view of seller.
Hire purchase is a contract in which goods are delivered and the price is allowed to be paid in instalments
on the condition that property (ownership) in goods will be passed only on payment of all the instalments
Under hire purchase system also the payment is received in instalments, but the hire purchaser does not
become owner of the goods till he has paid the stipulated number of instalment. If the hire purchaser
makes default in payment of even one instalment, the hire vendor (seller) can take back the goods from
the hire purchaser without requiring to refund the amount already received by him because the amount
is considered to be the hire-charges of the goods for the period for which the goods have already been
with the hire purchaser. On the payment of the last instalment alone, the hire purchaser becomes the
owner of the goods.
Question 5] Distinction Between: Lease & Hire Purchase
CS (Professional) - Dec 2007 (5 Marks), Dec 2009 (5 Marks)
CS (Professional) - Dec 2016 (4 Marks)
Ans.: Following are the main points of distinction between lease & hire purchase:
Points a Lease . Hire Purchase
Meaning A lease is a contractual arrangement calling for Hire purchase is a contract in which goods are
the lessee (user) to pay the lessor (owner) for use delivered and the price is allowed to be paid in
of an asset. instalments on the condition that property
(ownership) in goods will be passed only on
payment of all the instalments
Transfer of In leasing it is only in financial lease, the In Hire purchase, the agreement is entered for the
ownership ownership will get transferred. While in operating transfer of ownership after a fixed period.
lease, the ownership is not transferred.
Type of Leasing is bipartite agreement, involving lessor Hire purchase is a tripartite agreement involving
agreement and lessee. the seller, finance company and the hire
purchaser.
Depreciation In finance lease depreciation is claimed by the Depreciation is claimed by the hire purchaser.
lessee whereas in operating lease depreciation
claimed by lessor.
Question 6] Distinguish between Hire-Purchase and Instalment System
Ans.: Following are the main points of distinction between hire-purchases and instalment purchase
system:
Points Hire Purchase System Instalment System
Meaning Hire purchase is a contract in which goods are Instalment system is a contract in which property
delivered and the price is allowed to be paid in (ownership) in goods will be passed immediately,
instalments on the condition that property however price is allowed to be paid in
(ownership) in goods will be passed only on instalments.
payment of all the instalments
Mature of The hire purchase system is an agreement of Instalment purchase system, is an agreement of
contract hiring. outright purchase.
Passing of Under the hire-purchase system the property in In instalment purchase system, the title of goods
Ownership goods or the ownership remains with hire vendor passes to the seller immediately on agreement to
until the last instalment is paid by the purchaser purchase as is usual in the case of sale.
Return of goods Under the hire-purchase system the hirer may Under instalment purchase system the goods are
return the goods without further payment as to not returnable unless there is some default on the
future instalment. part of the seller, the buyer is liable for the price
agreed to or reasonable price if not agreed.
Rights of seller Under the hire-purchase system the seller may The seller can only sue for price if the buyer is in
take possession of the goods if the hirer is in default under instalment purchase system.
default in the payment of instalment.
Rights of Under the hire-purchase system, the buyer The buyer may dispose of and give good title to
disposal cannot hire-out sell, pledge, assign, destroy, any bona fide purchaser under the instalment
damage or transfer goods. purchase system.
Leveraged lease involves lessor, lessee and financier. In leveraged lease, the lessor makes a substantial
borrowing, even up to 80% of the assets purchase price. He provides remaining amount - about 20% or
so - as equity to become the owner. The lessor claims all tax benefits related to the ownership of the
assets. Lenders, generally large financial institutions, provide loans on a non-recourse basis to the lessor.
Their debt is served exclusively out of the lease proceeds. To secure the loan provided by the lenders, the
lessor also agrees to give them a mortgage on the asset. Leveraged lease are called so because the high
non-recourse debt creates a high degree of leverage.
Question 8] Explain the concept of closed and open ended lease.
Ans.: In the close-ended lease, the assets gets transferred to the lessor at the end of lease, the risk of
obsolescence, residual values etc. remain with the lessor being the legal owner of the assets. In the open-
ended lease, the lessee has the option of purchasing the assets at the end of lease period.
Question 9] Write a short note on: Sale and lease back
CS (Professional) - Dec 2015 (5 Marks)
Ans.: A sale and leaseback constitutes an arrangement where the seller of an asset leases back the same
asset from the purchaser.
The lease arrangement is made immediately after the sale of the asset with the amount of the payments
and the time period specified. Essentially, the seller of the asset becomes the lessee and the purchaser
becomes the lessor in this arrangement.
A leaseback arrangement is useful when companies need to untie the cash invested in an asset for other
investments, but the asset is still needed in order to operate. Leaseback deals can also provide the seller
with additional tax deductions. The lessor benefits in that they will receive stable payments for a specified
period of time.
Question 10] Distinguish between: 'Finance lease' and 'sale and lease back'
CS (Professional) - Dec 2015 (5 Marks)
Ans.: Following are main points of distinction between finance lease and sale & lease back:
Points Finance Lease Sale and lease back
Meaning A lease is classified as a finance lease if it transfers A sale and leaseback constitutes an arrangement
substantially all the risks and rewards incident to where the seller of an asset leases back the same
ownership. asset from the purchaser.
Real owner Real owner of asset become lessor after leasing Real owner who is bought the asset once sale it
the asset. and by taking on lease again become lessee of the
asset.
Lease rentals Real owner receive the lease rentals. Real owner pays the lease rentals.
Lease term In finance lease, the lease term is for the major In sale and leaseback, the lease term may be for
part of the economic life of the asset even if title
is not transferred. part of the economic life of the asset.
Question 11] For lessor, lease decision is akin to a capital budgeting exercise. Examine the statement
and explain its implications.
CS (Professional) - Dec 2005 (5 Marks)
Ans.: Lease or buy decision involves applying capital budgeting principles to determine if leasing as asset
is a better option than buying it.
Leasing in a contractual arrangement in which the lessee obtains an asset from the lessor against periodic
payments of lease rentals. It may typically also involve an option to transfer the ownership of the asset to
the lessee at the end of the lease.
Buying the asset involves purchase of the asset with company’s own funds or arranging a loan to finance
the purchase.
In finding out whether leasing is better than buying, we need to find out the periodic cash flows under
both the options and discount them using the after-tax cost of debt to see where does the present value
of the cost of leasing stands as compared to the present value of the cost of buying. The alternative with
lower present value of cash outflows is selected.
Problem No. 1] Welsh Ltd. is faced with a decision to purchase or acquire on lease a mini car. The cost of
the mini car is ` 1,26,965. It has a life of 5 years. The mini car can be obtained on lease by paying equal
lease rentals annually. Such lease rentals are payable at the end of the year. The leasing company desires
a return of 10% on the gross value of the asset.
Welsh Ltd. can also obtain 100% finance from its regular banking channel. The rate of interest will be 15%
p.a. and the loan will be paid in 5 annual equal instalments, payable at the end of the year, inclusive of
interest.
The effective tax rate of the company is 40%. For the purpose of taxation it is to be assumed that the asset
will be written off over a period of 5 years on a straight line basis:
(a) Advise Welsh Ltd. about the method of acquiring the car.
(b) What should be the annual lease rental to be charged by the leasing company to match the loan
option?
For your exercise use the following discount factors:
Discount rate Year 1 Year 2 Year 3 Year 4 Year 5
10% 0.91 0.83 0.75 0.68 0.62
15% 0.87 0.76 0.66 0.57 0.50
9% 0.92 0.84 0.77 0.71 0.65
Ans.: Evaluation of loan option:
Loan Amount = Investment cost/Annuity factor of 15% for 5 years = 1,26,965/3.35 = 37,900
Calculation of principal & interest:
Year Instalment Principal Interest @15% Balance
0 - - - 1,26,965
1 37,900 18,855 19,045 1,08,110
2 37,900 21,683 16,217 86,427
3 37,900 24,936 12,964 61,491
4 37,900 28,676 9,224 32,815
5 37,900 32,815 5,085 -
PV Factor = 15(1 - 0.4) = 9%
Calculation of net present value of cash outflow:
Year Principal Interest Depreciation Tax benefit [(2) + (3) Net cash PV Factor 9% PV
x 40%] outflow
(1) (2) (3) (4) (5) (6) (7)
1 18,855 19,045 25,393 17,775 20,125 0.92 18,515
2 21,683 16,217 25,393 16,644 21,256 0.84 17,855
3 24,936 12,964 25,393 15,343 22,557 0.77 17,369
4 28,676 9,224 25,393 13,847 24,053 0.71 17,078
5 32,815 5,085 25,393 12,191 25,709 0.65 16,711
Total present value of cash outflow 87,528

Evaluation of lease option :


Lease Rental = Cost of asset/Annuity factor of 1096 for 5 years = 1,26,965/3.79 = 33,500

Year Lease Rented Tax benefit Net cash outflow PV Factor 9% PV


1 to 5 33,500 13,400 20,100 3.89 78,189
Analysis: Present value of cash outflow is less for leasing option and hence company is advised to take car
availing lease option.
Calculation of annual lease rental to be charged by leasing company to match the loan option:
Let the lease rental be 'x’.
Year Lease Rental Tax benefit Net cash outflow PV Factor 9% PV
1 to 5 * 0.4* 0.6* 3.89 87,528
0.6x x 3.89 = 87,528 2.334 x = 87,528
x = 37,501
Problem No. 2] Solve Problem No. 1 assuming that ‘Loan Instalments’ and ‘Lease Rentals’ both are payable
at the beginning of the year.
Ans.:
Evaluation of loan option:
Loan Amount = Investment cost/1 + Annuity factor of 1596 for 4 years = 1,26,965/3.86 = 32,892
Calculation of principal & interest:
Year Instalment Principal Interest @1596 Balance
0 32,892 32,892 - 94,073
1 32,892 18,781 14,111 75,292
2 32,892 21,598 11,294 53,694
3 32,892 24,838 8,054 28,856
4 32,892 28,856 4,036 -
PV Factor = 15(1 -0.4) = 996
Calculation of net present value of cash outflow:
Year Principal Interest Depreciation Tax benefit [(2) + (3) Net cash PV Factor 996 PV
x 40%] outflow
(1) (2) (3) (4) (5) (6) (7)
0 32,892 - - - 32,892 1.00 32,892
1 18,781 14,111 25,393 15,802 17,090 0.92 15,723
2 21,598 11,294 25,393 14,675 18,217 0.84 15,302
3 24,838 8,054 25,393 13,379 19,513 0.77 15,025
4 28,856 4,036 25,393 11,772 21,120 0.71 14,995
5 - - 25,393 10,157 (10,157) 0.65 (6,602)
Total present value of cash outflow 87,335
Evaluation of lease option :
Lease Rental = Cost of asset/1 + Annuity factor of 10% for 4 years = 1 26 965/4.17 = 30,447

Year Lease Rental Tax benefit Net cash outflow PV Factor 9% PV


0 30,447 - 30,447 LOO 30,447
1 30,447 12,179 18,268 0.92 16,807
2 30,447 12,179 18,268 0.84 15,345
3 30,447 12,179 18,268 0.77 14,066
4 30,447 12,179 18,268 0.71 12,970
5 - 12,179 (12,179) 0.65 (7,916)
Total present value of cash outflow 81,719
Analysis: Present value of cash outflow is less for leasing option and hence company is advised to take car
availing lease option.
Calculation of annual lease rental to be charged by leasing company to match the loan option:
Let the lease rental be 'x’.
Year Lease Rental Tax benefit Net cash outflow PV Factor 9% PV
0 * - * 1.00 *
1 * 0.4* 0.6* 0.92 0.552*
2 * 0.4* 0.6* 0.84 0.504*
3 * 0.4* 0.6* 0.77 0.462*
4 * 0.4* 0.6* ' 0.71 0.426*
5 - 0.4* (0.4*) 0.65 (0.26*)
Total present value of cash outflow 87,335
x + 0.552* + 0.504* + 0.462* + 0.426* - 0.26* = 87,335 2.684* = 87,335 * = 32,539
Problem No. 3] GST Ltd. is considering to acquire a machine costing ` 20,00,000. Life of the machine is 10
years with no residual value. It has following two options:
(1) To purchase the machine by taking loan repayable in 10 year end equal instalments. Rate of interest
is 16% p.a. Machine will be depreciated by straight line method (SLM).
(2) To take a machine on lease from leasing company. Lessor requires the assets to be completely
amortized over its useful life and the assets will yield him return of 10%. Such lease rentals are payable at
the end of the year.
Applicable tax rate to GST Ltd. is 50%. As a financial consultant advise the company whether it should buy
or lease the machine.
Ans.: Evaluation of loan option:
Loan Amount = Investment cost/Annuity factor of 16% for 10 years = 20,00,000/4,833 = 4,13,822

Calculation
of principal
& interest :
Year Instalment Principal Interest @16% Balance
0 - - - 20,00,000
1 4,13,822 93,822 3,20,000 19,06,178
2 4,13,822 1,08,834 3,04,988 17,97,344
3 4,13,822 1,26,247 2,87,575 16,71,097
4 4,13,822 1,46,446 2,67,376 15,24,651
5 4,13,822 1,69,878 2,43,944 13,54,773
6 4,13,822 1,97,058 2,16,764 11,57,715
7 4,13,822 2,28,588 1,85,234 9,29,127
8 4,13,822 2,65,162 1,48,660 6,63,965
9 4,13,822 3,07,588 1,06,234 3,56,377
10 4,13,822 3,56,377 57,445 -
PV Factor = 16(1 -0.5) = 8%
Calculation of net present value of cash outflow:
Year Principal Interest Depreciation Tax benefit PV Factor 8% PV
Net cash
outflow
1 93,822 3,20,000 2,00,000 2,60,000 1,53,822 0.926 1,42,439
2 1,08,834 3,04,988 2,00,000 2,52,494 1,61,328 0.857 1,38,258
3 1,26,247 2,87,575 2,00,000 2,43,788 1,70,034 0.794 1,35,007
4 1,46,446 2,67,376 2,00,000 2,33,688 1,80,134 0.735 1,32,398
5 1,69,878 2,43,944 2,00,000 2,21,972 1,91,850 0.681 1,30,650
6 1,97,058 2,16,764 2,00,000 2,08,382 2,05,440 0.630 1,29,427
7 2,28,588 1,85,234 2,00,000 1,92,617 2,21,205 0.583 1,28,963
8 2,65,162 1,48,660 2,00,000 1,74,330 2,39,492 0.540 1,29,326
9 3,07,588 1,06,234 2,00,000 1,53,117 2,60,705 0.500 1,30,353
10 3,56,377 57,445 2,00,000 1,28,723 2,85,099 0.463 1,32,001
Total present value of cash outflow 13,28,822
Evaluation of lease option:
Lease Rental = Cost of asset/Annuity factor of 10% for 10 years = 20,00,000/6.145 = 3,25,468

Year Lease Rental Tax benefit Net cash outflow PV Factor 8% PV


1 to 10 3,25,468 1,62,734 1,62,734 6.71 10,91,945
Analysis: Present value of cash outflow is less for leasing option and hence company is advised to take
machine availing lease option.
Problem No. 4] Solve Problem No. 3 assuming that ‘Loan Instalments’ and ‘Lease Rentals’ both are payable
at the beginning of the year.
Ans.:
Evaluation of loan option:
Loan Amount = Investment cost/1 + Annuity factor of 16% for 9 years = 20,00,000/5.607 = 3,56,697

Calculation
of principal
& interest :
Year Instalment Principal Interest @ 16% Balance
0 3,56,697 3,56,697 - 16,43,303
1 3,56,697 93,769 2,62,928 15,49,534
2 3,56,697 1,08,772 2,47,925 14,40,762
3 3,56,697 1,26,175 2,30,522 13,14,587
4 3,56,697 1,46,363 2,10,334 11,68,224
5 3,56,697 1,69,781 1,86,916 9,98,443
6 3,56,697 1,96,946 1,59,751 8,01,497
7 3,56,697 2,28,457 1,28,240 5,73,040
8 3,56,697 2,65,011 91,686 3,08,029
9 3,56,697 3,08,029 48,668 -
10 - -
PV Factor =16(1- 0.5) = 8%
Calculation of net present value of cash outflow:
Year Principal Interest Depreciation Tax benefit Net cash PV
outflow PV Factor 8%
0 3,56,697 - - ; 3,56,697 1,000 3,56,697
1 93,769 2,62,928 2,00,000 2,31,464 1,25,233 0.926 1,15,966
2 1,08,772 2,47,925 2,00,000 2,23,963 1,32,734 0.857 1,13,753
3 1,26,175 2,30,522 2,00,000 2,15,261 1,41,436 0.794 1,12,300
4 1,46,363 2,10,334 2,00,000 2,05,167 1,51,530 0.735 1,11,375
5 1,69,781 1,86,916 2,00,000 1,93,458 1,63,239 0.681 1,11,166
6 1,96,946 1,59,751 2,00,000 1,79,876 1,76,821 0.630 1,11,397
7 2,28,457 1,28,240 2,00,000 1,64,120 1,92,577 0.583 1,12,272
8 2,65,011 91,686 2,00,000 1,45,843 2,10,854 0.540 1,13,861
9 3,08,029 48,668 2,00,000 1,24,334 2,32,363 0.500 1,16,182
10 - - 2,00,000 1,00,000 (1,00,000) 0.463 (46,300)
Total present value of cash outflow 13,28,669
Evaluation of lease option:
Lease Rental = Cost of asset/1 + Annuity factor of 10% for 9 years = 20,00,000/6.759 = 2 95 902
= 6.759
Year Lease Rental Tax benefit Net cash outflow PV Factor 8% PV
0 2,95,902 - 2,95,902 1,000 2,95,902
1 to 9 2,95,902 1,47,951 1,47,951 6.247 9,24,250
10 - (1,47,951) 0.463 (68,501)
Total present value of cash outflow 11,51,651
Analysis: Present value of cash outflow is less for leasing option and hence company is advised to take
machine availing lease option.
Problem No. 5] ABC Leasing Ltd. has been approached by a client to write a five years lease on an asset
costing ` 10,00,000 and having estimated salvage value of ` 1,00,000 thereafter. The company has after
tax required rate of return of 10% and its tax rate is 50%. It provides depreciation @ 33.3333% on written
down value of the asset.
What lease rental will provide the company its after tax required rate of return?
Ans.: In order to find out the annual lease rent, the cash flows from the asset must be evaluated as follows:
Year Depreciation Tax benefit Cash flow PV Factor 1096 PV
1 3,33,333 1,66,667 1,66,666 0.909 1,51,499
2 2,22,222 1,11,111 1,11,111 0.826 91,778
3 1,48,148 74,074 74,074 0.751 55,630
4 98,766 49,383 49,383 0.683 33,729
5 65,844 32,922 32,922 0.621 20,445
5 *1,00,000 0.621 62,100
Present value of inflows 4,15,181
Outflow 10,00,000
Net Present Value (NPV) 5,84,819
* Salvage Value
The firm therefore, should have total recovery of ` 5,84,820 through the lease rentals.
The annual lease rental after tax may be calculated as follows:
Lease rental (after tax) = Total recovery required/PV Factor 1096 for 5 years = 5,84,819/3.791 = 1,54,265
Lease rental (before tax) = 1,54,265/0.5 = 3,08,530
Therefore, the firm should charge a lease rental of ` 3,08,530 in order to earn a required rate of return of
10% after tax.
Problem No. 6] A Company is planning to acquire a machine costing ` 5,00,000. Effective life of the
machine is 5 years. The Company is considering two options. One is to purchase the machine by lease and
the other is to borrow ` 5,00,000 from its bankers at 10% interest p.a. The principal amount of loan will
be paid in 5 equal instalments to be paid annually. The machine will be sold at ` 50,000 at the end of 5th
year.
Following further information is given:
(a) Principal, interest, lease rentals are payable on the last day of each year.
(b) The machine will be fully depreciated over its affective life.
(c) Tax rate is 30% and after tax cost of capital is 8%.
Ans.:
Calculation of net present value of cash outflow under loan option:
Year Principal Interest Depreciation Tax benefit Net cash PV Factor 896 PV
outflow
1 1,00,000 50,000 1,00,000 45,000 1,05,000 0.926 97,230
2 1,00,000 40,000 1,00,000 42,000 98,000 0.857 83,986
3 1,00,000 30,000 1,00,000 39,000 91,000 0.794 72,254
4 1,00,000 20,000 1,00,000 36,000 84,000 0.735 61,740
5 1,00,000 10,000 1,00,000 33,000 77,000 0.681 52,437
5 - - - - *(35,000) 0.681 23,835

Total present value of cash outflow 3,43,812


Cost of asset 5,00,000
(-) Depreciation for 5 years (5,00,000)
Book value at the end of 5th year 0
Profit/Loss 50,000
Sales price 50,000
(-) Tax on profit (50,000 x 30%) (15,000)
Net cash flow on sale *35,000

The firm should have total recovery of ` 3,43,812 through the lease rentals.
The annual lease rental after tax may be calculated as follows:
Lease rental (after tax) = Total recovery required/PV Factor 8% for 5 years = 3,43,812/3.993 = 86,104
Lease rental (before tax) = 86,104/0.7 = 1,23,006
Therefore, the firm should charge a lease rental of ` 1,23,006 in order to earn a required rate of return of
8% after tax.
Problem No. 7] Agrani Ltd. is in the business of manufacturing bearings. Some more product lines are
being planned to be added to the existing system. The machinery required may be bought or may be taken
on lease. The cost of machine is ` 40,00,000 having a useful life of 5 years with the salvage value of `
8,00,000. The full purchase value of machine can be financed by 20% loan repayable in 5 equal instalments
falling due at the end of each year. -
Alternatively, the machine can be procured on a 5 years lease, year-end lease rentals being ` 12,00,000
p.a. The Company follows the WDV method of depreciation at the rate of 25%.
Company’s tax rate is 35%.
(i) Advise the company which option it should choose - lease or borrow.
(ii) Assess the proposal from the lessor’s point of view examining whether leasing the machine is
financially viable at 14% cost of capital.
Ans.: Evaluation of loan option:
Loan Amount = Investment cost/Annuity factor of 20% for 5 years = 40,00,000/2.991 = 13,37,345
Calculation of principal & interest:
Year Instalment Principal Interest @ 20% Balance
0 - - - 40,00,000
1 13,37,345 5,37,345 8,00,000 34,62,655
2 13,37,345 6,44,814 6,92,531 28,17,841
3 13,37,345 7,73,777 5,63,568 20,44,064
4 13,37,345 9,28,532 4,08,813 11,15,532
5 13,37,345 11,15,532 2,21,813 -
PV Factor = 20 (1 - 0.35) = 13%
Calculation of net present value of cash outflow:
Year Principal Interest Depreciation Tax benefit Net cash PV Factor PV
outflow 1396
1 5,37,345 8,00,000 10,00,000 6,30,000 7,07,345 0.885 6,26,000
2 6,44,814 6,92,531 7,50,000 5,04,886 8,32,459 0.783 6,51,815
3 7,73,777 . -5,63,568 5,62,500 3,94,124 9,43,221 0.693 6,53,652
4 9,28,532 4,08,813 4,21,875 2,90,741 10,46,604 0.613 6,41,568
5 11,15,532 2,21,813 3,16,406 1,88,377 11,48,968 0.543 6,23,890
5 - -. - *(8,52,227) 0.543 (4,62,759)
Total present value of cash outflow 27,34,166

Cost of asset 40,00,000


(-) Depreciation for 5 years (30,50,781)
Book value at the end of 5th year 9,49,219
(-) Loss (1,49,219)
Sales price 8,00,000
(+) Tax benefit on loss (1,49,219 X 35%) 52,227
Net cash flow on sale *8,52,227
Evaluation of lease option:
Year Lease Rental Tax benefit * Net cash outflow PV Factor 13% PV
1 to 5 12,00,000 4,20,000 7,80,000 3.517 27,43,260
Analysis: Present value of cash outflow is less for loan option and hence company is advised to buy
machine availing loan option.
Assessment of the proposal from the lessor’s point of view:
Calculation of lessor’s net cash inflow:
Particulars Year 1 Year 2 Year 3 Year 4 Year 5
Lease Rentals 12,00,000 12,00,000 12,00,000 12,00,000 12,00,000
(-) Depreciation (10,00,000) (7,50,000) (5,62,500) (4,21,875) (3,16,406)
EBT 2,00,000 4,50,000 6,37,500 7,78,125 8,83,594
(-) Tax (70,000) (1,57,500) (2,23,125) (2,72,344) (3,09,258)
PAT 1,30,000 2,92,500 4,14,375 5,05,781 5,74,336
(+) Depreciation 10,00,000 7,50,000 5,62,500 4,21,875 3,16,406
CFAT 11,30,000 10,42,500 9,76,875 9,27,656 8,90,742

Year Cash inflow PV Factor 14% PV


1 11,30,000 0.877 9,91,010
2 10,42,500 0.769 8,01,683
3 9,76,875 0.675 6,59,391
4 9,27,656 0.592 5,49,172
5 8,90,742 0.519 4,62,295
5 *8,52,227 0.519 4,42,306
Total present value of cash inflow 39,05,857
(-) Initial investment (40,00,000)
Net present value (94,143)
* Net cash inflow on sale at the end of 5th year.
Analysis: Lease is not financially viable; hence not recommended.
Problem No. 8] Hindustan Machine Tools Ltd. is considering the acquisition equipment to set up in its
factory for ` 12,00,000. The useful life of equipment is 8 years. It can be financed with 8 year term loan at
14% interest, repayable in equal instalments of ` 2,58,676 per year or by an equivalent amount of lease
rent per year. In both cases, payments are due at the end of the year.
You are required to:
(A) Compute net advantage of leasing (NAL) to the lessee assuming:
(i) The company follows WDV method of charging depreciation at 25%.
(ii) The corporate tax is 35%.
(iii) Post tax marginal cost of capital (Ko) is 12%.
(iv) The company has several assets in the asset block of 25%.
(B) Also determine the break even lease rentals (BELR) for the lessee.
Ans.:
Computation of net advantage of leasing to the lessee:
Particulars `
Benefits of lease:
Saving in cost of equipment 12,00,000
Tax benefit on lease rentals (Note 1) 4,49,786
16,49,786
Cost of lease:
PV of lease rental (Note 2) 12,00,000
PV of tax benefit of depreciation forgone (Note 3) 2,72,333
PV of tax benefit of interest forgone (Note 4) 2,08,381
16,80,714
Net advantage of leasing (30,928)
Analysis: Lease is not financial viable.
Working Notes:
Note 1:
Tax benefit on lease rentals = Lease rentals x Tax rate x Annuity factor of 12% for 8 years
= 2,58,676 x 35% x 4.968 = 4,49,786
Note 2:
PV of lease rental = Lease rentals x Annuity factor of 14% for 8 years = 2,58,676 x 4.639 = 11,99,998 say
12,00,000
Note 3:
Year Depreciation Tax benefit PV Factor 12% PV
1 3,00,000 1,05,000 0.893 93,765
2 2,25,000 78,750 0.797 62,764
3 1,68,750 59,062 0.712 42,052
4 1,26,562 44,297 0.636 28,173
5 94,922 33,223 0.567 18,837
6 71,191 24,917 0.507 12,633
7 53,393 18,688 0.452 8,447
8 40,045 14,016 0.404 5,662
PV of tax benefit of depreciation forgone 2,72,333
Note 4 :
Year Interest Tax benefit PV Factor 1296 PV
1 1,68,000 58,800 0.893 52,508
2 1,55,305 54,357 0.797 43,322
3 1,40,833 49,292 0.712 35,096
4 1,24,335 43,517 0.636 27,677
5 1,05,528 36,935 0.567 20,942
6 84,087 29,430 0.507 14,921
7 59,644 20,875 0.452 9,436
8 31,676 11,087 0.404 4,479
PV of tax benefit of interest forgone 2,08,381
Calculation of break-even lease rentals (BELR) for the lessee:

Particulars `
Benefits of lease:
Saving in cost of equipment 12,00,000
Tax benefit on lease rentals (Note 5) 1.62365*
Cost of lease:
PV of lease rental (Note 6) 4.639*
PV of tax benefit of depreciation forgone (Note 3) 2,72,333
PV of tax benefit of interest forgone (Note 4) 2,08,381
12,00,000 + 1.62365x = 4.639x + 2,72,333 + 2,08,381
7,19,286 = 3.01535x
x = 2,38,541 Note 5:
Tax benefit on lease rentals = Lease rentals * Tax rate x Annuity factor of 14% for 8 years
= x x 35% x 4.639
= 1.62365x
Note 6:
PV of lease rental = Lease rentals x Annuity factor of 14% for 8 years
= x x 4.639
= 4.639x
Problem No. 9] NS Ltd. recently leased assets worth ` 2,500 lakhs form the XYZ Leasing Ltd. The following
facts are available:
(1) Lease period - 9 years, of which the first 6 years constitute the lease term.
(2) Annual lease rates:
First 6 years - ` 360/` 1,000 Next 3 years-` 15/` 1,000
(3) Incremental borrowing rate for NS Ltd. - 22%.
Assuming 14 years as the average economic life of the equipment, is the lease a finance lease or an
operating lease.
Assuming further (i) physical life of 14 years, (ii) technological life of 9 years and (iii) product market life
of 11 years, how will you classify the lease?
Ans.: A lease is finance lease if one of the following conditions is satisfied:
(1) The lease term exceeds 75% of the useful life of the asset (ie. minimum of physical life, technological
life or product market life)
(2) The PV of lease payment exceeds 90% of the fair market value (FMV) of the asset (the discount rate
being incremental borrowing rate for lessee and cost of capital for lessor).
Term of lease is 9/14 ie. 64%
Calculation of PV of lease payments:
(` in lakhs)
Year Lease Payments PV Factor 22% PV
1 to 6 900 3.167 2,850.30
7 to 9 37.5 0.619 23.21
2,873.51

Year PV Factor 22%


1 0.820
2 0.672
3 0.550
4 0.452
5 0.370
6 0.303
7 0.249
8 0.204
9 0.167
3.168
0.619
Since, PV of lease payments exceeds cost of asset, it is finance lease. Problem No. 10] The following data
are furnished by Hi-Fi Leasing Ltd.:
Investment cost : ` 500 Lakhs
Primary lease term : 5 Years
Residual value : NIL
Pre-tax required rate of return : 24%
The Hi-Fi Leasing Ltd. seeks your help in determining the annual lease rental under the following rental
structures:
(a) Equated
(b) Stepped (annual increase of 15%)
(c) Ballooned (annual rental of ` 80 lakhs for year 1 & 4)
(d) Deferred (2 years deferment period)
You are required to compute the annual rental.
Ans.: (a) Calculation of equated lease rentals:
Lease rentals = Investment cost/Annuity factor for 5 years of 24% = 500 lakh/2.745 =182.15 lakh
(b) Calculation of stepped lease rentals (annual increase of 15%): Let the lease rental for first year be
Year Lease Payments PV Factor 24% PV
1 * 0.806 0.806*
2 1.15* 0.650 0.7475*
3 1.3225* 0.524 0.69299*
4 1.5209* 0.423 0.6433407*
5 1.749* 0.341 0.596409*
500
0.806x + 0.7475x + 0.69299x + 0.6433407x + 0.596409x = 500
3.4862397x = 500
x = 143.42 lakh
Thus, stepped lease rentals will be as follows:
Year Lease Payments
1 143.42
2 164.93
3 189.67
4 218.12
5 250.84

(c) Calculation of ballooned lease rentals (annual rental of ` 80 lakhs for year 1 & 4):
Year Lease Payments PV Factor 24% PV
1 80 0.806 64.48
2 80 0.650 52.00
3 80 0.524 41.92
4 80 0.423 33.84
5 * 0.341 0.341x
500
64.48 + 52.00 + 41.92 + 33.84 + 0.341x = 500
192.24+ 0.341x = 500
0.341* = 307.76
x = 902.52 lakh
(d) Calculation of Deferred lease rentals (2 years deferment period):
Year Lease Payments PV Factor 24% PV
1 - 0.806 -
2 - 0.650 -
3 * 0.524 0.524*
4 .* 0.423 0.423*
5 * 0.341 0.341*
500
0.524x + 0.423x + 0.341x = 500
1.288x = 500
x = 388.20 lakh
Problem No. 11] ABC Ltd. is considering to acquire an additional sophisticated computer to augment its
time-share computer services to its clients. It has two options:
Either, (a) To purchase the computer at a cost of ` 44,00,000
Or (b) To take the computer on lease for 3 years from a leasing company at an annual lease rental of `
10 lakhs plus 10% of the gross time-share service revenue. The agreement also requires an additional
payment of ` 12 lakhs at the end of the third year. Lease rentals are payable at the year end and the
computer reverts back to lessor after period of contract.
The company estimates that the computers will the worth ` 20 lakhs at the end of the third year. The gross
revenue to be earned is as follows:
Year 12 3
` in lakhs 45 50 55

Annual operating cost (excluding depreciation/lease rental) are estimated at ` 18 lakhs with an additional
cost of ` 2 lakhs for start up and training at the beginning of the first year. These costs are to be borne by
the lessee in case of lease arrangement also. The company proposes to borrow @16% interest to finance
the purchase of the computer and the repayments are to be made as per the following schedule:

(`)
Year end Repayment of principal Interest for year Total
1 10,00,000 7,04,000 17,04,000
2 17,00,000 5,24,000 22,24,000
3 17,00,000 2,72,000 19,72,000
For the purpose of this computation assume that the company uses the straight-line method of
depreciation on assets and pays 50% tax on its income. You are required to analyze and recommend to
the company, which of the two options is better.
Year 1 2 3
PV Factor @ 8% 0.926 0.857 0.794
PV Factor @ 16% 0.862 0.743 0.641
[CS (Professional) - June 1995 & 1999] (20 Marks)
Ans.: Evaluation of loan option:
Depreciation = 44,00,000-20,00,000/3 = 8,00,000
PV Factor = 16(1 - 0.5) = 8%
Calculation of net present value of cash outflow:
Year Principal Interest Depreciation Tax benefit Net cash
outflow PV Factor 8% PV
1 10,00,000 7,04,000 8,00,000 7,52,000 9,52,000 0.926 8,81,552
2 17,00,000 5,24,000 8,00,000 6,62,000 15,62,000 0.857 13,38,634
3 17,00,000 2,72,000 8,00,000 5,36,000 14,36,000 0.794 11,40,184
3 - - - - *(20,00,000) 0.794 (15,88,000)
Total present value of cash outflow 17,72,370
* Salvage value
Evaluation of lease option:
Year Lease 10% of gross Total Tax benefit Net cash PV Factor 8% PV
Rental revenue payment outflow

1 10,00,000 4,50,000 14,50,000 7,25,000 7,25,000 0.926 6,71,350


2 10,00,000 5,00,000 15,00,000 7,50,000 7,50,000 0.857 6,42,750
3 10,00,000 5,50,000 15,50,000 7,75,000 7,75,000 0.794 6,15,350
3 - *12,00,000 . 12,00,000 6,00,000 6,00,000 0.794 4,76,400
Total present value of cash outflow 24,05,850
*Lump sum payment at the end of third year.
Analysis: Present value of cash outflow is less for loan option and hence company is advised to buy
computer availing loan option.
Problem No. 12] The FFM Ltd. is in the tax bracket of 35% and discounts its cash flows at 16%. In the
acquisition of an asset worth ` 10,00,000, it is given two offers: either to acquire the asset by taking a bank
loan @15% p.a. repayable in 5 yearly instalments of ` 2,00,000 each plus interest or to lease-in the asset
at yearly rentals of ` 3,24,000 for 5 years.
In both cases, the instalment is payable at the end of the year.
Applicable rate of depreciation is 15% using ‘written down value’ (WDV) method.
You are required to suggest the better alternative.
Year 1 2 3 4 5
PV Factor @ 16% 0.862 0.743 0.641 0.552 0.476
[CS (Professional) - Dec 1999] (8 Marks)
Ans.:
Evaluation of loan option:
Calculation of net present value of cash outflow:
Year Principal Interest Depreciation Tax benefit Net cash PV Factor 16% PV
outflow
1 2,00,000 1,50,000 1,50,000 1,05,000 2,45,000 0.862 2,11,190
2 2,00,000 1,20,000 1,27,500 86,625 2,33,375 0.743 1,73,398
3 2,00,000 90,000 1,08,375 69,431 2,20,569 0.641 1,41,385
4 2,00,000 60,000 92,119 53,242 2,06,758 0.552 1,14,130
5 2,00,000 30,000 78,301 37,905 1,92,095 0.476 91,437
5 - - - - *(1,55,297) 0.476 (73,921)
Total present value of cash outflow 6,57,619

Cost of asset 10,00,000


(-) Depreciation for 5 years (5,56,295)
Book value at the end of 5th year 4,43,705
(-) Loss (4,43,705)
Sales price 0
(+) Tax benefit on loss (4,43,705 x 35%) 1,55,297
Net cash flow on sale *1,55,297

Evaluation of lease Lease Rental Tax benefit Net cash outflow PV Factor 16% PV
option
Year
1 to 5 3,24,000 1,13,400 2,10,600 3.274 6,89,504
Analysis: Present value of cash outflow is less for loan option and hence company is advised to buy car
availing loan option.
Problem No. 13] Z Securities Ltd., Chennai, is engaged in the business of leasing and hire purchase. The
company also functions as a merchant banker, equity researcher, corporate financier, portfolio manager,
etc. The Company provides fund based as well as non-fund based financial solutions to both wholesale
and retail segments.
P. Securities Ltd., has been approached by AA Ltd., Mumbai, for financial help. AA Ltd. manufactures
process system for food processing, pharmaceuticals, engineering, dairy and chemical industries. A wide
range of centrifugal separators, plate, spray druggers, custom fabricated equipment for exotic metals,
refrigeration compressors, etc., are also manufactured by the company. One of the major strengths of the
company is project management.
AA Ltd. has a well-equipped R&D centre. It has pilot plant facilities and a modern laboratory for chemical,
metallurgical and mechanical analyzer. The company has also set up a technology centre with advanced
testing facilities. Recently, the manager of the technology centre has requisitioned for the acquisition of
computerized sophisticated equipment for conducting important tests.
The equipment is likely to have the useful life of three years. The cost of the equipment is ` 10 Crore. The
scrap value of the equipment at the end of its useful life will be zero for the Company. The Finance
Manager of AA Ltd. has suggested that the company should take a loan for three years from a commercial
bank. Repayment of the loan would be made at the end of each year in three equal instalments. The
repayments would comprise of the (i) principal and (ii) interest at 10% p.a. (on the outstanding amount in
the beginning of the year). AA Ltd. uses a cost of capital of 15% to evaluate the investments of this type.
The equipment will be depreciated @ 33% p.a. (WDV).
Z. Securities Ltd. has agreed to give the equipment to the company on a three year lease. The annual
rental for the lease, payable in the beginning of each year, would be ` 4 Crore ? Z Securities Ltd. discount
its cash flows @ 14%. The equipment is depreciable at 33.33% p.a. (straight-line method). The lessee may
exercise its option to purchase the equipment for ` 4 Crore at the termination of the lease.
AA Ltd. would bear all maintenance, insurance and other charges in both the alternatives. Both the
companies pay tax @ 35%.
You are a Company Secretary in practice. You are approached by the Managing Director of AA Ltd. to help
the company in evaluating the proposal.
Prepare a report for the Managing Director of AA Ltd. showing the effect of the lease alternative on the
wealth of its shareholders. Support your answer with appropriate calculations.
Note: Present value of ` 1 is:
Year 6% 1% 10% 14% 15%
1 0.943 0.935 0.909 0.877 0.870
2 0.890 0.873 0.826 0.769 0.756
3 0.840 0.816 0.751 0.675 0.658
4 0.792 0.763 0.683 0.592 0.572

Present value of an annuity of ` 1 is:


Year 6% 7% 10% 14% 15%
1 0.943 0.935 0.909 0.877 0.870
2 1.833 1.808 1.736 1.647 1.626
3 2.673 2.624 2.487 2.322 2.283
4 3.465 3.387 3.170 2.914 2.855
[CS (Professional) - June 2000] (15 Marks)
Ans. : Evaluatin of loan option :
Loan Amount = Investment cost/Annuity factor of 10% for 3 years = 10 Crore/2.487 = 4.021 Crore
Calculation of principal & interest:
Year Instalment Principal Interest @ 10% Balance
0 - - - 10,000
1 4.021 3.021 1,000 6.979
2 4.021 3.323 0.698 3.656
3 4.021 3.656 0.365 -
Calculation of net present value of cash outflow:
Year Principal Interest Depreciation Tax benefit Net cash PV Factor PV
outflow 15%
1 3.021 1,000 3.333 1.517 2.504 0.870 2.178
2 3.323 0.698 2.222 1.022 2.999 0.756 2.267
3 3.656 0.365 1.482 0.646 3.375 0.658 2.221
3 - - - - (1.037) 0.658 (0.682)
Total present value of cash outflow 5.989

Cost of asset 10,000


(-) Depreciation for 3 years (7.037)
Book value at the end of 3rd year 2.963
(-) Loss (2.963)
Sales price 0
(+) Tax benefit on loss (2.963 X 35%) 1.037
Net cash flow on sale 1.037
Evaluation of lease option:
Year Lease Rental Tax benefit Net cash outflow PV Factor 15% PV
0 4 - 4 1,000 4,000
1 4. 1.4 2.6 0.870 2.262
2 4 1.4 2.6 0.756 1.966
3 - 1.4 (1.4) 0.658 (0.921)
Total present value of cash outflow 7.307
Analysis: Present value of cash outflow is less for loan option and hence company is advised to buy asset
availing loan option.
Problem No. 14] Alfa Ltd. is thinking of installing a computer. Decide whether the computer is to be
purchased outright (through 15% borrowing) or to be acquired on lease rental basis. The rate of income
tax may be taken at 40%. The other data available are as under:
Purchase of computer:

Purchase price ` 20,00,000

Annual maintenance (to be paid in advance) ` 50,000 per year


Expected economic useful life 6 years
Depreciation (for tax purposes) Straight line method

Salvage value ` 2,00,000


Leasing of computer:

Lease charges to be paid in advance ` 4,50,000

Maintenance expenses to be borne by lessor Payment of loan: 6 year-end equal instalments of ` 5,28,474.
Note: Present value of ` 1 for 6 years.
Year PV @ 6% PV @ 9% PV @15%
1 0.9434 0.9174 0.8696
2 0.8900 0.8417 0.7561
3 0.8396 0.7722 0.6575
4 0.7921 0.7084 0.5718
5 0.7473 0.6499 0.4972
6 0.7050 0.5963 0.4323
[CS (Professional) - Dec 2001] (12 Marks)
Ans.: Evaluation of loan option:
Calculation of principal & interest:
Year Instalment Principal Interest @ 1596 Balance
0 - - - 20,00,000
1 5,28,474 2,28,474 3,00,000 17,71,526
2 5,28,474 2,62,745 2,65,729 15,08,781
3 5,28,474 3,02,157 2,26,317 12,06,624
4 5,28,474 3,47,480 1,80,994 8,59,144
5 5,28,474 3,99,602 1,28,872 4,59,542
6 5,28,474 4,59,542 68,932 -
PV Factor =15(1- 0.4) = 996
Calculation of net present value of cash outflow:
Year Principal Interest Depreciation Tax benefit Net cash PV Factor PV
outflow 996
1 2,28,474 3,00,000 3,00,000 . 2,40,000 2,88,474 0.9174 2,64,646
2 2,62,745 2,65,729 3,00,000 2,26,292 3,02,182 0.8417 2,54,347
3 3,02,157 2,26,317 3,00,000 2,10,527 3,17,947 0.7722 2,45,519
4 3,47,480 1,80,994 3,00,000 1,92,398 3,36,076 0.7084 2,38,076
5' 3,99,602 1,28,872 3,00,000 1,71,549 3,56,925 0.6499 2,31,966
6 4,59,542 68,932 3,00,000 1,47,573 3,80,901 0.5963 2,27,131
6 - - - - *(2,00,000) 0.5963 (1,19,260)
Total present value of cash outflow 13,42,425

Cost of asset 20,00,000


(-) Depreciation for 6 years (18,00,000
Book value at the end of 6th year 2,00,000
Profit/Loss Nil
Sales price 2,00,000
Tax on Profit/Loss -
Net cash flow on sale *2,00,000
Evaluation of lease option:
Actual lease rental = 4,50,000
Saving in maintenance expenses — 50,000
Lease rental to be considered for calculation purpose — 4,00,000
Year Lease Rental fax benefit Net cash outflow PV Factor 9% PV
0 4,00,000 4,00,000 1,000 4,00,000
1 to 5 4,00,000 1,60,000 2,40,000 3.8896 9,33,504
6 - 1,60,000 (1,60,000) 0.5963 (95,408)
Total present value of cash outflow 12,38,096
Analysis: Present value of cash outflow is less for leasing option and hence company is advised to take
computer availing lease option.
Problem No. 15] Diligent Ltd. is considering the lease of an equipment which has a purchase price of `
3,50,000. The equipment has an estimated economic life of 5 years with salvage value zero. As per the
income-tax rules, a written down depreciation @ 25% is allowed. The lease rentals per year are ` 1,20,000.
Assume that the Company’s corporate tax rate is 50%. If the before-tax of borrowing for the Company is
16%, should the Company lease the equipment?
Note: Present value of ` 1 for 5 years are:
Year 1 2 3 4 5
PV @ 8% 0.9259 0.8573 0.7938 0.7350 0.6806
PV @16% 0.8621 0.7432 0.6407 0.5523 0.4761
[CS (Professional) - June 2002] (11 Marks)
Ans.:
Diligent Ltd. is considering the lease of equipment by forgoing buy option. So, net advantage of 'lease
option’ over ‘buy option’ has to be considered.
Calculation of tax benefit forgone on depreciation if asset is purchased:
Year Depreciation Tax benefit
1 87,500 43,750
2 65,625 32,813
3 49,219 24,610
4 36,914 18,457
5 27,685 13,843
Net cash outflow if lease option is availed:
Particulars Year 1 Year 2 Year 3 Year 4 Year 5
Lease rentals 1,20,000 1,20,000 1,20,000 1,20,000 1,20,000
(-) Tax benefit on lease rentals (60,000) (60,000) (60,000) (60,000) (60,000)
60,000 60,000 60,000 60,000 60,000
(+) Tax benefit forgone on depreciation 43,750 32,813 24,610 18,457 13,843
Net cash outflow 1,03,750 92,813 84,610 78,457 73,843
Calculation of present value of cash outflow:
Cash outflow PV
Year
1 1,03,750 0.9259 96,062
2 92,813 0.8573 79,569
3 84,610 0.7938 67,163
4 78,457 0.7350 57,666
5 73,843 0.6806 50,258
Present value of cash outflow 3,50,718

If company buy an asset out of its own fund it cost ` 3,50,000 on the other hand net cash outflow in case
of lease option is ` 3,50,718.
3,50,000-3,50,718 = (718)
Analysis: It is suggested to purchase equipment to minimize the cash outflow.
Problem No. 16] Hypothetical Ltd. is planning to have an access to a machine for a period of 5 years. The
company can either have an access through the leasing arrangement or it can borrow money at 14% to
buy the machine. The company is in 50% tax bracket.
In case of leasing, the company will be required to pay annual year-end lease rent of ` 1,20,000 for 5 years.
All maintenance, insurance and other costs are to be borne by the lessee.
In case of purchasing the machine'(which costs ` 3,43,300), the company would have to repay 14% five
year loan in 5 equal annual instalments, each instalment becoming due at the end of each year.
Machine would be depreciated on a straight-line basis, with no salvage value. Advise the company which
option it should go for, assuming lease rents are paid at the end of the year.
[CS (Professional) - June 2003] (10 Marks)
Ans.: Evaluation of loan option:
Loan Amount = Investment cost/Annuity factor of 14% for 5 years = 3,43,300/3.433 = 1,00,000
Calculation of principal & interest:
Year Instalment Principal Interest @ 14% Balance
0 - - - 3,43,300
1 1,00,000 51,938 48,062 2,91,362
2 1,00,000 59,209 40,791 2,32,153
3 1,00,000 67,499 32,501 1,64,654
4 1,00,000 76,948 23,052 87,706
5 1,00,000 87,706 12,294 -
PV Factor = 14 (1 - 0.5) = 7%
Calculation of net present value of cash outflow:
Year Principal Interest Depreciation Tax benefit Net cash PV Factor 7% PV
outflow
1 51,938 48,062 68,660 58,361 41,639 0.935 38,932
2 59,209 40,791 68,660 54,726 45,274 0.873 39,524
3 67,499 32,501 68,660 50,581 49,419 0.816 40,326
4 76,948 23,052 68,660 . 45,856 54,144 0.763 41,312
5 87,706 12,294 68,660 40,477 59,523 0.713 42,440
Total present value of cash outflow 2,02,534
Evaluation of lease
option
Year Lease Rental Tax benefit Net cash outflow PV Factor 7% PV
1 to 5 1,20,000 60,000 60,000 4.100 2,46,000
Analysis: Present value of cash outflow is less for loan option and hence company is advised to buy
machine availing loan option.
Problem No. 17] Alfa Ltd. is in the business of manufacturing bearings. Some more product lines are being
planned to be added to the existing system. To manufacture the planned product lines, the firm needs a
machine which if purchased outright will cost ` 10,00,000. Modern Hire-Purchase and Leasing Co. has
offered two proposals as below:
Proposal-I (Hire Purchase): ` 2,50,000 will be payable on signing of the agreement. Three annual
instalments of ` 4,00,000 will be payable at the end of each year starting from year first. The ownership of
the machine will be transferred automatically at the end of third year. The company will be able to claim
depreciation on straight line basis with zero salvage value.
Proposal-II (Lease): ` 20,000 will be payable towards initial service fee upon signing of the agreement
which is tax-deductible expense. Annual lease rental of ` 4,32,000 is payable at the end of each year
starting from the first year for a period of three years.
Evaluate the above two proposals and advise the company as to which proposal implies lesser cost given
that tax rate is 35% and discount rate is 20%.
(Calculations may be rounded off to Rupee.)
[CS (Professional) - Dec 2009] (20 Marks)
Ans.:
Evaluation of hire purchase option:
Cash price of machine = 10,00,000
Hire purchase price = 14,50,000 (2,50,000 + 4,00,000 + 4,00,000 + 4,00,000)
Interest = 4,50,000
Allocation of interest (in the ratio of balance due at the end of respective years):
Year Balance due Interest
1 12,00,000 2,25,000
2 8,00,000 1,50,000
3 4,00,000 75,000
24,00,000 4,50,000
Calculation of net present value of cash outflow:
Year Hire Interest Depreciation Tax benefit Net cash PV Factor PV
charges outflow 20%
0 2,50,000 - - - 2,50,000 1,000 2,50,000
1 4,00,000 2,25,000 3,33,333 1,95,417 2,04,583 0.833 1,70,418
2 4,00,000 1,50,000 3,33,333 1,69,167 2,30,833 0.694 1,60,198
3 4,00,000 75,000 3,33,333 1,42,917 2,57,083 0.579 1,48,851
Total present value of cash outflow 7,29,467
Evaluation of lease option:
Calculation of net present value of cash outflow:
Year Lease Rental Tax benefit Net cash outflow PV Factor 20% PV
0 20,000 7,000 13,000 1,000 13,000
1 to 3 4,32,000 1,51,200 2,80,800 2.106 5,91,365
Total present value of cash outflow 6,04,365
Analysis: Present value of cash outflow is less for leasing option and hence company is advised to take
machine availing lease option.
Problem No. 18] Apple Ltd. has decided to invest in earth-moving equipment. The equipment costs `
5,50,000. The company can take it on lease for 7 years at ` 90,000 p.a. payable in advance. Alternatively,
it can borrow at 20%. The asset can be written-off over 6 years under straight line method of depreciation
and this is allowed under tax. The asset’s useful life is 7 years. In the terminal year the asset will be sold
for a net value of ` 40,000. The applicable tax rate is 30%. Should the company borrow for buying the
equipment or take it on lease?
[CS (Professional) - June 2016] (8 Marks)
Ans.: Evaluation of loan option:
Loan Amount = Investment cost/Annuity factor of 20% for 7 years = 5,50,000/3.605 = 1,52,566
Calculation of principal & interest:
Year Instalment Principal Interest @ 20% Balance
0 - 1 - 5,50,000
1 1,52,566 42,566 1,10,000 5,07,434
2 1,52,566 51,079 1,01,487 4,56,355
3 1,52,566 61,295 91,271 3,95,060
4 1,52,566 73,554 79,012 3,21,506
5 1,52,566 88,265 64,301 2,33,241
6 1,52,566 1,05,918 46,648 1,27,323
7 1,52,566 1,27,323 25,243 -
PV Factor = 20 (1 - 0.3) = 14%
Calculation of net present value of cash outflow: 1,52,566
Year Principal Interest Depreciation Tax benefit outflow PV Factor PV
1 42,566 1,10,000 72,857 54,857 97,709 0.877 85,691
2 51,079 1,01,487 72,857 52,303 1,00,263 0.769 77,102
3 61,295 91,271 72,857 49,238 1,03,328 0.675 69,746
4 73,554 79,012 72,857 45,561 1,07,005 0.592 63,347
5 88,265 64,301 72,857 41,147 1,11,419 0.519 57,826
6 1,05,918 46,648 72,857 35,852 1,16,714 0.456 53,222
7 1,27,323 25,243 72,857 29,430 1,23,136 0.400 49,254
7 - - - - (40,000) 0.400 (16,000)
Total present value of cash outflow 4,40,188
Evaluation of lease option:
Calculation of net present value of cash outflow:
Year Lease Rental Tax benefit Net cash outflow PV Factor 14% PV
0 90,000 - 90,000 1,000 90,000
1 to 6 90,000 27,000 63,000 3.889 2,45,007
7 - 27,000 (27,000) 0.400 (10,800)
Total present value of cash outflow 3,24,207
Analysis: Present value of cash outflow is less for leasing option and hence company is advised to take
earth-moving equipment availing lease option.
CHAPTER
12
CAPITAL BUDGETING - ASSET REPLACEMENT DECISIONS
Problem No. 1] New Style Ltd. is considering the replacement of one of its moulding machines, The existing
machine is in good operating condition, but is smaller than required for the firm to expand its operations.
The old machine is 5 years old, and has remaining depreciable life of 10 years. The machine was originally
purchased for ` 1,50,000 and is being depreciated at ` 10,000 per year for tax purposes.
The new machine will cost ` 2,20,000 or ` 1,70,000 if exchanged with the existing machine. It will be
depreciated on a straight-line basis for 10 years with no salvage value. The management anticipates that,
with the increased operations, there will be need for an additional net working capital of ` 30,000. The
new machine will allow the company to expand current operations, thereby increasing annual revenue by
` 60,000 and variable operating costs from ` 2,00,000 to ` 2,20,000.
The company’s tax rate is 35% and its cost of capital is 10%.
Should the company replace its existing machine? Assume that the loss on exchange of existing machine
can be claimed as short-term capital loss in the current year itself.
Ans.: Calculation of net cash inflow on sale of existing machine:
Cost of old machine 1,50,000
(-) Depreciation for 5 years (50,000)
Current book value 1,00,000
(-) Loss on sale (50,000)
Sale price (2,20,000 - 1,70,000) 50,000
(+) Tax benefit on loss (50,000 x 35%) 17,500
Net cash inflow on sale of old machine 67,500
Calculation of initial investment in new machine:
Cost of new machine 2,20,000
(+) Additional working capital 30,000
(-) Cash inflow on sale of old machine (67,500)
Net investment in new machine 1,82,500
Calculation of incremental cash inflow if new machine is purchased:
Incremental sales revenue 60,000
(-) Incremental variable operating cost (20,000)
(-) Incremental depreciation (22,000 - 10,000) (12,000)
EBT 28,000
(-) Tax @ 35% (9,800)
PAT 18,200
(+) Depreciation 12,000
CFAT 30,200
Calculation of NPV:
Year Cash flow PV Factor PV
1 to 10 30,200 6.145 1,85,579
10 *30,000 0.386 11,580
Total present value 1,97,159
(-) Initial investment (1,82,500)
NPV 14,659
* Working capital released.
Analysis: Since incremental NPV is positive, the company is suggested to replace the existing machine.
Problem No. 2] X Ltd. acquired a machine for ` 3,00,000 having its life for about 10 years. Five years has
gone so far with this machine which has its book value ` 1,50,000. Now management can continue with
this machine for the remaining lifetime. However, a new machine worth ` 5,00,000 is also available for
consideration. Comment on the suitability or replacement of the old machine. Following information is
furnished in this regard:
(i) The activity level of both the machines is the same.
(ii) Expected life of the new machine is 5 years.
(iii) Corporate tax 50%.
(iv) Residual value of new machine at the end ` 30,000.
(v) X Ltd. expects a minimum return of 10% on the investment.
(vi) The new machine can cut down the variable cost by ` 2,00,000 p.a.
(vii) The new machine requires one-time service at the end of the second year of its installation, costing `
12,000.
(viii) If the old machine is sold out, then it will fetch ` 50,000.
Ans.: Calculation of net cash inflow on sale of existing machine:
Cost of old machine 3,00,000
(-) Depreciation for 5 years (1,50,000)
Current book value 1,50,000
(-) Loss on sale (1,00,000)
Sale price 50,000
(+) Tax benefit on loss (1,00,000 x 50%) 50,000
Net cash inflow on sale of old machine 1,00,000
Calculation of initial investment in new machine:
Cost of new machine 5,00,000
(-) Cash inflow on sale of old machine (1,00,000)
Net investment in new machine 4,00,000
Calculation of incremental cash inflow if new machine is purchased:
Cash inflow due to saving in variable cost 2,00,000
(-) Incremental depreciation (1,00,000 - 30,000) (70,000)
1,30,000
(-) Tax @ 50% (65,000)
PAT 65,000
(+) Depreciation 70,000
CFAT 1,35,000
Calculation of net cash inflow on sale of new machine at the end of 5th year:
Cost of new machine 5,00,000
(-) Depreciation for 5 years (5,00,000)
Book value at the end of 5th year 0
Profit/loss 30,000
Sale price 30,000
(-) Tax on profit (30,000 x 50%) (15,000)
Net cash inflow on sale of new machine 15,000
Calculation of NPV:
Year Cash inflow PV Factor PV
1 to 5 1,35,000 3.791 5,11,785
5 15,000 0.621 9,315
Present value of cash inflow 5,21,100
Year Cash outflow PV Factor PV
0 4,00,000 1,000 4,00,000
2 12,000 0.827 9,924
Present value of cash outflow 4,09,924
NPV 1,11,176
Analysis: Since, NPV is positive, the company is suggested to replace the existing machine.
Problem No. 3] P Ltd. has a machine having an additional life of 5 years, which costs ` 10,00,000 and has
a book value of ` 4,00,000. A new machine costing ` 20,00,000 is available. Though its capacity is the same
as that of the old machine, it will mean a saving in variable costs to the extent of ` 7,00,000 p.a. The life of
the machine will be 5 years at the end of which it will have a scrap value of ` 2,00,000.
The rate of income tax is 46% and P Ltd. policy is not to make an investment if the yield is less than 12% '
p.a. The old machine, if sold today, will realize ` 1,00,000; it will have no salvage value if sold at the end of
5th year. Advise P Ltd., whether or not the old machine should be replaced.
[CS (Professional) - June 1994] (10 Marks)
[CS (Executive) - Dec 2018] (7 Marks)
Ans.: Calculation of net cash inflow on sale of existing machine:
Cost of old machine 10,00,000
(-) Depreciation (6,00,000)
Current book value 4,00,000
(-) Loss on sale (3,00,000)
Sale price 1,00,000
(+) Tax benefit on loss (3,00,000 x 46%) 1,38,000
Net cash inflow on sale of old machine 2,38,000
Calculation of initial investment in new machine:
Cost of new machine 20,00,000
(-) Cash inflow on sale of old machine (2,38,000)
Net investment in new machine 17,62,000
Calculation of incremental cash inflow if new machine is purchased:
Saving in variable costs 7,00,000
(-) Incremental depreciation (3,60,000 - 80,000) (2,80,000)
EBT 4,20,000
(-) Tax @ 46% (1,93,200)
PAT 2,26,800
(+) Depreciation 2,80,000
CFAT 5,06,800
Calculation of NPV:
Year Cash flow PV Factor 12% PV
1 to 5 5,06,800 3.605 18,27,014
5 *1,08,000 0.567 61,236
Total present value 18,88,250
(-) Initial investment (17,62,000)
NPV 1,26,250
Analysis: Since, incremental NPV is positive, the company is suggested to replace the existing machine.
Working Note:
Depreciation on new machine = 20,00,000-2,00,000/5 = 3,60,000
Depreciation on old machine = Book value/Remaining useful life = 4,00,000/5 = 80,000
* Net cash flow on sale of new machine at the end of 5th year = 2,00,000 (1 - 0.46) = 1,08,000
Problem No. 4] A machine purchase 6 year back for ` 1,50,000 has been depreciated to a book value of `
90,000. It originally had a projected life of 15 years and zero salvage value.
A new machine will cost ` 2,50,000 and result in a reduced operating cost of ` 30,000 per year for the next
9 years. The older machine could be sold for ` 50,000. The new machine will also be depreciated on a
straight line method on 9 year with salvage value of ` 25,000.
The company’s tax rate is 50% and cost of capital is 10%. Determine whether the old machine should be
replaced.
[CS (Professional) - June 1995] (10 Marks)
Ans.: Calculation of net cash inflow on sale of existing machine:
Cost of old machine 1,50,000
(-) Depreciation for 6 years (60,000)
Current book value 90,000
(-) Loss on sale (40,000)
Sale price 50,000
(+) Tax benefit on loss (40,000 x 50%) 20,000
Net cash inflow on sale of old machine 70,000
Calculation of initial investment in new machine:
Cost of new machine 2,50,000
(-) Cash inflow on sale of old machine (70,000)
Net investment in new machine 1,80,000
Calculation of incremental cash inflow if new machine is purchased:
Cash inflow due to saving in operating cost 30,000
(-) Incremental depreciation (25,000 - 10,000) (15,000)
EBT 15,000
(-) Tax @ 50% (7,500)
PAT 7,500
(+) Depreciation 15,000
CFAT 22,500
Calculation of NPV:
Year Cash flow PV Factor 10% PV
1 to 9 22,500 5.759 1,29,578
9 *12,500 0.424 5,300
Total present value (-) Initial investment NPV 1,34,878
(1,80,000)
(45,122)
Analysis: Since incremental NPV is negative, the company is suggested not to replace the existing machine.
Working Note:
Depreciation on new machine = 2,50,000-25,000/9 = 25,000
Depreciation on old machine = Book value/Remaining usetul life = 90,000/9 = 10,000
* Net cash flow on sale of new machine at the end of 5th year = 25,000 (1 - 0.5) = 12,500
Problem No. 5] An existing company has a machine which has been in operation for 2 years. Its remaining
estimated useful life is 10 years, with no salvage value at the end. Its current market value is ` 1,00,000.
The management is considering a proposal to purchase an improved model of a similar machine, which
gives increased output. The relevant particulars are as follows:
Existing Machine New Machine
Purchase price ` 2,40,000 ` 4,00,000
Estimated life 12 years 10 years
Salvage value Nil Nil
Annual operating hours 2,000 2,000
Selling price per unit ` 10 ` 10
Output per hour 15 units 30 units
Material cost per unit `2 `2
Labour cost per hour ` 20 ` 40
Consumable stores per year ` 2,000 ` 5,000
Repairs & maintenance per year ` 9,000 ` 6,000
Working capital ` 25,000. ` 40,000
The company follows the straight line method of depreciation and is subject to 50% tax. Should the
existing machine be replaced? Assume that the company’s required rate of return is 15%.
[CS (Professional) - Dec 1995 & Dec 2003] (20 Marks)
Ans.: Calculation of net cash inflow on sale of existing machine:
Cost of old machine 2,40,000
(-) Depreciation for 2 years (40,000)
Current book value 2,00,000
(-) Loss on sale (1,00,000)
Sale price 1,00,000
(+) Tax benefit on loss (1,00,000 * 50%) 50,000
Net cash inflow on sale of old machine 1,50,000
Calculation of initial investment in new machine:
Cost of new machine 4,00,000
(+) Incremental working capital 15,000
(-) Cash inflow on sale of old machine (1,50,000)
Net investment in new machine 2,65,000
Output per annum:
Existing machine = 2,000 hours x 15 units = 30,000 units New machine = 2,000 hours x 30 units = 60,000
units
Calculation of cash inflow:
Particulars Existing New Differential
machine machine
30,000 units 60,000 units 30,000 units
Sales 3,00,000 6,00,000 3,00,000
(-) Expense:
- Material cost 60,000 1,20,000 60,000
- Labour cost 40,000 80,000 40,000
- Consumable stores 2,000 5,000 3,000
- Repairs & maintenance 9,000 6,000 (3,000)
- Depreciation 20,000 40,000 20,000
Total expenses 1,31,000 2,51,000 1,20,000
EBT . 1,69,000 3,49,000 1,80,000
(-) Taxes (84,500) (1,74,500) (90,000)
PAT 84,500 1,74,500 90,000
(+) Depreciation 20,000 40,000 20,000
CFAT 1,04,500 2,14,500 1,10,000

Calculation of NPV : Cash flow PV Factor PV


Year
1 to 10 1,10,000 5.019 5,52,090
10 *15,000 0.247 3,705

Total present value (-) Initial investment NPV 5,55,795


(2,65,000)

2,90,795

* Working capital released.


Analysis: Since incremental NPV is positive, the company is suggested to replace the existing machine.
Problem No. 6] A product is currently being manufactured on a machine that has a book value of ` 30,000.
The machine was originally purchased for ` 60,000 ten years ago.
Per unit costs of the product are:
Direct labour ` 8.00, Direct materials ? f0.00, Variable overheads ` 5.00, Fixed overheads ` 5.00, and total
cost is ` 28.00. In the past year 6,000 units were produced and sold for ` 50.00 per unit. It is expected that
the old machine can be used indefinitely in the future.
An equipment manufacturer has offered to accept the old machine at ` 20,000, a trade in for a new
version. The purchase price of the new machine is ` 1,00,000. The projected per unit costs associated with
the new machine are direct labour ` 4.00, direct materials ` 7.00, variable overheads ` 4.00, fixed overheads
` 7.00 and total cost is ` 22.00
The management also expects that, if the new machine is purchased, the new working capital requirement
of the company would be less by ` 10,000. The fixed overheads costs are allocations from other
departments plus the depreciation of the equipment. The new machine has an expected life of 10 years
with no salvage value, the straight line method of depreciation is employed by the company. It is expected
that the future demand of the product would remain at 6,000 units per year. Should the new equipment
be acquired? Corporate tax is @ 50%. Cost of capital is 10%
[ CS (Professional) - Dec 1996] (20 Marks)
Ans.: Calculation of net cash inflow on sale of existing machine:
Cost of old machine 60,000
(-) Depreciation for 10 years (30,000)
Current book value 30,000
(-) Loss on sale (10,000)
Sale price 20,000
(+) Tax benefit on loss (10,000 x 50%) 5,000
Net cash inflow on sale of old machine 25,000
Calculation of initial investment in new machine:
Cost of new machine 1,00,000
(-) Reduction in working capital (10,000)
(-) Cash inflow on sale of old machine (25,000)
Net investment in new machine 65,000
Calculation of incremental cash inflow if new machine is purchased:
Cash inflow due to saving in variable cost (23 -15)* 6,000 48,000
(-) Incremental depreciation (10,000 - 3,000) (7,000)
EBT 41,000
(-) Tax @ 50% (20,500)
PAT 20,500
(+) Depreciation 7,000
CFAT 27,500

Calculation of NPV:
Year Cash flow PV Factor PV
1 to 10 27,500 6.145 1,68,988
(-) Initial investment (65,000)
NPV 1,03,988
Analysis: Since, incremental NPV is positive, the company is suggested to replace the existing machine.
Problem No. 7] AP Udyog is considering a new automatic blender. The new blender would last for 10
years and would be depreciated to zero over the 10 year period. The old blender would also last for 10
more years and would be depreciated to zero over the same 10 years period. The old blender has a book
value of ` 20,000 but could be sold for ` 30,000 (the original cost was ` 40,000). The new blender would
cost ` 1,00,000.
It would reduce labour expense by ` 12,000 a year. The company is subject to a 50% tax rate on regular
income and 30% tax rate on capital gains. Their cost of capital is 8%. There is no investment tax credit in
effect.
You are required to:
(`) Identify all the relevant cash flows for this replacement decision.
(ii) Compute the present value and net present value.
(iii) Find out whether this is an attractive project.
[CS (Professional) - June 1997] (5 + 3 + 2=10 Marks)
Ans.: Calculation of net cash inflow on sale of existing machine:
Cost of old machine 40,000
(-) Depreciation (20,000)
Current book value 20,000
(+) Profit on sale 10,000
Sale price 30,000
(-) Capital gain tax on profit (10,000 x 30%) (3,000)
Net cash inflow on sale of old machine 27,000
Calculation of initial investment in new machine:
Cost of new machine 1,00,000
(-) Cash inflow on sale of old machine (27,000)
Net investment in new machine 73,000
Calculation of incremental cash inflow if new machine is purchased:
Cash inflow due to saving labour expense 12,000
(-) Incremental depreciation (10,000 - 2,000) (8,000)
EBT 4,000
(-) Tax @ 50% (2,000)
PAT 2,000
(+) Depreciation 8,000
CFAT 10,000
Calculation of NPV:
Year Cashflow PV Factor 8% PV
1 to 10 10,000 6.71 67,100
(-) Initial investment . (73,000)
NPV (5,900)
Analysis: Since incremental NP V is negative, the company is suggested not to replace the existing
machine.
Problem No. 8] Prithvi Ltd. is a manufacturer of variety of electrical equipments. The existing machine is
based on old technology. In order to improve the quality of the product and bring down operating cost,
the management is planning to replace the existing machine with a new one based on latest technology.
Following are the relevant information:
Existing machine:
Purchased - 5 years ago
Remaining life - 5 years
Salvage value - ` 20,000
Depreciation -` Straight line basis
Current book value - ` 3,00,000
Realizable market value - ` 3,50,000
Annual depreciation - ` 28,000
New replacement machine:
Capital cost - ? 10,00,000
Estimated useful life - 5 years
Estimated salvage value - ` 1,00,000
The replacement machine would permit an output expansion. As a result, sales is expected to increase by
` 1,00,000 per year, operating expenses would decline by ` 2,00,000 per year. It would require an
additional inventory of ` 2,00,000 and would cause an increase in accounts payable by ` 50,000. Assuming
a corporate tax rate of 30% and cost of capital of 12%, advise the company about replacement of the
existing machine.
[CS (Professional) - June 2007] (10 Marks)
Ans.:
Calculation of net cash inflow on sale of existing machine:
Current book value 3,00,000
(+) Profit 50,000
Sale price 3,50,000
(-) Tax on profit (50,000 x 30%) (15,000)
Net cash inflow on sale of old machine 3,35,000
Calculation of initial investment in new machine:
Cost of new machine 10,00,000
(+) Additional working capital (2,00,000 - 50,000) 1,50,000
(-) Cash inflow on sale of old machine (3,35,000)
Net investment in new machine 8,15,000
Calculation of incremental cash inflow if new machine is purchased:
1,00,000
Incremental sales revenue
Cash inflow due to saving in operating expenses 2,00,000
(-) Incremental depreciation (1,80,000 - 28,000) (1,52,000)
EBT 1,48,000
(-) Tax @ 30% (44,400)
PAT 1,03,600
(+) Depreciation 1,52,000
CFAT 2,55,600
Calculation of NPV:
Year Cash flow PV Factor 12% PV
1 to 5 2,55,600 3.605 9,21,438
5 *1,50,000 0.567 85,050
5 *1,00,000 0.567 56,700
Total present value (-) 10,63,188
Initial investment (8,15,000)
NPV 2,48,188
* Working capital & salvage value.
Analysis: Since incremental NPV is positive, the company is suggested to replace the existing machine.
Problem No. 9] An existing machine in Bipasha Ltd. can be sold today for ` 1,00,000 net. The cash flow
after tax (CFAT) for the balance life of 4 years is ` 30,000 p.a. At the end of the 4th year, the existing
machine can be sold for ` 20,000 net.
A new machine can replace the existing machine at a net cash outflow of ` 1,50,000 will generate annual
CFAT of ` 46,000. The scrap value at the end of its useful life will be ` 25,000 net.
If the discount rate is 10%, decide whether the existing machine should be replaced with a new machine.
[CS (Professional) - Dec 2007] (8 Marks)
Ans.:
Calculation of NPV if existing machine is continued:
Year Cash flow PV Factor PV
1 to 4 30,000 3.170 95,100
4 20,000 0.683 13,660
Total present value 1,08,760
(-) Initial investment* ‘f' (1,00,000)
NPV 8,760
’Cash forgone by not selling exiting machine Calculation of NPV if new machine is purchased:
Year Cash flow PV Factor PV
1 to 4 46,000 3.170 1,45,820
4 25,000 0.683 17,075
Total present value 1,62,895
(-) Initial investment (1,50,000)
NPV 12,895
Analysis: Since, NPV of the new machine is greater, it is suggested to replace the old machine with new
one.
Problem No. 10] A product is currently manufactured in a machine that is not fully depreciated for tax
purpose and has a book value of ` 60,000 (it was bought for ` 1,20,000 six years ago). The cost of the
product is as under.
(Unit cost ?)
Direct cost 24
Indirect labour 8
Other variable overheads 16
Fixed overheads 16
64
Normally 10,000 units of the product are produced. It is expected that the old machine can be used
indefinitely into the future, after suitable repairing estimated to cost ` 40,000 annually is carried out.
There is an offer for a new machine with latest improved technology at ` 3,00,000 after trading off the old
existing machinery for ` 30,000. The projected cost of the product will then be as under:
(Unit cost ?)
Direct cost 14
Indirect labour 12
Other variable overheads 12
Fixed overheads 20
58
The fixed overheads are allocations from other departments plus the depreciation of the plant and
machinery.
The old machine can be sold in the open market for ` 40,000. The new machine will last for 10 years at the
end of which it will have a salvage value of ` 20,000. Assume rate of corporate tax at 50%. For tax purpose
the cost of the new machine and that of the old one may be depreciated in 10 years. The minimum rate
of return expected is 10%. It is also expected that the future demand of the product will remain steady at
10,000 units. Advise whether the new machine should be purchased. Ignore capital gains tax.
Present values of ` 1 at 10% for 10 years are:
Year 1 2 3 4 5 6 7 8 9 10 .
PV 0.909 0.826 0.751 0.683 0.621 0.564 0.513 0.467 0.424 0.386
[CS (Professional) - June 1991 & Dec 2008] (20 Marks)
Ans.:
Calculation of initial investment in new machine:
Price of new machine after trading off the old machinery = 3,00,000 Calculation of value for depreciation
purpose:
Price of new machine after trading off the old machinery 3,00,000
(+) Value of old plant 30,000
(-) Salvage value (20,000)
3,10,000

Depreciation on new machinery = 3,10,000/10 = 31,000


Depreciation on old machinery = 60,000/6 = 10,000
Variable cost per unit under existing machine = 24+ 8 + 16 = 48 Variable cost per unit under new machine
= 14 + 12 + 12 = 38 Calculation of incremental cash inflow if new machine is purchased:
Cash inflow:
Saving in variable cost (10,000 x 10) 1,00,000
Saving in annual repair of old machine 40,000
1,40,000
(-) Incremental depreciation (31,000 - 10,000) (21,000)
EBT 1,19,000
(-) Tax @ 50% (59,500)
PAT 59,500
(+) Depreciation 21,000
CFAT 80,500
Calculation of NPV:
Year Cash inflow PV Factor PV
1 to 10 80,500 6.144 4,94,592
10 *10,000 0.386 3,860
Present value of cash inflow 4,98,452
(-) Initial investment (3,00,000)
NPV 1,98,452
* Salvage value after tax = 20,000 (1 - 0.5) = 10,000
Analysis: Since NPV is positive; existing machine can be replaced by new machine.
Problem No. 11] Exacta Ltd. is considering the replacement of its existing machine by a new one which is
expected to cost ` 2,70,000 with a life of 5 years and salvage value being ` 20,000. The machine will yield
annual cash revenue of ` 5,70,000 and annual cash expenses of ` 2,96,000.
The existing machine has a book value of 192,000 and can be sold for ` 46,000 today. It has a remaining
useful life of 5 years. Cash revenues will be ` 4,50,000 and associated cash expenses will be ` 3,20,000 p.a.
The existing machine will have a salvage value of ` 4,600 at the end of 5 years.
Exacta Ltd. is in a 35% tax bracket and writes off depreciation @ 25% p.a. on WDV method. Exacta Ltd.
has a target debt-equity ratio of 20%. The company in the past has raised debt at 12% and it can now be
raised at 10%. Exacta Ltd. follows the dividend discount model to estimate the cost of equity capital. Last
year the company paid a dividend of ` 1.85 per share. The current market price of the company’s equity
share is ` 20 per share. A growth rate of 8%'p.a. is anticipated. (Ignore capital gain tax)
Required -
(i) Investment required on incremental basis.
(ii) Incremental depreciation per year.
(iii) Weighted average cost of capital.
(iv) Computation of present worth factors.
(v) Before tax incremental cash flow based on revenue and expenses.
(vi) Incremental terminal cash flow.
(vii) Computation of NPV.
(viii) Should the new machine be acquired? Why?
[CS (Professional) - June 2008] (20 Marks)
Ans.:
(i) Calculation of investment required on incremental basis :
Cost of new machine 2,70,000
(-) Cash inflow on sale of old machine (46,000)
Net investment in new machine 2,24,000
(ii) Calculation of incremental depreciation per year:
Year Existing Machine New Machine Incremental Depreciation
1 23,000 67,500 44,500
2 17,250 50,625 33,375
3 12,938 37,969 25,031
4 9,703 28,477 18,774
5 7,277 21,357 14,080
(iii) Computation of weighted average cost of capital:
D1 =D0 (1+g)
Ke = D1/P0 + g
= 1.85 (1 +0.08)
= 1.998/20 + 0.08
= 1.998
Ke = 0.1799 i.e. 17.99% say 1896
Kd = I(1-t)
= 10 (1 - 0.35)
= 6.5%
Types of capital 96 Cost of Capital Product
Equity 8096 1896 1,440
Debt 2096 6.596 130
10096 1,570
WACC = 1,570/100 = 15.7%
(iv) Computation of present worth factors (i.e. PV Factors):
Year PV Factors 15.7%
1 0.864
2 0.747
3 0.646
4 0.558
5 0.482
(v) Calculation of incremental terminal cash flow:
Particulars Existing New
Machine Machine
Cash revenue 4,50,000 5,70,000
(-) Cash expense (3,20,000) (2,96,000)
Cash earnings 1,30,000 2,74,000
Increase in cash earnings = 1,44,000
Particulars Year 1 Year 2 Year 3 Year 4 Year 5
Cash inflow 1,44,000 1,44,000 1,44,000 1,44,000 1,44,000
(-) Incremental depreciation (44,500) (33,375) (25,031) (18,774) (14,080)
EBT 99,500 1,10,625 1,18,969 1,25,226 1,29,920
(-) Tax @ 3596 (34,825) (38,719) (41,639) (43,829) (45,472)
PAT 64,675 71,906 77,330 81,397 84,448
(+) Depreciation 44,500 33,375 25,031 18,774 14,080
CFAT 1,09,175 1,05,281 1,02,361 1,00,171 98,528
(vi) Computation of NPV:
Year CFAT PV Factors 15.796 PV
1 1,09,175 0.864 94,327
2 1,05,281 0.747 78,645
3 1,02,361 0.646 66,125
4 1,00,171 0.558 55,895
5 98,528 0.482 47,490
5 *15,400 0.482 7,423
Total present value 3,49,905
(-) Initial investment (2,24,000)
NPV 1,25,905
‘Incremental terminal cash flow = Salvage value of new machine - Salvage value of old machine
= 20,000 - 4,600
= 15,400
(vii) Analysis: Since incremental NPV is positive, the company is suggested to replace the existing machine.
CHAPTER
13
CAPITAL BUDGETING - RISK ANALYSIS
Question 1] Risk and uncertainty is quite inherent in capital budgeting. Comment.
CS (Professional) - June 2011 (5 Marks), What are the risk and uncertainty in capital budgeting
decisions?
CS (Professional) - Dec 2015 (4 Marks)
Ans.: Risk analysis gives management better information about the possible outcomes that may occur so
that management can use their judgment and experience to accept an investment or reject it. Since risk
analysis is costly, it should be used relatively in costly and important projects.
Risk and uncertainty are quite inherent in capital budgeting decisions. This is so because investment
decisions and capital budgeting are actions of today which bear fruits in future which is unforeseen. Future
is uncertain and involves risk. The projection of probability of cash inflows made today is not certain to be
achieved in the course of future. Seasonal fluctuations and business cycles both deliver heavy impact upon
the cash inflows and outflows projected for different project proposals. The cost of capital which offers
cut-off rates may also be inflated or deflated under business cycle conditions. Inflation and deflation are
bound to effect the investment decision in future period rendering the degree of uncertainty more severe
and enhancing the scope of risk.
Technological developments are other factors that enhance the degree of risk and uncertainty by
rendering the plants or equipments obsolete and the product out of date. Tie up in the procurement in
quantity and/or the marketing of products may at times fail and frustrate a business unless possible
alternative strategies are kept in view. All these circumstances combined together affect capital budgeting
decisions.
It is therefore necessary to allow discounting factor to cover risk. One way to compare risk in alternative
proposals is the use of Standard Deviation.
Lower standard deviation indicates lower risk.
However, wherever returns are expressed in revenue terms the co-efficient of variation gives better
measurement for risk evaluation.
Question 2] There are number of statistical/mathematical techniques of risk evaluation in capital
budgeting. Comment.
CS (Professional) - Dec 2014 (5 Marks)
Ans.: Following statistical/mathematical techniques of risk evaluation are used in capital budgeting:
(a) Certainty Equivalent Approach
(b) Probability Assignment
(c) Expected Net Present Value
(d) Standard Deviation
(e) Coefficient of Variation (/) Sensitivity Analysis
(g) Simulation
(h) Probability Distribution Approach
(i) Normal Probability Distribution (;) Linear Programming
Question 3] Write a short note on: Certainty Equivalent Approach
Ans.: Certainty Equivalent Factor (CEF) is the ratio of assured cash flows to uncertain cash flows. Under
this approach, the cash flows expected in a project are converted into risk-less equivalent amount. The
adjustment factor used is called CEF. This varies between 0 and 1. A co-efficient of 1 indicates that cash
flows are certain. The greater the risk in cash flow, the smaller will be CEF 'for receipts’, and larger will be
the CEF ‘for payments’. While employing this method, the decision maker estimates the sum he must be
assured of receiving, in order that he is indifferent between an assured sum and expected value of a risky
sum.
Method of Computation under CE approach:
Step 1: Convert uncertain cash flows to certain cash flows by multiplying it with the CEF.
Step 2: Discount the certain cash flows at the risk free rate to arrive at NPV. - Decision Rule: If the resultant
NPV is positive project can be accepted.
Illustration: NZ Ltd. is considering to take a new project. The management of the company use Certainty
Equivalent (CE) approach to evaluate such type of projects.'
Following information is available for the project:
Year CFAT CE
1 1,15,000 0.90
2 1,15,000 0.85
3 1,15,000 0.75
4 1,15,000 0.70
5 1,15,000 0.65

Projects requires initial investment of ` 3,00,000. The Company’s cost of capital is 12% and risk free
borrowing rate is 7%.
Advise the company whether it should take project or not?
Solution:
Year CFAT CE Adjusted PV
CFAT PV Factor 1%
1 1,15,000 0.90 1,03,500 0.935 96,772
2 1,15,000 0.85 97,750 0.873 85,336
3 1,15,000 0.75 86,250 0.816 70,380
4 1,15,000 0.70 80,500 0.763 61,422
5 1,15,000 0.65 74,750 0.713 53,297
Total Present Valve 3,67,207
(-) Initial Investment (3,00,000)
Net present value 67,207
Since NPV is positive, project can be accepted.
Question 4] Write a short note on: Risk adjusted discount rate
Ans.: Risk-adjusted discount rate is the rate used in the calculation of the present value of> a risky
investment. It is calculated as follows:
Formula:
Rf+β (Rm-Rf)
The risk-adjusted discount rate is the total of the risk-free rate, ie. the required return on risk-free
investments, and the market premium, ie. the required return of the market. Financial analysts use the
risk-adjusted discount rate to discount a firm's cash flows to their present value and determine the risk
that investor should accept for a particular investment.
Question 5] How expected cash flow are calculated by assigning probabilities to estimated cash flows
in capital budgeting?
Ans.: The concept of probability is fundamental to the use of the risk analysis techniques. It may be
defined as the likelihood of occurrence of an event. If an event is certain to occur, the probability of its
occurrence is one but if an event is certain not to occur, the probability of its occurrence is zero. Thus,
probability of all events to occur lies between zero and one.
Probability distribution can be used to compute expected values. For this purpose following procedure is
adopted:
Step 1: Establish probability distribution
Step 2: Multiply values with probability of each outcome
Step 3: Aggregate the result of Step 2
Illustration: X Ltd. is considering to start a new project for which it has gathered following data:
Cash flow Probability
30,000 0.1
60,000 0.4
1,20,000 0.4
1,50,000 0.1
Calculate the expected cash flow.
Solution:
Cash flow Probability Expected Cash flow
3,000 0.1 300
6,000 0.4 2,400
12,000 0.4 4,800
15,000 1,500
0.1
CF = 9,000
Question 6] How standard deviation (i.e. risk) and coefficient of variance of project is calculated in
capital budgeting?
Ans.: Standard deviation is a statistical measure of dispersion. It measures the deviation from a central
number ie. mean. By calculating standard deviation in capital budgeting, we can measure in each case the
extent of variation. Higher the standard deviation, higher is the risk associated with the project.
However, wherever returns are expressed in revenue terms the co-efficient of variation gives better
measurement for risk evaluation.
Coefficient of variation is calculated as follows:
Coefficient of variation = (??)/NPV
Procedure to calculate standard deviation can be explained with the help of following illustration:
Illustration 1: X Ltd. is considering to start a new project for which it has gathered following data:
NPV Probability
80,000 0.3
1,10,000 0.3
1,42,500 0.2
Compute the risk associated with the project Le. standard deviation.
Solution:
NPV Probability Expected NPV
80,000 0.3 24,000
1,10,000 0.3 33,000
1,42,500 0.2 28,500
NPV = 85,500
Calculation of standard deviation of Project A:
NPV D D2 P PEP
80,000 - 5,500 3,02,50,000 0.3 90,75,000
1,10,000 24,500 60,02,50,000 0.3 18,00,75,000
1,42,500 57,000 3,24,90,00,000 0.2 64,98,00,000
2 = 83,89,50,000
= 28,965
Coefficient of variation = /NPV = 28,965/85,500 = 0.34
Illustration 2: A company is considering Projects X and Y with following information:
Project Expected NPV (`) Standard deviation

X 18,000 6,500
Y 22,000 7,200
Which project will you recommend?
Solution:
On the basis of information about standard deviation of Project X & Y, the Project X is better as it has
lower standard deviation (Le. risk). However, the coefficient of variation for these projects may be found
as follows:
Coefficient of variation = /NPV
Project X = 6,500/18,000 =0.361 7,200
Project Y = 7,200/22,000 = 0.327
Project Y is better as its CV is lesser than Project X.
Question 7] Write a short note on: Decision tree technique in capital budgeting
Ans.: Decision tree technique is a method to evaluate risky proposals. A decision tree shows the sequential
outcome of a risky decision. The decision tree approach gets its name because of resemblance with a
tree having number of branches. A capital budgeting decision tree shows the cash flows and net present
value of the project under differing possible circumstances.
Illustration: A company has made following estimates if the CFAT of the proposed project. The company
use decision tree analysis to get clear picture of project's cash inflow. The project cost ` 80,000 and the
expected life of the project is 2 years. The net cash inflows are:
In Year 1, there is 0.4 probability that CFAT will be ^ 50,000 and 0.6 probability that CFAT will be ` 60,000.
The probabilities assigned to CFAT for the Year 2 are as follows:

If CFAT = ` 50,000 If CFAT = ` 60,000


Probability Probability
24,000 0.2 40,000 0.4
32,000 0.3 50,000 0.5
44,000 0.5 60,000 0.1
The firm uses 10% discount rate for this type of investments. Solution: Decision Tree:

Net present value of cash flows:


Combination CFAT, PV Factor PV, CFAT, PV Factor PV2
A 50,000 0.909 45,450 24,000 0.826 19,824
B 50,000 0.909 45,450 32,000 0.826 26,432
C 50,000 0.909 45,450 44,000 0.826 36,344
D 60,000 0.909 54,540 40,000 0.826 33,040
E 60,000 0.909 54,540 50,000 0.826 41,300
F 60,000 0.909 54,540 60,000 0.826 49,560
Table continued.....
Combination Total PV (PV1 + Initial NPV Joint Expected NPV
PV2) investment Probabilities
A 65,274 80,000 - 14,276 0.08 - 1,178
B 71,882 80,000 -8,118 0.12 -974
C 81,794 80,000 1,794 0.20 358
D 87,850 80,000 7,580 0.24 1,819
E 95,840 80,000 15,840 0.30 4,752
F 1,04,100 24,100 1,446
80,000 0.06
6,223
If worst outcome is realized, the NP V of the project would be ? - 14,726. Probability of this outcome is
0.08. If best outcome is realized, the NPV of the project would be ` 24,100. Probability of this outcome is
0.06. Analysis: Since expected NPV is positive, project can be accepted.
Question 8] Write a short note on: Sensitivity analysis in capital budgeting CS (Professional) - Dec 2013
(5 Marks), June 2015 (4 Marks)
Ans.: A sensitivity analysis is a technique used to determine how different values of an independent
variable impact a particular dependent variable under a given set of assumptions. This technique is used
within specific boundaries that depend on one or more input variables.
In relation to capital budgeting, sensitivity analysis deals with the consideration of sensitivity of the NPV
to different variables contributing to the NPV
Steps to find out sensitivity of NPV to different variables:
(1) Cash flows for the project are identified. NPV is calculated on the basis of these cash flows.
(2) Variables which have a bearing on the cash flows of project are identified. Some of these variables
may be selling price, operating costs, discounting factor, initial cash outflows etc.
(3) , Perform the calculation for variables to find out how much change is required. For example, if one
has to calculate sensitivity of NPV to selling price, then calculation has to be done to see that how much
change in selling price will make the NPV of the project zero.
Advantages of sensitivity analysis:
(a) It compels the decision maker to identify the variables which affect the cash flow forecasts. This
helps him in understanding the investment project in totality.
(b) It indicates the critical variables for which additional information may be obtained. The decision
maker can consider actions which may help in strengthening the “weak spots” in the project.
(c) It helps to expose inappropriate forecasts and thus guides the decision maker to concentrate on
relevant variables.
Disadvantages of sensitivity analysis:
(i) It does not provide clear cut results. The terms ‘optimistic’ and ‘pessimistic’ could mean different things
to different people.
(ii) It fails to focus on the interrelationship between underlying variables. For example sales volume may
be related to price and cost but we analyze each variable differently.
(iii) Sensitivity analysis does not eliminate risk. It merely measures the resilience level of identified
variables' to change in assumption.
Question 9] Distinguish between: Risk Evaluation & Sensitivity Analysis
CS (Professional) - Dec 2012 (5 Marks)
Ans.: Risk analysis gives management better information about the possible outcomes that may occur so
that management can use their judgment and experience to accept an investment or reject it.
In relation to capital budgeting, sensitivity analysis deals with the consideration of sensitivity of the NPV
to different variables contributing to the NPV.
Question 10] Distinguish between: Sensitivity Analysis & Scenario Analysis
CS (Professional) - June 2014 (4 Marks)
Ans.: In relation to capital budgeting, sensitivity analysis deals with the consideration of sensitivity of the
NPV to different variables contributing to the NPV.
Sensitivity analysis takes care of only one or two variable which is at times inadequate. This limitation is
partially overcome by what is known as scenario analysis, where scenario of certain prices, cost and other
variables are created and the financial parameters are computed.
Question 11] Write a short note on: Simulation for risk evaluation
Ans.: Simulation is known as simulated sampling or more fully Monte-Carlo simulation is as much an art
as a technique. It has been described as “what to do when all else fails". Some investment projects may
depend on so many stochastic variables that analytical results are unobtainable. In simulation a
mathematical model is constructed and artificial data is fed. The desired parameters of the system are
then determined from the output of the mode.
Simulation like sensitivity analysis is not an optimizing technique. It merely provides a convenient
representation of reality in some more advanced work than can be used to improve NPV by adjusting
certain variables. There are two important things in simulation viz, the construction of the model and the
judgment of changes to be made to controllable variables.
PROBLEMS & SOLUTIONS - BASIC
Problem No. 1] NZ Ltd. is examining two mutually exclusive projects. The management of the company
use Certainty Equivalent (CE) approach to evaluate projects.
Following information is available for the two proposed projects:
Year Project A Project B
CFAT CE CFAT CE
1 1,15,000 0.90 1,09,000 0.90
2 1,15,000 0.85 1,18,000 0.85
3 1,15,000 0.75 1,12,000 0.80
4 1,15,000 0.70 1,20,000 0.75
5 1,15,000 0.65 1,25,000 0.60

Both projects requires initial investment of ` 3,00,000. The Company’s cost of capital is 12% and risk free
borrowing rate is 7%.
Advise the company which project should be taken by it.
Ans.:
Project A:
Year CFAT CE Adjusted PV
CFAT PV Factor 7%
1 1,15,000 0.90 1,03,500 0.935 96,773
2 1,15,000 0.85 97,750 0.873 85,336
3 1,15,000 0.75 86,250 0.816 70,380
4 1,15,000 0.70 80,500 0.763 61,422
5 1,15,000 0.65 74,750 0.713 53,297
Total present value 3,67,208
(-) Initial Investment (3,00,000)
Net present value 67,208
Project B:
Adjusted
Year CFAT CE PV Factor 7% PV
CFAT
1 1,09,000 0.90 98,100 0.935 91,724
2 1,18,000 0.85 1,00,300 0.873 87,562
3 1,12,000 0.80 89,600 0.816 73,114
Adjusted
Year CFAT CE PV Factor 7% PV
CFAT
4 1,20,000 0.75 90,000 0.763 68,670
5 1,25,000 0.60 75,000 0.713 53,475
Total present value 3,74,545
(-) Initial Investment (3,00,000)
Net present value 74,545
Analysis: The Company should take Project B.
The given cash flows are multiplied by the certainty equivalent (CE) to convert into certain cash flows.
Thereafter, the NPV is calculated by discounting certain cash flow @ 7% Le. risk free rate.
Problem No. 2] X Ltd. is considering to purchase one of the two machines. Each machine requires
investment of ` 62,500 and is expected to have useful life of 4 years. Sellers of machines have guaranteed
to buy back machine at the end of 4 years for ` 5,000. The management of the company use Certainty
Equivalent (CE) approach to evaluate risky projects.
Year Machine Machine II
CFAT CE CFAT CE
1 37,500 0.8 22,500 0.9
2 37,500 0.7 45,000 0.8
3 37,500 0.6 30,000 0.7
4 37,500 0.5 40,000 0.4
The Company’s risk adjusted discount rate is 16% and risk free rate is 10%. Which machine should be
purchased by X Ltd.?
Ans.: Machine I:
Year TAT CE Adjusted PV Fa PV
CFAT
1 37,500 0.8 30,000 0.909 27,270
2 37,500 0.7 26,250 0.826 21,683
3 37,500 0.6 22,500 0.751 16,898
4 37,500' 0.5 18,750 0.683 12,806
4 5,000 1.0 5,000 0.683 3,415
Total present value 82,072
(-) Initial Investment (62,500)
Net present value 19,572
Machine II:
Year CFAT CE Adjusted CFAT PV Factor 10% PV
1 22,500 0.9 20,250 0.909 18,407
2 45,000 0.8 36,000 0.826 29,736
3 30,000 0.7 21,000 0.751 15,771
4 40,000 0.4 16,000 0.683 10,928
4 5,000 1.0 5,000 0.683 3,415
Total present value 78,257
(-) Initial Investment (62,500)
Net present value 15,757
Analysis: The Company should select Machine I.
Problem No. 3] Lucky Ltd. is considering a proposal to buy one of the two machines to manufacture a new
product. Each machine requires investment of ` 1,37,500 and have useful life of 12 years. The Company
has made following estimates associated with these two machines.
Machine X Machine Y
CFAT:
Pessimistic 22,000 0
Most likely 33,000 27,500
Optimistic 44,000 55,000
The Company’s cost of capital is 14% and risk free borrowing rate is 7%. Which machine it should be
purchased by the company?
Ans.:
Machine X:
Estimates CFAT PV Factor 14% PV Cash outflow NPV
Pessimistic 22,000 5.660 1,24,520 1,37,500 (12,980)
Most likely 33,000 5.660 1,86,780 1,37,500 49,280
Optimistic 44,000 5.660 2,49,040 1,37,500 1,11,540
Machine Y:
Estimates CFAT PV Factor 14% PV Cash outflow NPV
Pessimistic 0 5.660 0 1,37,500 (1,37,500)
Most likely 27,500 5.660 1,55,650 1,37,500 18,150
Optimistic 55,000 5.660 3,11,300 1,37,500 1,73,800

Analysis: Purchasing Machine Y will be more risky as NPV is negative as high as ` 1,37,500 while, if Machine
X is purchased, the NPV can be negative only ` 12,980. Hence, it advised to purchase Machine X.
Problem No. 4] Zebra Ltd. is considering two mutually exclusive Projects A & B. Project A cost ` 30,000 and
Project B ` 36,000. NPV probability distribution of the two projects is as follows:
Project A
NPV Probability
3,000 0.1
6,000 0.4
12,000 0.4
15,000 0.1

Project B
NPV Probability
3,000 0.2
6,000 0.3
12,000 0.3
15,000 0.2
You are required to:
(i) Compute the expected NPV of Projects A & B.
(ii) Compute the risk associated with each project i.e. standard deviation,
(iii) Advise as to which project is more risky and why.
(iv) Compute the profitability index of each project.
Ans.:
Calculation of expected NPV for each Project:
Project A
NPV Probability Expected NPV
3,000 0.1 300
6,000 0.4 2,400
12,000 0.4 4,800
15,000 0.1 1,500
NPV = 9,000

Project B
NPV Probability Expected NPV
3,000 0.2 600
6,000 0.3 1,800
12,000 0.3 3,600
15,000 0.2 3,000
NPV = 9,000
Calculation of standard deviation of Project A:
NPV D IP P PD2
3,000 - 6,000 3,60,00,000 0.1 36,00,000
6,000 - 3,000 90,00,000 0.4 36,00,000
12,000 3,000 90,00,000 0.4 36,00,000
15,000 6,000 3,60,00,000 0.1 36,00,000
a2 = 1,44,00,000
a= 3,795
Coefficient of variation = /NPV = 3,795/9,000 = 0.42
Calculation of standard deviation of Project B:
NPV D D2 P PD2
3,000 - 6,000 3,60,00,000 0.2 72,00,000
6,000 - 3,000 90,00,000 0.3 27,00,000
12,000 3,000 90,00,000 0.3 27,00,000
15,000 6,000 3,60,00,000 0.2 72,00,000
a2 — 1,98,00,000
o= 4,450
Coefficient of variation = /NPV = 4,450/9,000 = 0.49
Analysis: Project Y is more risky because of higher coefficient of variation.
Calculation of profitability index:
Particulars Project A Project B
Expected NPV 9,000 9,000
(+) Initial investment 30,000 36,000
Total present value 39,000 45,000

Profitability Index = Total present value/Initial Investment


Project A = 39,000/30,000 = 1.3
Project B = 45,000/36,000 = 1.25
Problem No. 5] A company is considering Projects X and Y with following information:
Project Expected Standard
NPV ) deviation
X 1,22,000 90,000
Y 2,25,000 1,20,000
(i) Which project will you recommend based on the above data?
(ii) Explain whether your opinion will change, if you use coefficient of variation as a measure of risk.
Ans.: On the basis of information about standard deviation of Projects X & Y, the Project X is better as it
has lower standard deviation (Le. risk). However, the coefficient of variation for these projects may be
found as follows:
Coefficient of variation = /NPV
Project X = 90,000/1,22,000 = 0.74
Project Y= 1,20,000/2,25,000 =0.53
Project Y is better as its CV is lesser than Project X.
Problem No. 6] Bipasha Ltd. is considering the renovation of its one of department. The renovation will
cost ` 10 lakh. Its CFAT sensitive to various events as show below:
Event Probability Incremental CFAT
I 0.1 2,00,000
II 0.2 3,00,000
III 0.4 4,00,000
IV 0.2 5,00,000
V 0.1 8.00,000
The company estimates that the probability distribution of incremental CFAT will exists for 4 years. The
company’s cost of capital is 10%.
(1) What is expected annual incremental CFAT?
(2) What is the project’s expected NPV?
(3) If the standard deviation of project is ` 1,80,000, what is project’s risk per rupee of expected return?
Ans.: Calculation of expected annual incremental CFAT:
Incremental CFAT Probability Expected CFAT
2,00,000 0.1 20,000
3,00,000 0.2 60,000
4,00,000 0.4 1,60,000
5,00,000 0.2 1,00,000
8,00,000 0.1 80,000
4,20,000
Calculation of expected NPV:
Year Expected CFAT PV Factor 10% PV
1 to 4 4,20,000 3.170 13,31,400
(-) Initial investment (10,00,000)
Expected NPV 3,31,400
Calculation of projects risk per rupee of expected return:
Risk per rupee of expected return is given by the coefficient of variation.
Coefficient of variation = /NPV = 1,80,000/3,31,400 = 0-543
Problem No. 7] Following information is available in respect of project:
Initial investment : 12,00,000
Useful life : 6 years
Salvage value : Nil
Depreciation : SLM
Risk free rate . : 8%
Certainty Factor assigned to cash inflow : 0.9
Tax rate : 35%
Cash earnings before depreciation in each next 6 years are as follows:
Probability
0.4 6,00,000
0.2 2,00,000
0.4 4,00,000
You are required to:
(1) Calculate CFAT prior to any risk adjustment
(2) Calculate risk adjusted NPV for the project
(3) Calculate risk adjusted IRR
(4) Decide whether project can be accepted. Ans.:
Particulars `
Expected cash earnings before depreciation 4,40,000
(6,00,000 X 0.4) + (2,00,000 X 0.2) + (4,00,000 X 0.4)
(-) Depreciation (2,00,000)
-
?
Particulars
EBT 2,40,000
(-) Tax @35% (84,000)
PAT 1,56,000
(+) Depreciation 2,00,000
CFAT 3,56,000
Calculation of risk adjusted NPV for the project:
Year CFAT CE Adjusted CFAT PV Factor 8% PV
1 to 6 3,56,000 0.9 3,20,400 4.623 14,81,209
(-) Initial Investment (12,00,000)
Net present value 2,81,209

Year CFAT CE Adjusted CFAT PV Factor 16% PV


1 to 6 3,56,000 0.9 3,20,400 3.685 11,80,674

(-) Initial Investment (12,00,000)


Net present value (19,326)
IRR = 8 + 2,81,209/2,81,209 + 19,326 x 8
= 8 + 2,81,209/3,00,535 X 8
= 8 + 7.49
= 15.49% (Approximate)
Analysis: Since IRR is greater than risk free rate, project can be accepted.
Problem No. 8] Delta Ltd. is considering to start a new project. The project has initial outlay of ` 50,000.
The finance department has estimated following data for projects life of 3 years.
Period 1 Period 2 Period 3
CFAT Probability CFAT Probability CFAT Probability
30,000 0.1 28,000 0.2 50,000 0.3
25,000 0.3 25,000 0.2 35,000 0.3
20,000 0.4 23,000 0.1 30,000 0.3
15,000 0.2 20,000 0.5 25,000 0.1
Probability distributions of cash flow are independent. The Company’s cost of capital is 10% and risk free
rate is 5%.
You are required to:
(a) Determine expected NPV
(b) Determine the standard deviation of NPV
Ans. : Year 1 Year 2 Year 3
CFAT Probability Expected CFAT Probability Expected CFAT Probability Expected
CFAT CFAT CFAT
30,000 0.1 3,000 28,000 0.2 5,600 50,000 0.3 15,000
25,000 0.3 7,500 25,000 0.2 5,000 35,000 0.3 10,500
20,000 0.4 8,000 23,000 0.1 2,300 30,000 0.3 9,000
15,000 0.2 3,000 20,000 0.5 10,000 25,000 0.1 2,500
21,500 22,900 37,000
Calculation of expected NPV:
Year Expected PV
CFAT PV Factor 5%
1 21,500 0.952 20,468
2 22,900 0.907 20,770
3 37,000 0.864 31,968
73,206
(-) Initial Investment (50,000)
Expected NPV 23,206
Calculation of standard deviation of cash flow for year 1:
CFAT D D2 P PD2
30,000 8,500 7,22,50,000 0.1 72,25,000
25,000 3,500 1,22,50,000 0.3 36,75,000
20,000 - 1,500 22,50,000 0.4 9,00,000
15,000 - 6,500 4,22,50,000 0.2 84,50,000
(??) 2,02,50,000
= 4,500
Calculation of standard deviation of cash flow for year 2:
CFAT D D2 P PD2
28,000 5,100 2,60,10,000 0.2 52,02,000
25,000 2,100 44,10,000 0.2 8,82,000
23,000 100 10,000 0.1 1,000
20,000 - 2,900 84,10,000 0.5 42,05,000
(??) 1,02,90,000
(??) 3,208
Calculation of standard deviation of cash flow for year 3:
CFAT D D2 P PD2
50,000 13,000 . 16,90,00,000 0.3 5,07,00,000
35,000 - 2,000 40,00,000 0.3 12,00,000
30,000 - 7,000 4,90,00,000 0.3 1,47,00,000
25,000 - 12,000 14,40,00,000 0.1 1,44,00,000
(??) 8,10,00,000
(??) 9,000

 NPV =9,343
Problem No. 9] A company has made following estimates if the CFAT of the proposed project. The
company use decision tree analysis to get clear picture of project’s cash inflow. The project cost 19,000
and the expected life of the project is 2 years. The net cash inflows are:
In Year 1, there is 0.3 probability that CFAT will be X 6,000 and 0.4 probability that CFAT will be ` 8,000.
The probabilities assigned to CFAT for the Year 2 are as follows:

If CFAT= ` 6,000 If CFAT =1 7,000


Probability Probability
2,000 0.3 4,000 0.2
3,000 0.5 4,000 0.3
4,000 0.2 5,000 0.4
The firm uses 8% discount rate for this type of investments.
You are required to:
(i) Present above information in the form of decision tree.
(ii) Find the NPV under (a) the worst outcome; and (b) the best outcome.
(iii) Decide whether project can be accepted?
Ans.:
(i) Decision Tree:

Net present value of cash flows:


Combination CFAT, PV Factor PV, CFAT, PV Factor PV2
A 6,000 0.926 5,556 2,000 0.857 1,714
B 6,000 0.926 5,556 3,000 0.857 2,571
C 6,000 0.926 5,556 4,000 0.857 3,428
D 8,000 0.926 7,408 4,000 0.857 3,428
E 8,000 0.926 7,408 4,000 0.857 3,428
F 8,000 0.926 7,408 5,000 0.857 4,285
Table continued....
Combination Total PV (PV1 + Initial NPV Joint Expected NPV
PV2) investment Probabilities
A 7,270 9,000 - 1,730 0.09 - 155.70
B 8,127 9,000 - 873 0.15 - 130.95
C 8,984 9,000 - 16 0.06 -0.96
D 10,836 9,000 1,836 0.08 146.88
E 10,836 9,000 1,836 0.12 220.32
F 11,693 9,000 2,693 0.16 430.88
510.47
(ii) If worst outcome is realized, the NPV of the project would be ? - 1,730. If best outcome is realized, the
NPV of the project would be ` 2,693.
(iii) Analysis: Since expected NPV is positive, project can be accepted.
Problem No. 10] The following forecasts are made about a proposal which is being evaluated by a firm:
Initial outlay : ` 30,00,000
Life : 4 years
CFAT :` 11,25,000
PVAF(14% 3) : 2.3216
PVAF(H% 4) : 2.9137
You are required to calculate NPV and analyze the sensitivity with respect to -
(i) Initial outlay
(ii) CFAT
(iii) Discount rate
Ans.:
Calculation of NPV:
Year CFAT PV Factor 14% PV
1 to 4 11,25,000 2.9137 32,77,913
(-) Initial investment (30,00,000)
NPV 2,77,913
Sensitivity with respect to initial outlay:
Since NPV is ` 2,77,913, the outlay can increase from ` 30,00,000 to ` 32,77,913 before NPV becomes zero.
Therefore, there is margin of ` 2,77,913 or 9.26% of the initial outlay.
Margin for initial outlay = 2,77,913/30.00,000 x 100 = 9.26%
Sensitivity with respect to CFAT:
Let the annual cash inflow be ‘x'.
Year CFAT PV Factor 14% PV
1 to 4 X 2.9137 2.9137x
(-) Initial investment (30,00,000)
NPV 0
2.9137x - 30,00,000 = 0
2.9137x = 30,00,000
x = 10,29,619
The CFAT can decrease from the present level of ` 11,25,000 to ` 10,29,619 before NPV becomes zero. So,
the CFAT have a margin of ` 95,381 or 8.48%
Margin for CFAT = 95,381/11,25,000 x 100 = 8.48%
Sensitivity with respect to discount rate:
Let the discount factor be ‘x’.
Year CFAT PV Factor PV
1 to 4 11,25,000 X ll,25,000x
(-) Initial investment (30,00,000)
NPV 0
11,25,000x- 30,00,000 = 0
1,25,000x = 30,00,000
x = 2.6667
The PVAF 2.6667 is found for 4 years in 18% column in the PVAF table. Thus, the discount rate can increase
from the present level of 14% to 18% before the NPV becomes zero. Therefore, there is margin of 4%
(18% - 14%) or 29% (Le. 4/14 X 100).
Problem No. 11] XYZ Ltd. is considering a project for which the following estimates are available:
Initial Cost of the project ` 10,00,000

Sales price/unit `60


Cost/ unit ` 40
Sales volume
Year 1 20,000 units
Year 2 30,000 units
Year 3 30,000 units
Discount rate 10% p.a.
You are required to measure the sensitivity of the project in relation td each of the following parameters.
(a) Sales price/unit
(b) Unit cost
(c) Sales volume
(d) Initial outlay
(e) Project lifetime Taxation may be ignored.
Ans.: Contribution/Revenue per unit = 60 - 40 = 20. Calculation of cash flow per year:
Year 1 = 20,000 X 20 = 4,00,000 Year 2 = 30,000 X 20 = 6,00,000 Year 3 = 30,000 X 20 = 6,00,000
Calculation of NPV:
Year Cash flow PV Factor 1( PV
1 4,00,000 0.909 3,63,600
2 6,00,000 0.826 4,95,600
3 6,00,000 0.751 4,50,600
Total present value 13,09,800
(-) Initial investment (10,00,000)
NPV 3,09,800
Sensitivity as to sale price per unit:
Let the cash flow per unit be 'x' to get the NPV zero.
Year Cash flow PV Factor 10% PV
1 20,000x 0.909 18,180*
2 30,000x 0.826 24,780*
3 30,000x 0.751 22,530*
Total present value 65,490*
(-) Initial investment (10,00,000)
NPV 0
65,490x - 10,00,000 = 0 65,490x= 10,00,000x = 10,00,000x = 15.27
Sale Price - Variable Cost = Cash Flow Sale Price - 40 = 15.27 Sale Price = 55.27
Change in sale price per unit needed to become NPV zero = 60 - 55.27 = 4.73
Percentage wise change in sale price per unit = 4.73/60 x 100 = 7.88%
Sensitivity as to cost per unit:
Sale Price - Variable Cost = Cash Flow 60 - Variable Cost = 15.27 Variable Cost = 44.73
Increase in variable cost per unit needed to become NPV zero = 44.73 - 40 = 4.73
Percentage wise change in variable cost per unit = 4.73/40 X 100 = 11.825%
Sensitivity as to sales volume:
The required fall in sales volume to get NPV zero = 13,09,800 - 10,00,000 = 3,09,800
Percentage wise change in sale volume = 3,09,800/13,09,800 X 100 = 23.65%
Sensitivity as to initial outlay:
PV of cash inflow is ` 13,09,800 so initial outlay can be increased by ` 3,09,800 to get the NPV zero. Increase
need in initial outlay to become NPV zero = 13,09,800 - 10,00,000 = 3,09,800
Percentage wise change in initial outlay = 3,09,800/10,00,000 x 100 = 30.98%
Sensitivity as to project lifetime:
Calculation of discounted payback period:
Year Present Value Cumulative PV
1 3,63,600 3,63,600
2 4,95,600 8,59,200
3 4,50,600 13,09,800
Discounted payback period = 2 years + 10,00,000-8,59,200/4,50,600 x 12
= 2 years & 3.75 months
Therefore, if the project runs for 2 years & 3.75 months, break-even would be achieved.
Problem No. 12] Uncertain Ltd. is evaluating a new equipment to replace its old equipment to meet the
greater demand for its product. The new equipment will cost 2,80,000 and the scrap value of old
equipment is estimated to ` 1,12,000. However, due to uncertainty in the expected demand for the
product, the cash flow cannot be accurately estimated. Following probabilities have been assigned in this
respect:
Year 1 Year 2 Year 3
CFAT Probability CFAT Probability CFAT Probability
40,000 0.3 40,000 0.1 40,000 0.3
60,000 0.4 80,000 0.2 80,000 0.5
80,000 0.3 1,20,000 0.4 1,20,000 0.2
- - 1,60,000 0.3 - —
The cost of capital of Uncertain Ltd. is 15%.
Random numbers for 5 sets of cash flow are given below:
Year Set 1 Set 2 Set 3 Set 4 Set 5
1 4 7 6 5 0
2 2 4 8 0 1
3 7 9 4 0 3
You are required to analyze the risk inherent by simulating the NPV values. Also find out the expected
NPV and the probability that the NPV of the proposal will be less than zero.
Ans.: Calculation of expected CFAT:
Year 1 Year 2 Year 3
CFAT Probability Expected CFAT Probability Expected CFAT Probability Expected
CFAT CFAT CFAT
40,000 0.3 12,000 40,000 0.1 4,000 40,000 0.3 12,000
60,000 0.4 24,000 80,000 0.2 16,000 80,000 0.5 40,000
80,000 0.3 24,000 1,20,000 0.4 48,000 1,20,000 0.2 24,000
- - - 1,60,000 0.3 48,000 - - -
60,000 1,16,000 76,000
Calculation of expected NPV:
Year Expected
PV
CFAT PV Factor 15%
1 60,000 0.870 52,200
2 1,16,000 0.756 87,696
3 76,000 0.658 50,008
Total present value 1,89,904
(-) Initial Investment (2,80,000 -1,12,000) (1,68,000)
Expected NPV 21,904
Allocation of random numbers to cash flow:
Year CFAT Probabilities Appearance
No.
40,000 0.3 0,1,2
1 60,000 0.4 3,4,5,6
80,000 0.3 7,8,9
40,000 0.1 0
2 80,000 0.2 1,2
1,20,000 0.4 3,4,5,6
1,60,000 0.3 7,8,9
40,000 0.3 0,1,2
3 80,000 0.5 3,4,5,6,7
1,20,000 0.2 8,9
Note: See explanation at the end of solution as to how “Appearance No.” has been allocated.
Application of simulation process by taking random numbers given in problem:
Year 1 (0.870) Year 2 (0.756) Year 3 (0.658)
App. App.
Set App. No. CFAT PV CFAT PV CFAT PV
No. No.
1 4 60,000 52,200 2 80,000 60,480 7 80,000 52,640
2 7 80,000 69,600 4 1,20,000 90,720 9 1,20,000 78,960
3 6 60,000 52,200 8 1,60,000 1,20,960 4 80,000 52,640
4 5 60,000 52,200 0 40,000 30,240 0 40,000 26,320
5 0 40,000 34,800 1 80,000 60,480 3 80,000 52,640
Set Total PV Cash outflow NPV
1 1,65,320 1,68,000 (2,680)
2 2,39,280 1,68,000 71,280
3 2,25,800 1,68,000 57,800
4 1,08,760 1,68,000 (59,240)
5 1,47,920 1,68,000 (20,080)
Average NPV = 47,080/5 = 9,416
Analysis: Out of 5 situations, Set 2 & 3 has positive NPV and Set 1, 4 & 5 has negative NPV. Therefore,
there is a 60% probability (Le. 3 out of 5) that NPV of the project may turn to be negative i.e. below zero.
Note: In Year 1 for cash flow of ` 40,000 probability is 0.3 which means this cash flow may appear 3 times
out of 10 and hence out of 10 possible outcomes Le. 0-9 first three numbers Le. 0, 1, 2 have been assigned
to cash flow of ` 40,000.
In Year 1 for cash flow of ` 60,000 probability is 0.4 which means this cash flow may appear 4 times out of
10 and hence out of 10 possible outcomes Le. 0-9 next four numbers Le. 3,4,5,6 have been assigned to
cash flow of ` 60,000.
In Year 1 for cash flow of ` 80,000 probability is 0.3 which means this cash flow may appear 3 times out of
10 and hence out of 10 possible outcomes Le. 0-9 next three numbers Le. 7,8,9 have been assigned to
cash flow of ` 60,000.
Same procedure has been adopted for Year 2 & 3.
PAST EXAMINATIONS - PROBLEMS & SOLUTIONS
Problem No. 13] Fast-run Automobiles Ltd. is considering investment in one of three mutually exclusive
projects Zeta-10, Meta-10, & Neta-10. The company’s cost of capital is 15% and the risk-free rate is 10%.
The income-tax rate for the company is 40%. FASL has gathered the following information for each project:
Zeta-10 Meta-10 Neta-10

` ` `
Initial investment 15,00,000 11,00,000 19,00,000
CFAT:
1 6,00,000 . 6,00,000 4,00,000
2 6,00,000 4,00,000 6,00,000
3 6,00,000 5,00,000 8,00,000
4 6,00,000 2,00,000 12,00,000
Risk index 1.80 1.00 0.60
Using risk adjusted discount rate, determine the risk adjusted NPV for each projects. Which project should
be accepted by the company? Why?
[CS (Professional) - Dec 2001] (10 Marks)
Ans.: Calculation of risk adjusted discount rates for various projects:
Formula = Rf + [3 (Rm - Rf)
Zeta-10 = 10+1.80(15-10)= 19%
Meta-10 = 10+1.00(15-10)= 15%
Neta-10 = 10 + 0.60(15-10)= 13%
Calculation of NPV of each project:
Zeta-10 Meta-10 Neta-10
CFAT PV PV CFAT PV PV CFAT PV PV
Year Factor Factor Factor
19% . 1596 1396
1 6,00,000 0.840 5,04,000 6,00,000 0.870 5,22,000 4,00,000 0.885 3,54,000
2 6,00,000 0.706 4,23,600 4,00,000 0.756 3,02,400 6,00,000 0.783 4,69,800
3 6,00,000 0.593 3,55,800 5,00,000 0.658 3,29,000 8,00,000 0.693 5,54,400
4 6,00,000 0.499 2,99,400 2,00,000 0.572 1,14,400 12,00,000 0.613 7,35,600
Total present value 15,82,800 12,67,800 21,13,800
(-) Initial investment (15,00,000) (11,00,000) (19,00,000)
Net present value 82,800 1,67,800 2,13,800
Analysis: Project Neta-10 will be selected on account of highest NPV.
Problem No. 14] Delta Corporation is considering an investment in one of the two mutually exclusive
proposals -
Project-A: It involves initial outlay of ` 1,70,000.

Project-B: It involves initial outlay of ` 1,50,000.


The certainty equivalent approach is employed in evaluating risky investments. The current yield on
treasury bill is 5% and the company uses this as riskless rate. Expected values of net cash inflow with their
respective certainty equivalent are:
Project-A Project-B
Year Cash inflow Certainty Equivalent Cash inflow Certainty Equivalent
1 90,000 0.8 90,000 0.9
2 1,00,000 0.7 90,000 0.8
3 1,10,000 0.5 1,00,000 0.6
Answer the following with reasons:
(i) Which project should be acceptable to the company?
(ii) Which project is riskier and why? Explain.
(iii) If the company was to use the risk-adjusted discount rate method, which project would be analyzed
with higher rate?
[CS (Professional) - June 2003] (8 Marks)
Ans.:
Project-A:
Adjusted
Year CFAT CE PV Factor 596 PV
CFAT
1 90,000 0.8 72,000 0.952 68,544
2 1,00,000 0.7 70,000 0.907 63,490
3 1,10,000 0.5 55,000 0.864 47,520
Total present value 1,79,554
(-) Initial Investment (1,70,000)
Net present value 9,554
Project-B:
Year CFAT CE Adjusted PV Factor 5% PV
CFAT
1 90,000 0.9 81,000 0.952 77,112
2 90,000 0.8 72,000 0.907 65,304
3 1,00,000 0.6 60,000 0.864 51,840
Total present value 1,94,256
(-) Initial Investment (1,50,000)
Net present value 44,256
(i) The Company should select Project-B on account of higher NPV.
(ii) Project-A is more riskier as its certainty equivalents are lower.
(iii) Project-A being more risky, it would be analyzed with higher discount rate.
Problem No. 15] Venkatesh Ltd. is always discarding old lines and introducing new lines of products and
is considering at present three alternative promotional plans for ushering in new products. Various
combinations of prices, development expenditure and promotional outlays are involved in these plans.
High, medium and low forecast of revenues under each plan have been formulated and their respective
probabilities of occurrence have been estimated. Their budgeted revenues and probabilities along with
other relevant data are summarized as under:
(Rupees in Lakhs)
Particulars Plan-I Plan-II Plan-Ill
Budgeted revenue with probability:
High 30 (0.3) 24 (0.2) 50 (0.2)
Medium 20 (0.3) 20 (0.7) 25 (0.5)
Low 5 (0.4) 15 (0.1) 0 (0.3)
Variable cost as percentage of revenue 60% 75% 70%
Initial investment 25 20 24
Life in years 8 8 8
The company’s cost of capital is 12% and the income-tax rate is 40%. Investment in promotional
programmes will be amortized by the straight line method. The company will have net taxable income
each year, regardless of the success or failure of the new products.
(i) Substantiating with figures, make a detailed analysis and find out which of the promotional plans is
expected to be the most profitable?
(ii) In the worst event, which of the plans would result in maximizing the profits?
[CS (Professional) - Dec 2003] (20 Marks)
Ans.: Calculation of CFAT for each plan:
(` in Lakhs)
Particulars Plan-I Plan-II Plan-HI
Budgeted revenue: (Revenue X Probability) High 9.00 4.80 10.00
Medium 6.00 14.00 12.50
Low 2.00 1.50 0
17.00 20.30 22.50
(-) Variable cost (10.20) (15.225) (15.75)
(-) Depreciation (3.125) (2.50) (3.00)
Particulars Plan-I Plan-II Plan-Ill
EBT 3.675 2.575 3.75
(-) Tax @ 40% (1.47) (1.03) (1.50)
PAT 2.205 1.545 2.25
(+) Depreciation 3.125 2.50 3.00
CFAT 5.33 4.045 5.25
Calculation of NPV of each Plan:
Year CFAT PV PV
Plan-I Plan-II Plan-Ill Factor 12% Plan-I Plan-II Plan-Ill
1 to 8 5.33 4.045 5.25 ' 4.968 26.48 20.10 26.08
(-) Initial Investment (25) (20) (24)
Net present value 1.48 0.10 2.08
Analysis: Plan HI should be selected on account of highest NPV.
Problem No. 16] Determine the risk adjusted net present value of the following projects:
Project-A Project-B Project-C

Net cash outlay (`) 1,00,000 1,20,000 2,10,000


Project life (Years) 5 5 5

Annual cash inflow (`) 30,000 42,000 70,000


Coefficient of variation 0.4 0.8 1.2
The company selects the risk adjusted rate of discount on the basis of coefficient of variation.
Coefficient of Variation Risk Adjusted Rate of Discount
0.0 10%
0.4 12%
0.8 14%
1.2 16%
1.6 18%
2.0 22%
More than 2.0 25%
[CS (Professional) - June 2005] (10 Marks)
Ans.:
Calculation of NPV of each project:
Project-A Project-B Project-C
PV PV
Year PV CFAT PV CFAT PV
CFAT PV Factor Factor Factor
12% 14% 16%
1 to 5 30,000 3.605 1,08,150 42,000 3.433 1,44,186 70,000 3.274 2,29,180
(-) Initial investment (1,00,000) (1,20,000) (2,10,000)
Risk adjusted NPV 8,150 24,186 19,180
Note: PV factors are selected on the basis of coefficient of variation.
Problem No. 17] A firm has an investment proposal, requiring an outlay of ` 40,000. The investment
proposal is expected to have 2 years economic life with no salvage value. In Year-1, there is a 0.4
probability that cash flow after tax (CFAT) will be ` 25,000 and 0.6 probability that CFAT will be ` 30,000.
The probabilities assigned to CFAT for the Year-2 are as follows:

If CFAT = ` 25,000 If CFAT = ` 30,000


Probability Probability
12,000 0.2 20,000 0.4
16,000 0.3 25,000 0.5
22,000 0.5 30,000 0.1
The firm uses a 10% discount rate for this type of investment. You are required to -
(i) Present the above information in the form of a decision tree.
(ii) Find out the NPV under (a) the worst outcome; and (b) under the best outcome.
(iii) Find out the profitability or otherwise of the above investment proposal.

[CS (Professional) - June 2006] (20 Marks)


Ans.:
(i) Decision Tree:

Net present value of cash flows:


Combination CFAT, PV Factor PV, CFAT, PV Factor PV,
A 25,000 0.909 22,725 12,000 0.826 9,912
B 25,000 0.909 22,725 16,000 0.826 13,216
C 25,000 0.909 22,725 22,000 0.826 18,172
D 30,000 0.909 27,270 20,000 0.826 16,520
E 30,000 0.909 27,270 25,000 0.826 20,650
F 30,000 0.909 27,270 30,000 0.826 24,780
Table continued.....
Initial investment NPV Joint Probabilities Expected NPV
Total PV (PV1+ PV2)
32,637 40,000 - 7,363 0.08 -589
35,941 40,000 - 4,059 0.12 -487
40,897 40,000 897 0.20 179
43,790 40,000 3,790 0.24 910
47,920 40,000 7,920 0.30 2,376
52,050 40,000 12,050 0.06 723
3.112
(ii) If worst outcome is realized, the NPV of the project would be ? - 7,363. If best outcome is realized, the
NPV of the project would be ^ 12,050.
(iii) Analysis: Since expected NPV is positive, project can be accepted.
Problem No. 18] Ankit Ltd. is considering to take up Project-X or Project-Y. Both the projects have same
life, require equal investment of ` 80 lakh and have almost same yield. An attempt is made to use
Probability Theory to analyze the pattern of cash flow from either project during first year of operation.
This pattern is likely to continue during life of these projects. The results of analysis are as follows:
Project-X Project-Y
Cash Flow ? Probability Cash Flow ? Probability
12,00,000 0.10 8,00,000 0.10
14,00,000 0.20 12,00,000 0.25
16,00,000 0.40 16,00,000 0.30
18,00,000 0.20 20,00,000 0.25
20,00,000 0.10 24,00,000 0.10
80,00,000 1.00 80,00,000 1.00
[CS (Professional) - Dec 2006] (5 Marks)
Ans.: .
Calculation of expected NPV for each Project: in '000)
Project-X Project-Y
NPV Probability Expected NPV NPV Probability Expected NPV
1,200 0.10 120 800 0.10 80
1,400 0.20 280 1,200 0.25 300
1,600 0.40 640 1,600 0.30 480
1,800 0.20 360 2,000 0.25 500
2,000 0.10 200 2,400 0.10 240
NPV = 1,600 NPV = 1,600
Calculation of standard deviation of Project-X:
NPV D D2 P PD2
1,200 -400 1,60,000 0.10 16,000
1,400 -200 40,000 0.20 8,000
1,600 0 0 0.40 0
1,800 200 40,000 0.20 8,000
2,000 400 1,60,000 0.10 16,000
o2 = 48,000
a= 219.09
Coefficient of variation = /NPV = 219.09/1,600 =0.137
Calculation of standard deviation of Project-Y:
NPV D D2 P PD2
800 - 800 6,40,000 0.10 64,000
1,200 -400 1,60,000 0.25 40,000
1,600 0 0 0.30 0
2,000 400 1,60,000 0.25 40,000
2,400 800 6,40,000 0.10 64,000
a2 = 2,08,000
o= 456.07
Coefficient of variation = /NPV = 456.07/1,600 = 0.285
Analysis: Project Y is more risky because of higher coefficient of variation.
Problem No. 19] A company is considering two mutually exclusive projects. The company uses certainty
equivalent approach. Estimated cash flows and certainty equivalents for each project are as follows:
Year Project-1 Project-2
Cash Flow Certainty Cash Flow Certainty
Equivalent Equivalent
0 - 30,000 1.00 - 40,000 1.00
1 15,000 0.95 25,000 0.90
2 15,000 0.85 20,000 0.80
3 10,000 0.70 15,000 0.70
4 10,000 0.65 10,000 0.60
Which project should be accepted, if the risk-free discount rate is 15%?
[CS (Professional) - Dec 2009] (10 Marks)
Ans.: Project-1:
Year CFAT CE Adjusted PV Factor 15% PV
CFAT
1 15,000 0.95 14,250 0.870 12,398
2 15,000 0.85 12,750 0.756 9,639
3 10,000 0.70 7,000 0.658 4,606
4 10,000 0.65 6,500 0.572 3,718
30,361
(-) Initial Investment (30,000)
Net present value 361
Project-2:
Year CFAT CE Adjusted PV Factor 15% PV
CFAT
1 25,000 0.90 22,500 0.870 19,575
2 20,000 0.80 16,000 0.756 12,096
3 15,000 0.70 10,500 0.658 6,909
4 10,000 0.60 6,000 0.572 3,432
42,012
(-) Initial Investment (40,000)
Net present value 2,012
Analysis: The Company should select Project-2 as it has higher NPV than Project-1.
Problem No. 20] Surya Manufacturers is planning to start a new manufacturing process. Following are the
estimated net cash flows and probabilities of the new manufacturing process:
Year Net Cash Flows (^)
0.2
P =

P= 0.6 P= 0.2
0 - 2,00,000 - 2,00,000 - 2,00,000
1 40,000 60,000 80,000
2 40,000 60,000 80,000
3 40,000 60,000 80,000
4 40,000 60,000 80,000
5' 40,000 60,000 80,000
(Salvage) 5 0 40,000 60,000
Surya Manufacturers cost of capital for an average risk project is 10%.
(a) The project has average risk. Find the project’s NPV.
(b) Find the best case and worst case NPVs. What is the probability of occurrence of the worst case if
the cash flows are perfectly dependent (perfectly positively correlated) over time and if they are
independent over time?
(c) Assume that all the cash flows are perfectly positively correlated, that is, there are only three
possible cash flow streams over time: (i) the worst case; (ii) the most likely or base case; and (iii) the best
case with probabilities 0.2, 0.6 and 0.2 respectively. These cases are represented by each of the columns
in the given table. Find the expected NPV, the standard deviation and co-efficient of variation.
[CS (Professional) - June 2010] (20 Marks)
Ans.: Calculation of expected cash inflow:
Year P = 0.2 P = 0.6 P = 0.2 Total
1 40,000 0.2 8,000 60,000 0.6 36,000 80,000 0.2 16,000 60,000
2 40,000 0.2 8,000 60,000 0.6 . 36,000 80,000 0.2 16,000 60,000
3 40,000 0.2 8,000 60,000 0.6 36,000 80,000 0.2 16,000 60,000
4 40,000 0.2 8,000 60,000 0.6 36,000 80,000 0.2 16,000 60,000
5 40,000 0.2 8,000 60,000 0.6 36,000 80,000 0.2 16,000 60,000
5 0 0.2 0 40,000 0.6 24,000 60,000 0.2 12,000 36,000
Calculation of expected cash outflow: '
(2,00,000 X 0.2) + (2,00,000 X 0.6) + (2,00,000 X 0.2) = 2,00,000
Calculation of NPV:
Year CFAT PV Factor 10% PV
1 to 5 60,000 3.791 2,27,460
5 36,000 0.621 22,356
2,49,816
(2,00,000)
(-) Initial Investment Net present value 49,816
Calculation of
worst case NPV :
Year CFAT PV Factor 10% PV
1 to 5 40,000 3.791 1,51,640
5 0 0.621 0
1,51,640
(-) Initial Investment (2,00,000)
Net present value (48,360)
Calculation of best case NPV:
Year CFAT PV Factor 10% PV
1 to 5 80,000 3.791 3,03,280
5 60,000 0.621 37,260
3,40,540
(-) Initial Investment (2,00,000)
Net present value 1,40,540

If cash flows are dependant, the probability of the worst case occurring is the probability of getting `
40,000 net cash flow in year 1 is 0.2 i.e. 20%.
Calculation of average case NPV:
Year CFAT PV Factor 10% PV
1 to 5 60,000 3.791 2,27,460
5 40,000 0.621 24,840
Total present value 2,52,300
(-) Initial Investment (2,00,000)
Net present value 52,300
Calculation of expected NPV:
Case NPV Probability Expected NPV
Worst (48,360) 0.2 (9,672)
Average 52,300 0.6 31,380
Best 1,40,540 0.2 28,108
NPV = 49,816
Calculation of standard deviation:
NPV D D2 P PD2
(48,360) -98,176 9,63,85,26,976 0.2 1,92,77,05,395
52,300 2,484 61,70,256 0.6 37,02,154
1,40,540 90,724 8,23,08,44,176 0.2 1,64,61,68,835
o2 = 3,57,75,76,384
a— 59,813
Coefficient of variation = /NPV = 59,813/49,816 = 12
Problem No. 21] The initial investment outlay for a capital investment project of Priyanka Ltd. consists of`
100 lakh for plant & machinery and ` 40 lakh for working capital. Other details are summarized as follows:
Calculation of
worst case NPV :
Year CFAT PV Factor 10% PV
1 to 5 40,000 3.791 1,51,640
5 0 0.621 0
1,51,640
(-) Initial Investment (2,00,000)
Net present value (48,360)
Calculation of best case NPV:
Year CFAT PV Factor 10% PV
1 to 5 80,000 3.791 3,03,280
5 60,000 0.621 37,260
3,40,540
(-) Initial Investment (2,00,000)
Net present value 1,40,540

If cash flows are dependant, the probability of the worst case occurring is the probability of getting `
40,000 net cash flow in year 1 is 0.2 i.e. 20%.
Calculation of average case NPV:
Year CFAT PV Factor 10% PV
1 to 5 60,000 3.791 2,27,460
5 40,000 0.621 24,840
Total present value 2,52,300
(-) Initial Investment (2,00,000)
Net present value 52,300
Calculation of expected NPV:
Case NPV Probability Expected NPV
Worst (48,360) 0.2 (9,672)
Average 52,300 0.6 31,380
Best 1,40,540 0.2 28,108
NPV = 49,816
Calculation of standard deviation:
NPV D D2 P PD2
(48,360) -98,176 9,63,85,26,976 0.2 1,92,77,05,395
52,300 2,484 61,70,256 0.6 37,02,154
1,40,540 90,724 8,23,08,44,176 0.2 1,64,61,68,835
o2 = 3,57,75,76,384
a— 59,813
Coefficient of variation = /NPV = 59,813/49,816 = 12
Problem No. 21] The initial investment outlay for a capital investment project of Priyanka Ltd. consists of`
100 lakh for plant & machinery and ` 40 lakh for working capital. Other details are summarized as follows:
Sales 1 lakh units of output per year for years 1 to 5

Selling price ` 120 per unit


Variable cost ` 60 per unit

Fixed overheads (excluding depreciation) ` 15 lakh per year for years 1 to 5


Rate of depreciation on plant & machinery 25% on WDV method
Salvage value of plant & machinery Equal to the WDV at the end of year 5
Applicable tax rate 40%
Time horizon 5 years
Post-tax cut-off rate 12%.
You are required to:
(i) Calculate the NPV and indicate the financial viability of the project.
(ii) Determine the sensitivity of the project’s NPV under each of the conditions -
(a) Decrease in selling price by 5% and
(b) Increase in variable cost by 10%.
[CS (Professional) - Dec 2012] (16 Marks)
Ans.: Calculation of initial investment:
Cost of plant & machinery 1,00,00,000
(+) Working capital 40,00,000
Net initial investment 1,40,00,000
Calculation of depreciation:
Cost of plant & machinery 1,00,00,000
(-) Depreciation for 1 st year (25,00,000)
75,00,000
(-) Depreciation for 2nd year (18,75,000)
56,25,000
(-) Depreciation for 3rd year (14,06,250)
42,18,750
(-) Depreciation for 4th year (10,54,688)
31,64,062
(-) Depreciation for 5th year (7,91,016)
23,73,046
Profit/loss -
Salvage value at the end of 5th year 23,73,046
Calculation of profit before depreciation & tax:
Selling price per unit 120
(-) Variable cost per unit. (60)
Contribution per unit 60
Total contribution (1,00,000 X 60) 60,00,0000
(-) Fixed cost other than depreciation (15,00,000)
Profit before depreciation & tax 45,00,000

Calculation of CFAT : Year 1 Year 2 Year 3 Year 4 Year 5


Particulars
Profit before depreciation & tax 45,00,000 45,00,000 45,00,000 45,00,000 45,00,000
(-) Depreciation (25,00,000) (18,75,000) (14,06,250) (10,54,688) (7,91,016)
EBT 20,00,000 26,25,000 30,93,750 34,45,312 37,08,984
(-) Tax @ 40% (8,00,000) (10,50,000) (12,37,500) (13,78,125) (14,83,594)
PAT 12,00,000 15,75,000 18,56,250 20,67,187 22,25,390
(+) Depreciation 25,00,000 18,75,000 14,06,250 10,54,688 7,91,016
CFAT 37,00,000 34,50,000 32,62,500 31,21,875 30,16,406
Working capital released - - - - 40,00,000
Salvage value - - - - 23,73,046
Net CFAT 37,00,000 34,50,000 32,62,500 31,21,875 93,89,452
Calculation of NPV:
Year CFAT PV Factor 12% PV
1 37,00,000 0.893 33,04,100
2 34,50,000 0.797 27,49,650
3 32,62,500 0.712 23,22,900
4 31,21,875 0.636 19,85,513
5 93,89,452 0.567 53,23,819
1,56,85,982
(-) Initial Investment (1,40,00,000)
Net present value 16,85,982
Analysis: As NPV is positive, project is financially viable.
Sensitivity of the project’s NPV with respect to decrease in selling price by 5%:
Selling price per unit after decrease of 5% 114
(-) Variable cost per unit (60)
Contribution per unit 54
Total contribution (1,00,000 X 54) 54,00,0000
Decrease in contribution/revenue = 60,00,000 - 54,00,000 — 6,00,000
Tax effect = 6,00,000 (1 - 0.4) = 3,60,000
Thus, revenue for 1 to 5 year will decrease by ` 3,60,000 per year.
Decline in NPV = 3,60,000 X 3.605
Decline in NPV = 12,97,800
Percentage decline in NPV to base NPV = 12,97,800/16,85,982 × 100 = 76.98%
Sensitivity of the project’s NPV with respect to increase in variable cost by 10%:
Selling price per unit 120
(-) Variable cost per unit after 10% increase (66)
Contribution per unit 54
Total contribution (1,00,000 X 54) 54,00,0000
Decrease in contribution/revenue = 60,00,000 - 54,00,000 = 6,00,000
Tax effect = 6,00,000 (1 - 0!4) = 3,60,000
Thus, revenue for 1 to 5 year will decrease by ` 3,60,000 per year.
Decline in NPV = 3,60,000 X 3.605 Decline in NPV = 12,97,800
Percentage decline in NPV to base NPV = 12,97,800/16,85,982 x 100 = 76.98%
Problem No. 22] Simon Ltd. is considering two mutually exclusive projects. Investment outlay of both the
projects is ^ 5 lakh and each is expected to have a life of 5 years. Under three possible situations their
annual cash flows and probabilities are as under:
Situation Probabilities Project-A cash flow (` ) Project-B cash flow (` )
Good 0.3 6 lakh 5 lakh
Normal 0.4 4 lakh 4 lakh
Worse 0.3 2 lakh 3 lakh
If the cost of capital is 7%, which project should be accepted? Consider the risk parameter also in decision
making. Explain with workings.
[CS (Professional) - June 2014 & June 2016] (10 Marks)
Ans.: Calculation of expected cash flow for each Project: (` in lakhs)
Project-A Project-B
Cash flow Probability Expected Cash flow Cash flow Probability Expected Cash flow
6 0.3 1.8 5 0.3 1.5
4 0.4 1.6 4 0.4 1.6
2 0.3 0.6 3 0.3 0.9
NPV = 4.0 NPV = 4 0
Calculation of standard deviation of Project-A:
NPV D D2 P PD2
6 2 4 0.3 1.2
4 0 0 0.4 0
2 -2 4 0.3 1.2
(??) 2.4
(??) 1.55
Coefficient of variation = /NPV = 1.55/4 = 0.3875
Calculation of standard deviation of Project-B:
NPV D D2 P PD2
5 1 1 0.3 0.3
4 0 0 0.4 0
3 -1 1 0.3 0.3
(??) 0.6
(??) 0.78
Coefficient of variation = /NPV = 0.78/4 = 0.195
Analysis: Project-A is more risk because of higher coefficient of variation.
Problem No. 23] Sagar Ltd. has been in IT business for 6 years and enjoys a favourable market reputation.
Corporate tax is 30%. They anticipated that the demand for IT solutions would increase considerably since
many foreign firms are setting-up their BPO centres in India. For an expansion project, they propose to
invest ` 22 Crore to be funded by new debt and equity on 50:50 basis. Enquiries with merchant bankers
reveal that funds can be available at following rates:
Debt Rate

- First ` 5 Crore 10%

- Next ` 5 Crore 12%

All additional funds 15.72%


Equity 12%
Risk gradation by company 2% over cost of capital.
You are required to compute the appropriate risk adjusted discount rate.
[CS (Professional) - Dec 2015] (4 Marks)
Ans.:
Cost of 10% debt: Cost of 12% debt: Cost of 15.72% debt:
Kd = 1(1 - t) Kd = I (1- t) Kd = I (1 -t)
= 10(1-0.3) = 12(1-0.3) = 15.72(1-0.3)
= 7% = 8.4% = 11%

The company propose to invest ` 22 Crore to be funded by new debt and equity on 50:50 basis. Hence, 11
Crore will be funded by debt and remaining 11 Crore will be funded by equity.
Calculation of WACC:
Types of capital ` in Crore % Cost of Capital Product
10% Debt 5 22.73% 7% 159.11
12% Debt 5 22.72% 8.4% 190.85
15.72% Debt 1 4.55% 11% 50.05
Equity 11 50% 12% 600.00
22 100% 1,000.01
WACC = 1,000.01/100 = 10%
Risk adjusted discount rate = 10% + 2% = 12%
Problem No. 24] You are the CFO of Desire Ltd. Your company proposes to buy equipment costing `
1,00,000. The equipment will last for 5 years. The cost of capital to the company is 10%. Your analyst has
suggested the expected revenues, cost and corresponding probabilities as under:
Expected Revenue Expected Cost

Amount (`) Probability Amount (`) Probability


1,00,000 0.15 62,500 0.10
1,25,000 0.40 75,000 0.25
1,37,500 0.30 87,500 0.35
1,50,000 0.15 1,00,000 0.30
You wish to run a simulation model and have picked the random numbers 81, 02, 60, 04, 46, 31, 67, 25 in
that order, alternatively for revenues and costs. Decide whether the project can be undertaken by the
company.
[CS (Professional) - Dec 2015] (8 Marks)
Ans.:
Assignment of cumulative probabilities and random digit allocation:
Revenue Probability Cumulative probabilities Random digit allocation
1,00,000 0.15 0.15 0-14*
1,25,000 0.40 0.55 15-54
1,37,500 0.30 0.85 55-84
1,50,000 0.15 1.00 85-99

Cost Probability Cumulative probabilities Random digit allocation


62,500 0.10 0.10 0-09
75,000 0.25 0.35 10-34
87,500 0.35 0.70 35-69
1,00,000 0.30 1.00 70-99
* Note: Probability is 15%, random number digit should be 15% of the range between 0 and 99.
Hence UC Cl = 0 to 14
Ascertainment of value by using random numbers:
Random numbers Range Revenue
81 55-84 1,37,500
60 55-84 1,37,500
46 15-54 1,25,000
67 55-84 1,37,500
Average Revenue = 5,37,500/4 = 1,34,375
Random numbers Range Cost
02 0-09 62,500
04 0-09 62,500
31 10-34 75,000
25 10-34 75,000
Average Cost = 2,75,000/4 = 68,750
Average cash inflow' = 1,34,375 - 68,750 = 65,625 Calculation of NPV:
Year Cash inflow PV Factor 10% PV
1 to 5 65,625 3.791 2,48,784
(-) Initial Investment (1,00,000)
Net present value 1,48,784
Analysis: As NPV is positive, the project is financially viable.
CHAPTER
14
DIVIDEND POLICY
Question 1] Why dividend policy is important for the firm? State the factors affecting dividend policy?
Ans.: Dividend policy determines what portion of earnings will be paid out to stock holders and what
portion will be retained in the business to finance long-term growth. Dividend constitutes the cash flow
that accrues to equity holders whereas retained earnings are one of the most significant sources of
funds for financing the corporate growth. Both dividend and growth are desirable but are conflicting
goals to each other. Higher dividend means less retained earnings and vice versa. This position is quite
challenging for the finance manager and necessitates the need to establish a dividend policy in the firm
which will evolve a pattern of dividend payments having no

You might also like